P-99: Ninety-Nine Prolog Problems



Authors: Daniela Ferreiro, Werner Hett (original version)

This is a presentation as an Active Logic Document (ALD) of the Ninety-Nine Prolog Problems written by Werner Hett at the Berne University of Applied Sciences (Switzerland). The original can be found here or here. It has been translated since to many programming languages.

Active Logic Documents such as this one (see also this poster for a quick summary) allow direct interaction with logic programming problems and solutions (editing, testing, issuing queries, etc.) while running everything within your browser. I.e., there is no interaction with a server and your programs run locally in an embedded version of (Ciao) Prolog that requires no installation. The document source is written in a markdown dialect. They are particularly useful, e.g., for teaching Prolog, embedding examples in documentation, manuals, tutorials, etc.

How to use this interactive document:

  • The code boxes generally need your interaction. You will identify them by a question mark (?) in the right top corner.
  • Your goal is to turn all question marks in such boxes into green checked marks ():
    • Boxes with a thinking face (🤔) are exercises that you must complete in order to progress. Once an exercise is completed, press the question mark.
    • Code errors or failed exercises turn the question mark into a red erroneous mark (). You can try again, as many times as you want.
    • When available, pressing the northeast pointing arrow () will load the code in a separate Prolog playground window.
  • Would you like to test your code? Feel free to submit your answer and make all the queries you want! In each exercise, there is also an editable box with a right-pointing triangle (▶) that allows interaction with the code loaded, by issuing queries. To use these query boxes, make sure the previous code box is checked () (the one with the proposed activity).

Introduction

The purpose of this problem collection is to give you the opportunity to practice your skills in logic programming. Your goal should be to find the most elegant solution of the given problems. Efficiency is important, but logical clarity is even more crucial. Some of the (easy) problems can be trivially solved using built-in predicates. However, in these cases, you learn more if you try to find your own solution.

Every predicate that you write should begin with a comment that describes the predicate in a declarative statement. Do not describe procedurally, what the predicate does, but write down a logical statement which includes the arguments of the predicate. You should also indicate the intended data types of the arguments and the allowed flow patterns.

The problems have different levels of difficulty. Those marked with a single asterisk (⭐️) are easy. If you have successfully solved the preceding problems you should be able to solve them within a few (say 15) minutes. Problems marked with two asterisks (⭐️⭐️) are of intermediate difficulty. If you are a skilled Prolog programmer it shouldn't take you more than 30-90 minutes to solve them. Problems marked with three asterisks (⭐️⭐️⭐️) are more difficult. You may need more time (i.e. a few hours or more) to find a good solution.

Working with Prolog lists

A list is either empty or it is composed of a first element (head) and a tail, which is a list itself. In Prolog we represent the empty list by the atom [] and a non-empty list by a term [H|T] where H denotes the head and T denotes the tail.

Last element of a list

(Easy ⭐️) Find the last element of a list.

:- module(_, _, [assertions]).

sorry :- throw(not_solved_yet).

:- test my_last(A, B) : (B = [a,b,c,d])
   => (A = d) + (not_fails, is_det).
   
%! \begin{hint}
% Example: 
% ?- my_last(X,[a,b,c,d]).
% A = d

my_last(X,L) :- sorry.
  % X is the last element of the list L

% Note: last(?Elem, ?List) is predefined
%! \end{hint}
%! \begin{solution}
% Example: 
% ?- my_last(X,[a,b,c,d]).
% A = d

my_last(X,[X]).
my_last(X,[_|L]) :- my_last(X,L).
%! \end{solution}
?- my_last(X,[a,b,c,d]).

Last but one element of a list

(Easy ⭐️) Find the last but one element of a list. (zweitletztes Element, l'avant-dernier élément)

:- module(_, _, [assertions]).

sorry :- throw(not_solved_yet).

:- test last_but_one(A, B) : (B = []) + (fails).
   
:- test last_but_one(A, B) : (B = [1,2,3,4])
   => (A = 3) + (not_fails, is_det).
   
%! \begin{hint}
% Example: 
% ?- last_but_one(X,[1,2,3,4]).
% A = 3

last_but_one(X,L) :- sorry.
  % X is the last but one element of the list L

%! \end{hint}
%! \begin{solution}
% Example: 
% ?- last_but_one(X,[1,2,3,4]).
% A = 3

last_but_one(X,[X,_]).
last_but_one(X,[_,Y|Ys]) :- last_but_one(X,[Y|Ys]).
%! \end{solution}
?- last_but_one(X,[a,b,c,d]).

Find element in a list

(Easy ⭐️) Find the K'th element of a list. The first element in the list is number 1.

:- module(_, _, [assertions]).

sorry :- throw(not_solved_yet).

:- test element_at(A, B, C) : (B = [a,b,c,d,e], C = 3) 
   => (A = c) + (not_fails, is_det).
   
%! \begin{hint}
% Example: 
% ?- element_at(X,[a,b,c,d,e],3).
% X = c

element_at(X,L,K) :- sorry.
  % X is the K'th element of the list L

% Note: nth1(?Index, ?List, ?Elem) is predefined
%! \end{hint}
%! \begin{solution}
% Example: 
% ?- element_at(X,[a,b,c,d,e],3).
% X = c

element_at(X,[X|_],1).
element_at(X,[_|L],K) :- K > 1, K1 is K - 1, element_at(X,L,K1).
%! \end{solution}
?- element_at(X,[a,b,c,d,e],3).

Number of elements of a list

(Easy ⭐️) Find the number of elements of a list.

:- module(_, _, [assertions]).

sorry :- throw(not_solved_yet).

:- test my_length(A, B) : (A = [1, 4, 5]) 
   => (B = 3) + (not_fails, is_det).
   
:- test my_length(A, B) : (A = []) 
   => (B = 0) + (not_fails, is_det).
%! \begin{hint}
% Example: 
% ?- my_length([1,2,3],X).
% X = 3

my_length(L,N) :- sorry.
  % the list L contains N elements

% Note: length(?List, ?Int) is predefined
%! \end{hint}
%! \begin{solution}
% Example: 
% ?- my_length([1,2,3],X).
% X = 3

my_length([],0).
my_length([_|L],N) :- my_length(L,N1), N is N1 + 1.
%! \end{solution}
?- my_length([1,2,3],X).

Reverse a List

(Easy ⭐️) Reverse a list.

:- module(_, _, [assertions]).

sorry :- throw(not_solved_yet).

:- test my_reverse(A, B) : (A = [1, 4, 5]) 
   => (B = [5, 4, 1]) + (not_fails, is_det).

:- test my_reverse(A, B) : (A = [a, b, c, d]) 
   => (B = [d, c, b, a]) + (not_fails, is_det).
   
%! \begin{hint}
% Example: 
% ?- my_reverse([1,4,5],X).
% X = [5,4,1]

my_reverse(L1,L2) :- sorry.
  % L2 is the list obtained from L1 by reversing 
  % the order of the elements.

% Note: reverse(+List1, -List2) is predefined
%! \end{hint}
%! \begin{solution}
% Example: 
% ?- my_reverse([1,4,5],X).
% X = [5,4,1]

my_reverse(L1,L2) :- my_rev(L1,L2,[]).

my_rev([],L2,L2) :- !.
my_rev([X|Xs],L2,Acc) :- my_rev(Xs,L2,[X|Acc]).
%! \end{solution}
?- my_reverse([1,4,5],X).

Palindrome

(Easy ⭐️) Find out whether a list is a palindrome. A palindrome can be read forward or backward; e.g. [x,a,m,a,x].

:- module(_, _, [assertions]).

:- use_module(library(classic/classic_predicates), [reverse/2]).

sorry :- throw(not_solved_yet).

:- test is_palindrome(A) : (A = [1, 4, 5]) + (fails).

:- test is_palindrome(A) : (A = [1, 2, 3, 2, 1]) + (not_fails).
   
:- test is_palindrome(A) : (A = []) + (not_fails).
%! \begin{hint}
% Example: 
% ?- is_palindrome([x,a,m,a,x]).
% yes

is_palindrome(L) :- sorry.
  % L is a palindrome list
%! \end{hint}
%! \begin{solution}
% Example: 
% ?- is_palindrome([x,a,m,a,x]).
% yes

is_palindrome(L) :- reverse(L,L).

%! \end{solution}
?- is_palindrome([x,a,m,a,x]).
Hint: Use the predefined predicate reverse/2

Flatten a list

(Intermediate ⭐️⭐️) Flatten a nested list structure. Transform a list, possibly holding lists as elements into a flat list by replacing each list with its elements (recursively).

:- module(_, _, [assertions]).

:- use_module(library(classic/classic_predicates),[append/3]).

sorry :- throw(not_solved_yet).

is_list([]).
is_list([_H|_T]) :- is_list(_T).

:- test my_flatten(A, B) : (A = [a, [b, [c, d], e]]) 
    => (B = [a, b, c, d, e]) + (not_fails, is_det).

%! \begin{hint}
% Example: 
% ?- my_flatten([a, [b, [c, d], e]], X).
% X = [a, b, c, d, e]

my_flatten(L1,L2) :- sorry.
  % the list L2 is obtained from the list L1 by
  % flattening; i.e. if an element of L1 is a list then it is replaced
  % by its elements, recursively. 

% Note: flatten(+List1, -List2) is a predefined predicate
%! \end{hint}
%! \begin{solution}
% Example: 
% ?- my_flatten([a, [b, [c, d], e]], X).
% X = [a, b, c, d, e]

my_flatten(X,[X]) :- \+ is_list(X).
my_flatten([],[]).
my_flatten([X|Xs],Zs) :- my_flatten(X,Y), my_flatten(Xs,Ys), append(Y,Ys,Zs).
%! \end{solution}
?- my_flatten([a, [b, [c, d], e]], X).
Hint: Use the predefined predicates is_list/1 and append/3

Eliminate duplicates

(Intermediate ⭐️⭐️) Eliminate consecutive duplicates of list elements. If a list contains repeated elements they should be replaced with a single copy of the element. The order of the elements should not be changed.

:- module(_, _, [assertions]).

sorry :- throw(not_solved_yet).

:- test compress(A, B) : (A = [1,1,1,2,2,3,2,2,2]) 
    => (B = [1,2,3,2]) + (not_fails, is_det).

:- test compress(A, B) : (A = []) 
    => (B = []) + (not_fails, is_det).
    
%! \begin{hint}
% Example: 
% ?- compress([a,a,a,a,b,c,c,a,a,d,e,e,e,e],X).
% X = [a,b,c,a,d,e]

compress(L1,L2) :- sorry.
  % the list L2 is obtained from the list L1 by
  % compressing repeated occurrences of elements into a single copy
  % of the element.

%! \end{hint}
%! \begin{solution}
% Example: 
% ?- compress([a,a,a,a,b,c,c,a,a,d,e,e,e,e],X).
% X = [a,b,c,a,d,e]

compress([],[]).
compress([X],[X]).
compress([X,X|Xs],Zs) :- 
        compress([X|Xs],Zs).
compress([X,Y|Ys],[X|Zs]) :-
        X \= Y, 
        compress([Y|Ys],Zs).
%! \end{solution}
?- compress([a,a,a,a,b,c,c,a,a,d,e,e,e,e],X).

Pack consecutive duplicates

(Intermediate ⭐️⭐️) Pack consecutive duplicates of list elements into sublists. If a list contains repeated elements they should be placed in separate sublists.

:- module(_, _, [assertions]).

sorry :- throw(not_solved_yet).

:- test pack(A, B) : (A = [a,a,a,a,b,c,c,a,a,d,e,e,e,e]) 
    => (B = [[a,a,a,a],[b],[c,c],[a,a],[d],[e,e,e,e]]) + (not_fails, is_det).

:- test pack(A, B) : (A = [1]) 
    => (B = [[1]]) + (not_fails, is_det).
    
%! \begin{hint}
% Example:
% ?- pack([a,a,a,a,b,c,c,a,a,d,e,e,e,e],X).
% X = [[a,a,a,a],[b],[c,c],[a,a],[d],[e,e,e,e]]

pack(L1,L2) :- sorry.
  % the list L2 is obtained from the list L1 by packing
  % repeated occurrences of elements into separate sublists.
  % (list,list) (+,?)


%! \end{hint}
%! \begin{solution}
% Example:
% ?- pack([a,a,a,a,b,c,c,a,a,d,e,e,e,e],X).
% X = [[a,a,a,a],[b],[c,c],[a,a],[d],[e,e,e,e]]

pack([],[]).
pack([X|Xs],[Z|Zs]) :- transfer(X,Xs,Ys,Z), pack(Ys,Zs).

% transfer(X,Xs,Ys,Z) Ys is the list that remains from the list Xs
% when all leading copies of X are removed and transfered to Z
transfer(X,[],[],[X]).
transfer(X,[Y|Ys],[Y|Ys],[X]) :- X \= Y.
transfer(X,[X|Xs],Ys,[X|Zs]) :- transfer(X,Xs,Ys,Zs).

%! \end{solution}
?- pack([a,a,a,a,b,c,c,a,a,d,e,e,e,e],X).

Run-length encoding

(Easy ⭐️) Run-length encoding of a list. Use the result of Pack consecutive duplicates of list elements into sublists problem to implement the so-called run-length encoding data compression method. Consecutive duplicates of elements are encoded as terms [N,E] where N is the number of duplicates of the element E.

:- module(_, _, [assertions]).

:- use_module(library(classic/classic_predicates),[length/2]).

sorry :- throw(not_solved_yet). 

:- test encode(A, B) : (A = [a,a,a,a,b,c,c,a,a,d,e,e,e,e]) 
    => (B = [[4,a],[1,b],[2,c],[2,a],[1,d],[4,e]]) + (not_fails, is_det).

:- test encode(A, B) : (A = [1,1,1]) 
    => (B = [[3,1]]) + (not_fails, is_det).
    
:- test encode(A, B) : (A = []) 
    => (B = []) + (not_fails, is_det).
pack([],[]).
pack([X|Xs],[Z|Zs]) :- transfer(X,Xs,Ys,Z), pack(Ys,Zs).

transfer(X,[],[],[X]).
transfer(X,[Y|Ys],[Y|Ys],[X]) :- X \= Y.
transfer(X,[X|Xs],Ys,[X|Zs]) :- transfer(X,Xs,Ys,Zs).
%! \begin{hint}
% Example:
% ?- encode([a,a,a,a,b,c,c,a,a,d,e,e,e,e],X).
% X = [[4,a],[1,b],[2,c],[2,a],[1,d],[4,e]]

encode(L1,L2) :- sorry.
  % the list L2 is obtained from the list L1 by run-length
  % encoding. Consecutive duplicates of elements are encoded as terms [N,E],
  % where N is the number of duplicates of the element E.
  % (list,list) (+,?)

%! \end{hint}
%! \begin{solution}
% Example:
% ?- encode([a,a,a,a,b,c,c,a,a,d,e,e,e,e],X).
% X = [[4,a],[1,b],[2,c],[2,a],[1,d],[4,e]]

encode(L1,L2) :- pack(L1,L), transform(L,L2).

transform([],[]).
transform([[X|Xs]|Ys],[[N,X]|Zs]) :- length([X|Xs],N), transform(Ys,Zs).
%! \end{solution}
?- encode([a,a,a,a,b,c,c,a,a,d,e,e,e,e],X).
Hint: Use the predefined predicates length/2

(Easy ⭐️) Modified run-length encoding. Modify the result of Run-length encoding of a list problem in such a way that if an element has no duplicates it is simply copied into the result list. Only elements with duplicates are transferred as [N,E] terms.

:- module(_, _, [assertions]).
:- use_module(library(classic/classic_predicates),[length/2]).

sorry :- throw(not_solved_yet). 

:- test encode_modified(A, B) : (A = [a,a,a,a,b,c,c,a,a,d,e,e,e,e]) 
    => (B = [[4,a],b,[2,c],[2,a],d,[4,e]]) + (not_fails, is_det).

:- test encode_modified(A, B) : (A = [1,1,1]) 
    => (B = [[3,1]]) + (not_fails, is_det).
    
:- test encode_modified(A, B) : (A = []) 
    => (B = []) + (not_fails, is_det).
    
pack([],[]).
pack([X|Xs],[Z|Zs]) :- transfer(X,Xs,Ys,Z), pack(Ys,Zs).

transfer(X,[],[],[X]).
transfer(X,[Y|Ys],[Y|Ys],[X]) :- X \= Y.
transfer(X,[X|Xs],Ys,[X|Zs]) :- transfer(X,Xs,Ys,Zs).

encode(L1,L2) :- pack(L1,L), transform(L,L2).

transform([],[]).
transform([[X|Xs]|Ys],[[N,X]|Zs]) :- length([X|Xs],N), transform(Ys,Zs).
%! \begin{hint}
% Example
% ?- encode_modified([a,a,a,a,b,c,c,a,a,d,e,e,e,e],X).
% X = [[4,a],b,[2,c],[2,a],d,[4,e]]

encode_modified(L1,L2) :- sorry.
  % the list L2 is obtained from the list L1 by 
  % run-length encoding. Consecutive duplicates of elements are encoded 
  % as terms [N,E], where N is the number of duplicates of the element E.
  % However, if N equals 1 then the element is simply copied into the 
  % output list.
  % (list,list) (+,?)
%! \end{hint}
%! \begin{solution}
% Example
% ?- encode_modified([a,a,a,a,b,c,c,a,a,d,e,e,e,e],X).
% X = [[4,a],b,[2,c],[2,a],d,[4,e]]

encode_modified(L1,L2) :- encode(L1,L), strip(L,L2).

strip([],[]).
strip([[1,X]|Ys],[X|Zs]) :- strip(Ys,Zs).
strip([[N,X]|Ys],[[N,X]|Zs]) :- N > 1, strip(Ys,Zs).
%! \end{solution}
?- encode_modified([a,a,a,a,b,c,c,a,a,d,e,e,e,e],X).
Hint: You can use the predefined predicate length/2.

(Intermediate ⭐️⭐️) Run-length encoding of a list (direct solution). Implement the so-called run-length encoding data compression method directly. I.e. don't explicitly create the sublists containing the duplicates, as in Pack consecutive duplicates of list elements into sublists problem, but only count them. Simplify the result list by replacing the singleton terms [1,X] by X.

:- module(_, _, [assertions]).

sorry :- throw(not_solved_yet). 

:- test encode_direct(A, B) : (A = [a,a,a,a,b,c,c,a,a,d,e,e,e,e]) 
    => (B = [[4,a],b,[2,c],[2,a],d,[4,e]]) + (not_fails, is_det).

%! \begin{hint}
% Example:
% ?- encode_direct([a,a,a,a,b,c,c,a,a,d,e,e,e,e],X).
% X = [[4,a],b,[2,c],[2,a],d,[4,e]]

encode_direct(L1,L2) :- sorry.
  % the list L2 is obtained from the list L1 by 
  % run-length encoding. Consecutive duplicates of elements are encoded 
  % as terms [N,E], where N is the number of duplicates of the element E.
  % However, if N equals 1 then the element is simply copied into the 
  % output list.
  % (list,list) (+,?)

%! \end{hint}
%! \begin{solution}
% Example:
% ?- encode_direct([a,a,a,a,b,c,c,a,a,d,e,e,e,e],X).
% X = [[4,a],b,[2,c],[2,a],d,[4,e]]

encode_direct([],[]).
encode_direct([X|Xs],[Z|Zs]) :- count(X,Xs,Ys,1,Z), encode_direct(Ys,Zs).

% count(X,Xs,Ys,K,T) Ys is the list that remains from the list Xs
%    when all leading copies of X are removed. T is the term [N,X],
%    where N is K plus the number of X's that can be removed from Xs.
%    In the case of N=1, T is X, instead of the term [1,X].

count(X,[],[],1,X).
count(X,[],[],N,[N,X]) :- N > 1.
count(X,[Y|Ys],[Y|Ys],1,X) :- X \= Y.
count(X,[Y|Ys],[Y|Ys],N,[N,X]) :- N > 1, X \= Y.
count(X,[X|Xs],Ys,K,T) :- K1 is K + 1, count(X,Xs,Ys,K1,T).
%! \end{solution}
?- encode_direct([a,a,a,a,b,c,c,a,a,d,e,e,e,e],X).

Run-length decoding

(Intermediate ⭐️⭐️) Decode a run-length encoded list. Given a run-length code list generated as specified in Modified run-length encoding problem Construct its uncompressed version.

:- module(_, _, [assertions]).

sorry :- throw(not_solved_yet). 

:- test decode(A, B) : (A =  [[4,a],b,[2,c],[2,a],d,[4,e]]) 
    => (B = [a,a,a,a,b,c,c,a,a,d,e,e,e,e]) + (not_fails, is_det).

:- test decode(A, B) : (A = [[3,1]]) 
    => (B = [1,1,1]) + (not_fails, is_det).
    
:- test decode(A, B) : (A = []) 
    => (B = []) + (not_fails, is_det).

is_list([]).
is_list([_H|_T]) :- is_list(_T).
%! \begin{hint}
% Example
% ?- decode([[4,a],b,[2,c],[2,a],d,[4,e]],X).
% X = [a,a,a,a,b,c,c,a,a,d,e,e,e,e]

decode(L1,L2) :- sorry.
  % L2 is the uncompressed version of the run-length
  % encoded list L1.
  % (list,list) (+,?)
%! \end{hint}
%! \begin{solution}
% Example
% ?- decode([[4,a],b,[2,c],[2,a],d,[4,e]],X).
% X = [a,a,a,a,b,c,c,a,a,d,e,e,e,e]

decode([],[]).
decode([X|Ys],[X|Zs]) :- \+ is_list(X), decode(Ys,Zs).
decode([[1,X]|Ys],[X|Zs]) :- decode(Ys,Zs).
decode([[N,X]|Ys],[X|Zs]) :- N > 1, N1 is N - 1, decode([[N1,X]|Ys],Zs).
%! \end{solution}
?- decode([[4,a],b,[2,c],[2,a],d,[4,e]],X).

Duplicate the Elements of a List

(Easy ⭐️) Duplicate the elements of a list.

:- module(_, _, [assertions]). 

sorry :- throw(not_solved_yet). 

:- test dupli(A, B) : (A = [a,b,c,c,d]) => (B = [a,a,b,b,c,c,c,c,d,d]) + (not_fails, is_det). 

:- test dupli(A, B) : (A = []) => (B = []) + (not_fails, is_det). 

%! \begin{hint} 
% Example: 
% ?- dupli([a,b,c,c,d],X). 
% X = [a,a,b,b,c,c,c,c,d,d] 

dupli(L1,L2) :- sorry. 
  % L2 is obtained from L1 by duplicating all elements. 
  % (list,list) (?,?) 
  
%! \end{hint} 
%! \begin{solution} 
% Example: 
% ?- dupli([a,b,c,c,d],X). 
% X = [a,a,b,b,c,c,c,c,d,d] 

dupli([],[]). 
dupli([X|Xs],[X,X|Ys]) :- dupli(Xs,Ys). 
%! \end{solution} 
?- dupli([a,b,c,c,d],X). 

Duplicate the elements of a list a given number of times

(Intermediate ⭐️⭐️) Duplicate the elements of a list a given number of times.

:- module(_, _, [assertions]). 

:- push_prolog_flag(multi_arity_warnings, off).

sorry :- throw(not_solved_yet). 

:- test dupli(A, B, C) : (A = [a,b,c], B = 3) => (C = [a,a,a,b,b,b,c,c,c]) + (not_fails, is_det). 

:- test dupli(A, _B, C) : (A = []) => (C = []) + (not_fails, is_det). 

%! \begin{hint} 
% Example: 
% ?- dupli([a,b,c],3,X).
% X = [a,a,a,b,b,b,c,c,c]

dupli(L1,N,L2) :- sorry.
  % L2 is obtained from L1 by duplicating all elements
  % N times.
  % (list,integer,list) (?,+,?)

dupli(L1,N,L2,K) :- sorry.
  % L2 is obtained from L1 by duplicating its leading
  % element K times, all other elements N times.
  %(list,integer,list,integer) (?,+,?,+)
%! \end{hint} 
%! \begin{solution} 
% Example: 
% ?- dupli([a,b,c],3,X).
% X = [a,a,a,b,b,b,c,c,c]

dupli(L1,N,L2) :- dupli(L1,N,L2,N).

dupli([],_,[],_).
dupli([_|Xs],N,Ys,0) :- dupli(Xs,N,Ys,N).
dupli([X|Xs],N,[X|Ys],K) :- K > 0, K1 is K - 1, dupli([X|Xs],N,Ys,K1).
%! \end{solution} 
?- dupli(X,3,Y).

Drop every N'th element from a list.

(Intermediate ⭐️⭐️) Drop every N'th element from a list.

:- module(_, _, [assertions]). 

:- push_prolog_flag(multi_arity_warnings, off).

sorry :- throw(not_solved_yet). 

:- test drop(A, B, C) : (A = [a,b,c,d,e,f,g,h,i,k], B = 3) => (C = [a,b,d,e,g,h,k]) + (not_fails, is_det). 

:- test drop(A, _B, C) : (A = []) => (C = []) + (not_fails, is_det). 

%! \begin{hint} 
% Example: 
% ?- drop([a,b,c,d,e,f,g,h,i,k],3,X).
% X = [a,b,d,e,g,h,k]

drop(L1,N,L2) :- sorry.
  % L2 is obtained from L1 by dropping every N'th element.
  % (list,integer,list) (?,+,?)

drop(L1,N,L2,K) :- sorry.
  % L2 is obtained from L1 by first copying K-1 elements
  % and then dropping an element and, from then on, dropping every
  % N'th element.
  % (list,integer,list,integer) (?,+,?,+)

%! \end{hint} 
%! \begin{solution} 
% Example: 
% ?- drop([a,b,c,d,e,f,g,h,i,k],3,X).
% X = [a,b,d,e,g,h,k]

drop(L1,N,L2) :- drop(L1,N,L2,N).

drop([],_,[],_).
drop([_|Xs],N,Ys,1) :- drop(Xs,N,Ys,N).
drop([X|Xs],N,[X|Ys],K) :- K > 1, K1 is K - 1, drop(Xs,N,Ys,K1).
%! \end{solution} 
?- drop([a,b,c,d,e,f,g,h,i,k],3,X).

Split a list

(Easy ⭐️) Split a list into two parts; the length of the first part is given. Do not use any predefined predicates.

:- module(_, _, [assertions]). 

sorry :- throw(not_solved_yet). 

:- test split(A, B, C, D) : (A = [a,b,c,d,e,f,g,h,i,k], B = 3) => (C = [a,b,c], D = [d,e,f,g,h,i,k]) + (not_fails, is_det). 

:- test split(A, B, C, D) : (A = [1,2,3], B = 0) => ( C = [], D = [1,2,3]) + (not_fails, is_det). 

%! \begin{hint} 
% Example: 
% ?- split([a,b,c,d,e,f,g,h,i,k],3,L1,L2).
% L1 = [a,b,c]
% L2 = [d,e,f,g,h,i,k]

split(L,N,L1,L2) :- sorry.
  % the list L1 contains the first N elements
  % of the list L, the list L2 contains the remaining elements.
  % (list,integer,list,list) (?,+,?,?)

%! \end{hint} 
%! \begin{solution} 
% Example: 
% ?- split([a,b,c,d,e,f,g,h,i,k],3,L1,L2).
% L1 = [a,b,c]
% L2 = [d,e,f,g,h,i,k]

split(L,0,[],L).
split([X|Xs],N,[X|Ys],Zs) :- N > 0, N1 is N - 1, split(Xs,N1,Ys,Zs).
%! \end{solution} 
?- split([a,b,c,d,e,f,g,h,i,k],3,L1,L2).

Extract a slice from a list

(Intermediate ⭐️⭐️) Extract a slice from a list. Given two indices, I and K, the slice is the list containing the elements between the I'th and K'th element of the original list (both limits included). Start counting the elements with 1.

:- module(_, _, [assertions]). 

sorry :- throw(not_solved_yet). 

:- test slice(A, B, C, D) : (A = [a,b,c,d,e,f,g,h,i,k], B = 3, C = 7) => 
   (D = [c,d,e,f,g]) + (not_fails, is_det). 

:- test slice(A, B, C, D) : (A = [1,2,3], B = 1, C = 1) => (D = [1]) + (not_fails, is_det). 

%! \begin{hint} 
% Example: 
% ?- slice([a,b,c,d,e,f,g,h,i,k],3,7,L).
% X = [c,d,e,f,g]

slice(L1,I,K,L2) :- sorry.
  % L2 is the list of the elements of L1 between
  % index I and index K (both included).
  % (list,integer,integer,list) (?,+,+,?)

%! \end{hint} 
%! \begin{solution} 
% Example: 
% ?- slice([a,b,c,d,e,f,g,h,i,k],3,7,L).
% X = [c,d,e,f,g]

slice([X|_],1,1,[X]).
slice([X|Xs],1,K,[X|Ys]) :- K > 1, 
   K1 is K - 1, slice(Xs,1,K1,Ys).
slice([_|Xs],I,K,Ys) :- I > 1, 
   I1 is I - 1, K1 is K - 1, slice(Xs,I1,K1,Ys).

%! \end{solution} 
?- slice([a,b,c,d,e,f,g,h,i,k],3,7,L).

Rotate a list

(Intermediate ⭐️⭐️) Rotate a list N places to the left.

:- module(_, _, [assertions]). 

:- use_module(library(classic/classic_predicates),[length/2, append/3]).

sorry :- throw(not_solved_yet). 

:- test rotate(A, B, C) : (A = [a,b,c,d,e,f,g,h], B = 3) 
  => (C = [d,e,f,g,h,a,b,c]) + (not_fails, is_det). 

:- test rotate(A, B, C) : (A = [a,b,c,d,e,f,g,h], B = -2) 
  => (C = [g,h,a,b,c,d,e,f]) + (not_fails, is_det). 

:- test rotate(A, B, C) : (A = [a,b,c,d,e,f,g,h], B = 0) 
  => (C = [a,b,c,d,e,f,g,h]) + (not_fails, is_det). 
  
split(L,0,[],L).
split([X|Xs],N,[X|Ys],Zs) :- N > 0, N1 is N - 1, split(Xs,N1,Ys,Zs).
%! \begin{hint} 
% Example: 
% ?- rotate([a,b,c,d,e,f,g,h],3,X).
% X = [d,e,f,g,h,a,b,c]

rotate(L1,N,L2) :- sorry.
  % the list L2 is obtained from the list L1 by 
  % rotating the elements of L1 N places to the left.
  % (list,integer,list) (+,+,?)
%! \end{hint} 
%! \begin{solution} 
% Example: 
% ?- rotate([a,b,c,d,e,f,g,h],3,X).
% X = [d,e,f,g,h,a,b,c]

rotate(L1,N,L2) :- N >= 0, 
   length(L1,NL1), N1 is N mod NL1, rotate_left(L1,N1,L2).
rotate(L1,N,L2) :- N < 0,
   length(L1,NL1), N1 is NL1 + N, rotate_left(L1,N1,L2).

rotate_left(L,0,L).
rotate_left(L1,N,L2) :- N > 0, split(L1,N,S1,S2), append(S2,S1,L2).
%! \end{solution} 
?- rotate([a,b,c,d,e,f,g,h],-2,X).
Hint: Use the predefined predicates length/2 and append/3, as well as the result of Split a list into two parts problem.

Remove an element from a list

(Easy ⭐️) Remove the K'th element from a list.

:- module(_, _, [assertions]). 

sorry :- throw(not_solved_yet). 

:- test remove_at(A, B, C, D) : (B = [a,b,c,d], C = 2) 
  => (A = b, D = [a,c,d]) + (not_fails, is_det). 

:- test remove_at(A, B, C, D) : (B = [a,b,c,d], C = 1) 
  => (A = a, D = [b,c,d]) + (not_fails, is_det).
%! \begin{hint} 
% Example: 
% ?- remove_at(X,[a,b,c,d],2,R).
% X = b
% R = [a,c,d]

remove_at(X,L,K,R) :- sorry.
  % X is the K'th element of the list L; R is the
  % list that remains when the K'th element is removed from L.
  % (element,list,integer,list) (?,?,+,?)
%! \end{hint} 
%! \begin{solution} 
% Example: 
% ?- remove_at(X,[a,b,c,d],2,R).
% X = b
% R = [a,c,d]

remove_at(X,[X|Xs],1,Xs).
remove_at(X,[Y|Xs],K,[Y|Ys]) :- 
    K > 1, 
    K1 is K - 1,
    remove_at(X,Xs,K1,Ys).
%! \end{solution} 
?- remove_at(X,[a,b,c,d],2,R).

Insert an element into a list

(Easy ⭐️) Insert an element at a given position into a list.

:- module(_, _, [assertions]). 

sorry :- throw(not_solved_yet). 

remove_at(X,[X|Xs],1,Xs).
remove_at(X,[Y|Xs],K,[Y|Ys]) :- 
    K > 1, 
    K1 is K - 1, 
    remove_at(X,Xs,K1,Ys).

:- test insert_at(A, B, C, D) : (A = alfa, B = [a,b,c,d], C = 2) 
  => (D = [a,alfa,b,c,d]) + (not_fails, is_det). 

:- test insert_at(A, B, C, D) : (A = alfa, B = [a,b,c,d], C = 1) 
  => (D = [alfa,a,b,c,d]) + (not_fails, is_det).
  
:- test insert_at(A, B, C, D) : (A = alfa, B = [a,b,c,d], C = 6) + fails.
%! \begin{hint} 
% Example: 
% ?- insert_at(alfa,[a,b,c,d],2,L).
% L = [a,alfa,b,c,d]

insert_at(X,L,K,R) :- sorry.
  % X is inserted into the list L such that it
  % occupies position K. The result is the list R.
  % (element,list,integer,list) (?,?,+,?)
%! \end{hint} 
%! \begin{solution} 
% Example: 
% ?- insert_at(alfa,[a,b,c,d],2,L).
% L = [a,alfa,b,c,d]

insert_at(X,L,K,R) :- remove_at(X,R,K,L).
%! \end{solution} 
?- insert_at(alfa,[a,b,c,d],2,L).
Hint: You can use the result of Remove the K'th element from a list problem.

Creating lists

(Easy ⭐️) Create a list containing all integers within a given range.

:- module(_, _, [assertions]). 

sorry :- throw(not_solved_yet). 

:- test range(A, B, C) : (A = 4, B = 9) 
  => (C = [4,5,6,7,8,9]) + (not_fails, is_det). 

:- test range(A, B, C) : (A = 3, B = 3) 
  => (C = [3]) + (not_fails, is_det). 
  
:- test range(A, B, C) : (A = 3, B = 2) + fails.
%! \begin{hint} 
% Example: 
% ?- range(4,9,L).
% L = [4,5,6,7,8,9]

range(I,K,L) :- sorry.
  % I <= K, and L is the list containing all 
  % consecutive integers from I to K.
  % (integer,integer,list) (+,+,?)

%! \end{hint} 
%! \begin{solution} 
% Example: 
% ?- range(4,9,L).
% L = [4,5,6,7,8,9]

range(I,I,[I]).
range(I,K,[I|L]) :- I < K, I1 is I + 1, range(I1,K,L).
%! \end{solution} 
?- range(4,9,L).

Generate combinations

(Intermediate ⭐️⭐️) Generate the combinations of K distinct objects chosen from the N elements of a list. In how many ways can a committee of 3 be chosen from a group of 12 people? We all know that there are possibilities ( denotes the well-known binomial coefficients). For pure mathematicians, this result may be great. But we want to really generate all the possibilities (via backtracking).

:- module(_, _, [assertions]). 

sorry :- throw(not_solved_yet). 

%! \begin{hint} 
% Example: 
% ?- combination(3,[a,b,c,d,e,f],L).
% L = [a,b,c] ;
% L = [a,b,d] ;
% L = [a,b,e] ;
% ...

combination(K,L,C) :- sorry.
  % C is a list of K distinct elements 
  % chosen from the list L

%! \end{hint} 
%! \begin{solution} 
% Example: 
% ?- combination(3,[a,b,c,d,e,f],L).
% L = [a,b,c] ;
% L = [a,b,d] ;
% L = [a,b,e] ;
% ...

combination(0,_,[]).
combination(K,L,[X|Xs]) :- K > 0,
   el(X,L,R), K1 is K-1, combination(K1,R,Xs).

% Find out what the following predicate el/3 exactly does.

el(X,[X|L],L).
el(X,[_|L],R) :- el(X,L,R).
%! \end{solution} 
?- combination(3,[a,b,c,d,e,f],L).

Grouping elements of a list

(Intermediate ⭐️⭐️) Group the elements of a set into disjoint subsets.

a) In how many ways can a group of 9 people work in 3 disjoint subgroups of 2, 3 and 4 persons? Write a predicate that generates all the possibilities via backtracking.

:- module(_, _, [assertions]). 

:- use_module(library(idlists), [subtract/3]).

sorry :- throw(not_solved_yet). 

%! \begin{hint} 
% Example: 
% ?- group3([aldo,beat,carla,david,evi,flip,gary,hugo,ida],G1,G2,G3).
% G1 = [aldo,beat], G2 = [carla,david,evi], G3 = [flip,gary,hugo,ida]
% ...

group3(G,G1,G2,G3) :- sorry.
  % distribute the 9 elements of G into G1, G2, and G3,
  % such that G1, G2 and G3 contain 2,3 and 4 elements respectively

selectN(N,L,S) :- sorry.
  % select N elements of the list L and put them in 
  % the set S. Via backtracking return all possible selections, but
  % avoid permutations; i.e. after generating S = [a,b,c] do not return
  % S = [b,a,c], etc.

%! \end{hint} 
%! \begin{solution} 
% Example: 
% ?- group3([aldo,beat,carla,david,evi,flip,gary,hugo,ida],G1,G2,G3).
% G1 = [aldo,beat], G2 = [carla,david,evi], G3 = [flip,gary,hugo,ida]
% ...

group3(G,G1,G2,G3) :- 
   selectN(2,G,G1),
   subtract(G,G1,R1),
   selectN(3,R1,G2),
   subtract(R1,G2,R2),
   selectN(4,R2,G3),
   subtract(R2,G3,[]).

selectN(0,_,[]) :- !.
selectN(N,L,[X|S]) :- N > 0, 
   el(X,L,R), 
   N1 is N-1,
   selectN(N1,R,S).

el(X,[X|L],L).
el(X,[_|L],R) :- el(X,L,R).
%! \end{solution} 
?- group3([aldo,beat,carla,david,evi,flip,gary,hugo,ida],G1,G2,G3).
Hint: Use the predefined predicate subtract/3.

b) Generalize the above predicate in a way that we can specify a list of group sizes and the predicate will return a list of groups.

:- module(_, _, [assertions]). 

:- use_module(library(idlists), [subtract/3]).

sorry :- throw(not_solved_yet). 

%! \begin{hint} 
% Example: 
% ?- group([aldo,beat,carla,david,evi,flip,gary,hugo,ida],[2,2,5],Gs).
% Gs = [[aldo,beat],[carla,david],[evi,flip,gary,hugo,ida]]
% ...

group(G,Ns,Gs) :- sorry.
  % distribute the elements of G into the groups Gs.
  % The group sizes are given in the list Ns.

%! \end{hint} 
%! \begin{solution} 
% Example: 
% ?- group([aldo,beat,carla,david,evi,flip,gary,hugo,ida],[2,2,5],Gs).
% Gs = [[aldo,beat],[carla,david],[evi,flip,gary,hugo,ida]]
% ...

group([],[],[]).
group(G,[N1|Ns],[G1|Gs]) :- 
   selectN(N1,G,G1),
   subtract(G,G1,R),
   group(R,Ns,Gs).
%! \end{solution} 
?- group([aldo,beat,carla,david,evi,flip,gary,hugo,ida],[2,2,5],Gs).
Hint: Use the predefined predicate subtract/3.

Note that we do not want permutations of the group members; i.e. [[aldo,beat],...] is the same solution as [[beat,aldo],...]. However, we make a difference between [[aldo,beat],[carla,david],...] and [[carla,david],[aldo,beat],...].

You may find more about this combinatorial problem in a good book on discrete mathematics under the term "multinomial coefficients".

Sort a list of lists

(Intermediate ⭐️⭐️) Sorting a list of lists according to length of sublists

a) We suppose that a list (InList) contains elements that are lists themselves. The objective is to sort the elements of InList according to their length. E.g. short lists first, longer lists later, or vice versa.

:- module(_, _, [assertions]). 

:- push_prolog_flag(multi_arity_warnings, off).

:- use_module(library(classic/classic_predicates), [keysort/2]).
:- use_module(library(classic/classic_predicates), [length/2]).

sorry :- throw(not_solved_yet). 

:- test lsort(A,B) : (A = [[a,b,c],[d,e],[f,g,h],[d,e],[i,j,k,l],[m,n],[o]]) 
    => (B = [[o], [d, e], [d, e], [m, n], [a, b, c], [f, g, h], [i, j, k, l]]) + (not_fails, is_det).
%! \begin{hint} 
% Example: 
% ?- lsort([[a,b,c],[d,e],[f,g,h],[d,e],[i,j,k,l],[m,n],[o]],L).
% L = [[o], [d, e], [d, e], [m, n], [a, b, c], [f, g, h], [i, j, k, l]]

lsort(InList,OutList) :- sorry.
  % it is supposed that the elements of InList 
  % are lists themselves. Then OutList is obtained from InList by sorting 
  % its elements according to their length. lsort/2 sorts ascendingly,
  % lsort/3 allows for ascending or descending sorts.
  % (list_of_lists,list_of_lists), (+,?)

%! \end{hint} 
%! \begin{solution} 
% Example: 
% ?- lsort([[a,b,c],[d,e],[f,g,h],[d,e],[i,j,k,l],[m,n],[o]],L).
% L = [[o], [d, e], [d, e], [m, n], [a, b, c], [f, g, h], [i, j, k, l]]

lsort(InList,OutList) :- lsort(InList,OutList,asc).

% sorting direction Dir is either asc or desc

lsort(InList,OutList,Dir) :-
   add_key(InList,KList,Dir),
   keysort(KList,SKList),
   rem_key(SKList,OutList).

add_key([],[],_).
add_key([X|Xs],[L-p(X)|Ys],asc) :- !, 
        length(X,L), add_key(Xs,Ys,asc).
add_key([X|Xs],[L-p(X)|Ys],desc) :- 
        length(X,L1), L is -L1, add_key(Xs,Ys,desc).

rem_key([],[]).
rem_key([_-p(X)|Xs],[X|Ys]) :- rem_key(Xs,Ys).
%! \end{solution} 
?- lsort([[a,b,c],[d,e],[f,g,h],[d,e],[i,j,k,l],[m,n],[o]],L).
Hint: Use the predefined predicate keysort/2 and length/2.
b) Again, we suppose that a list (InList) contains elements that are lists themselves. But this time the objective is to sort the elements of InList according to their length frequency; i.e. in the default, where sorting is done ascendingly, lists with rare lengths are placed first, others with a more frequent length come later.

:- module(_, _, [assertions]). 

:- push_prolog_flag(multi_arity_warnings, off).

:- use_module(library(classic/classic_predicates), [keysort/2]).
:- use_module(library(classic/classic_predicates), [length/2]).
:- use_module(library(llists), [flatten/2]).

sorry :- throw(not_solved_yet). 

lsort(InList,OutList,Dir) :-
   add_key(InList,KList,Dir),
   keysort(KList,SKList),
   rem_key(SKList,OutList).

add_key([],[],_).
add_key([X|Xs],[L-p(X)|Ys],asc) :- !, 
        length(X,L), add_key(Xs,Ys,asc).
add_key([X|Xs],[L-p(X)|Ys],desc) :- 
        length(X,L1), L is -L1, add_key(Xs,Ys,desc).

rem_key([],[]).
rem_key([_-p(X)|Xs],[X|Ys]) :- rem_key(Xs,Ys).

:- test lfsort(A,B) : (A = [[a,b,c],[d,e],[f,g,h],[d,e],[i,j,k,l],[m,n],[o]]) 
  => (B = [[i, j, k, l], [o], [a, b, c], [f, g, h], [d, e], [d,e], [m, n]]) + (not_fails, is_det).
%! \begin{hint} 
% Example: 
% ?- lfsort([[a,b,c],[d,e],[f,g,h],[d,e],[i,j,k,l],[m,n],[o]],L).
% L = [[i, j, k, l], [o], [a, b, c], [f, g, h], [d, e], [d,e], [m, n]]

lfsort(InList,OutList) :- sorry.
  % it is supposed that the elements of InList
  % are lists themselves. Then OutList is obtained from InList by sorting
  % its elements according to their length frequency; i.e. in the default,
  % where sorting is done ascendingly, lists with rare lengths are placed
  % first, others with more frequent lengths come later.
  
transf(L-X,Xs,Ys,Z) :- sorry.
  % Ys is the list that remains from the list Xs
  % when all leading copies of length L are removed and transfed to Z
%! \end{hint} 
%! \begin{solution} 
% Example: 
% ?- lfsort([[a,b,c],[d,e],[f,g,h],[d,e],[i,j,k,l],[m,n],[o]],L).
% L = [[i, j, k, l], [o], [a, b, c], [f, g, h], [d, e], [d,e], [m, n]]

lfsort(InList,OutList) :- lfsort(InList,OutList,asc).

% sorting direction Dir is either asc or desc

lfsort(InList,OutList,Dir) :-
        add_key(InList,KList,desc),
   keysort(KList,SKList),
   pack(SKList,PKList),
   lsort(PKList,SPKList,Dir),
   flatten(SPKList,FKList),
   rem_key(FKList,OutList).
   
pack([],[]).
pack([L-X|Xs],[[L-X|Z]|Zs]) :- transf(L-X,Xs,Ys,Z), pack(Ys,Zs).

transf(_,[],[],[]).
transf(L-_,[K-Y|Ys],[K-Y|Ys],[]) :- L \= K.
transf(L-_,[L-X|Xs],Ys,[L-X|Zs]) :- transf(L-X,Xs,Ys,Zs).
%! \end{solution} 
?- lfsort([[a,b,c],[d,e],[f,g,h],[d,e],[i,j,k,l],[m,n],[o]],L).
Hint: You can use the predefined predicate flatten/2 and the predicates defined in part a).

Note that in the above example, the first two lists in the result L have length 4 and 1, both lengths appear just once. The third and fourth list have length 3 which appears, there are two list of this length. And finally, the last three lists have length 2. This is the most frequent length.

Arithmetic

Is it prime?

(Intermediate ⭐️⭐️) Determine whether a given integer number is prime.

:- module(_, _, [assertions]). 

sorry :- throw(not_solved_yet). 

:- test is_prime(A) : (A = 7) + (not_fails, is_det).
%! \begin{hint} 
% Example: 
% ?- is_prime(7).
% Yes

is_prime(P) :- sorry.
  % P is a prime number
  % (integer) (+)

has_factor(N,L) :- sorry.
  % N has an odd factor F >= L.
  % (integer, integer) (+,+)
%! \end{hint} 
%! \begin{solution} 
% Example: 
% ?- is_prime(7).
% Yes
    
is_prime(2).
is_prime(3).
is_prime(P) :- integer(P), P > 3, P mod 2 =\= 0, \+ has_factor(P,3).

has_factor(N,L) :- N mod L =:= 0.
has_factor(N,L) :- L * L < N, L2 is L + 2, has_factor(N,L2).

%! \end{solution} 
?- is_prime(7).

Greatest common divisor

(Intermediate ⭐️⭐️) Determine the greatest common divisor of two positive integer numbers. Use Euclid's algorithm.
:- module(_, _, [assertions]). 

sorry :- throw(not_solved_yet). 

:- test gcd(A, B, C) : (A = 36, B = 63) => (C = 9) + (not_fails, is_det).
%! \begin{hint} 
% Example: 
% ?- gcd(36, 63, G).
% G = 9

gcd(X,Y,G) :- sorry.
  % G is the greatest common divisor of X and Y
  % (integer, integer, integer) (+,+,?)
%! \end{hint} 
%! \begin{solution} 
% Example: 
% ?- gcd(36, 63, G).
% G = 9
    
gcd(X,0,X) :- X > 0.
gcd(X,Y,G) :- Y > 0, Z is X mod Y, gcd(Y,Z,G).
%! \end{solution} 
?- gcd(36, 63, G).

Coprimes

(Easy ⭐) Determine whether two positive integer numbers are coprime. Two numbers are coprime if their greatest common divisor equals 1.
:- module(_, _, [assertions]). 

sorry :- throw(not_solved_yet). 

:- test coprime(A, B) : (A = 35, B = 64) + (not_fails, is_det).

gcd(X,0,X) :- X > 0.
gcd(X,Y,G) :- Y > 0, Z is X mod Y, gcd(Y,Z,G).
%! \begin{hint} 
% Example: 
% ?- coprime(35, 64).
% yes

coprime(X,Y) :- sorry.
  % X and Y are coprime.
  % (integer, integer) (+,+)
%! \end{hint} 
%! \begin{solution} 
% Example: 
% ?- coprime(35, 64).
% yes
    
coprime(X,Y) :- gcd(X,Y,1).
%! \end{solution} 
?- coprime(35, 64).
Hint: Use the solution of Determine the greatest common divisor of two positive integers problem.

Euler's totient function

(Intermediate ⭐️⭐️) Calculate Euler's totient function phi(m). Euler's so-called totient function is defined as the number of positive integers that are coprime to . Find out what the value of is if is a prime number. Euler's totient function plays an important role in one of the most widely used public key cryptography methods (RSA). In this exercise you should use the most primitive method to calculate this function (there are smarter ways that we shall discuss later).
:- module(_, _, [assertions]). 

:- push_prolog_flag(multi_arity_warnings, off).

sorry :- throw(not_solved_yet). 

:- test totient_phi(A, B) : (A = 10) => (B = 4) + (not_fails, is_det).

gcd(X,0,X) :- X > 0.
gcd(X,Y,G) :- Y > 0, Z is X mod Y, gcd(Y,Z,G).

coprime(X,Y) :- gcd(X,Y,1).
%! \begin{hint} 
% Example: 
% ?- Phi is totient_phi(10).
% Phi = 4

totient_phi(M,Phi) :- sorry.
  % Phi is the value of the Euler's totient function
  % phi for the argument M.
  % (integer, integer) (+,-)

t_phi(M,Phi,K,C) :- sorry.
  % Phi = C + N, where N is the number of integers R
  % such that K <= R < M and R is coprime to M.
  % (integer,integer,integer,integer) (+,-,+,+)

%! \end{hint} 
%! \begin{solution} 
% Example: 
% ?- Phi is totient_phi(10).
% Phi = 4
    
totient_phi(1,1) :- !.
totient_phi(M,Phi) :- t_phi(M,Phi,1,0).

t_phi(M,Phi,M,Phi) :- !.
t_phi(M,Phi,K,C) :- 
   K < M, coprime(K,M), !, 
   C1 is C + 1, K1 is K + 1,
   t_phi(M,Phi,K1,C1).
t_phi(M,Phi,K,C) :- 
   K < M, K1 is K + 1,
   t_phi(M,Phi,K1,C).
%! \end{solution} 
?- Phi is totient_phi(10)
Hint: Use the solution of Determine whether two positive integer numbers are coprime problem. For example, : ; thus . Note the special case: .

Prime factors

(Intermediate ⭐️⭐️) Determine the prime factors of a given positive integer. Construct a flat list containing the prime factors in ascending order.

:- module(_, _, [assertions]). 

:- push_prolog_flag(multi_arity_warnings, off).

sorry :- throw(not_solved_yet). 

:- test prime_factors(A, B) : (A = 315) => (B = [3,3,5,7]) + (not_fails, is_det).

:- test prime_factors(A, _B) : (A = 0) + (fails).

%! \begin{hint} 
% Example: 
% ?- prime_factors(315, L).
% L = [3,3,5,7]

prime_factors(N, L) :- sorry.
  % N is the list of prime factors of N.
  % (integer,list) (+,?)

prime_factors(N,L,K) :- sorry.
  % L is the list of prime factors of N. It is 
  % known that N does not have any prime factors less than K.
  
next_factor(N,F,NF) :- sorry.
  % when calculating the prime factors of N
  % and if F does not divide N then NF is the next larger candidate to
  % be a factor of N.
%! \end{hint} 
%! \begin{solution} 
% Example: 
% ?- prime_factors(315, L).
% L = [3,3,5,7]
    
prime_factors(N,L) :- N > 0,  prime_factors(N,L,2).

prime_factors(1,[],_) :- !.
prime_factors(N,[F|L],F) :-                           % N is multiple of F
   R is N // F, N =:= R * F, !, prime_factors(R,L,F).
prime_factors(N,L,F) :- 
   next_factor(N,F,NF), prime_factors(N,L,NF).        % N is not multiple of F
   
next_factor(_,2,3) :- !.
next_factor(N,F,NF) :- F * F < N, !, NF is F + 2.
next_factor(N,_,N).                                 % F > sqrt(N)
%! \end{solution} 
?- prime_factors(315, L).

(Intermediate ⭐️⭐️) Determine the prime factors of a given positive integer (2). Construct a list containing the prime factors and their multiplicity.

:- module(_, _, [assertions]). 

:- push_prolog_flag(multi_arity_warnings, off).

sorry :- throw(not_solved_yet). 

:- test prime_factors_mult(A, B) : (A = 315) => (B = [[3,2],[5,1],[7,1]]) + (not_fails, is_det).

next_factor(_,2,3) :- !.
next_factor(N,F,NF) :- F * F < N, !, NF is F + 2.
next_factor(N,_,N).                                 % F > sqrt(N)
%! \begin{hint} 
% Example: 
% ?- prime_factors_mult(315, L).
% L = [[3,2],[5,1],[7,1]]

prime_factors_mult(N, L) :- sorry.
  % L is the list of prime factors of N. It is
  % composed of terms [F,M] where F is a prime factor and M its multiplicity.
  % (integer,list) (+,?)

prime_factors_mult(N,L,K) :- sorry.
  % L is the list of prime factors of N. It is 
  % known that N does not have any prime factors less than K.
  
divide(N,F,M,R) :- sorry.
  % N = R * F**M, M >= 1, and F is not a factor of R. 
  % (integer,integer,integer,integer) (+,+,-,-)
%! \end{hint} 
%! \begin{solution} 
% Example: 
% ?- prime_factors_mult(315, L).
% L = [[3,2],[5,1],[7,1]]

prime_factors_mult(N,L) :- N > 0, prime_factors_mult(N,L,2).

prime_factors_mult(1,[],_) :- !.
prime_factors_mult(N,[[F,M]|L],F) :- divide(N,F,M,R), !, % F divides N
   next_factor(R,F,NF), prime_factors_mult(R,L,NF).
prime_factors_mult(N,L,F) :- !,                          % F does not divide N
   next_factor(N,F,NF), prime_factors_mult(N,L,NF).

divide(N,F,M,R) :- divi(N,F,M,R,0), M > 0.

divi(N,F,M,R,K) :- S is N // F, N =:= S * F, !,          % F divides N
   K1 is K + 1, divi(S,F,M,R,K1).
divi(N,_,M,N,M).
%! \end{solution} 
?- prime_factors_mult(315, L).
Hint: You can use predicate next_factor/3 defined in Determine the prime factors of a given positive integer problem.

Euler's totient function (improved)

(Intermediate ⭐️⭐️) Calculate Euler's totient function (improved). See the previous Euler's totient function problem for the definition of Euler's totient function. If the list of the prime factors of a number is known in the form of:
% ?- prime_factors_mult(315, L).
% L = [[3,2],[5,1],[7,1]]
then the function can be efficiently calculated as follows: Let [[p1,m1],[p2,m2],[p3,m3],...] be the list of prime factors (and their multiplicities) of a given number m. Then phi(m) can be calculated with the following formula: Note that stands for the 'th power of .

:- module(_, _, [assertions]). 

:- push_prolog_flag(multi_arity_warnings, off).

sorry :- throw(not_solved_yet). 

prime_factors_mult(N,L) :- N > 0, prime_factors_mult(N,L,2).

prime_factors_mult(1,[],_) :- !.
prime_factors_mult(N,[[F,M]|L],F) :- divide(N,F,M,R), !, % F divides N
   next_factor(R,F,NF), prime_factors_mult(R,L,NF).
prime_factors_mult(N,L,F) :- !,                          % F does not divide N
   next_factor(N,F,NF), prime_factors_mult(N,L,NF).

divide(N,F,M,R) :- divi(N,F,M,R,0), M > 0.

divi(N,F,M,R,K) :- S is N // F, N =:= S * F, !,          % F divides N
   K1 is K + 1, divi(S,F,M,R,K1).
divi(N,_,M,N,M).

next_factor(_,2,3) :- !.
next_factor(N,F,NF) :- F * F < N, !, NF is F + 2.
next_factor(N,_,N).                                 % F > sqrt(N)

:- test totient_phi_2(A,B) : (A = 315) => (B = 144) + (not_fails, is_det).
%! \begin{hint} 
% Example: 
% ?- totient_phi_2(315, N).
% N = 144

totient_phi_2(N,Phi) :- sorry.
  % Phi is the value of Euler's totient function
  % for the argument N.
  % (integer,integer) (+,?)

%! \end{hint} 
%! \begin{solution} 
% Example: 
% ?- totient_phi_2(315, N).
% N = 144

totient_phi_2(N,Phi) :- prime_factors_mult(N,L), to_phi(L,Phi).

to_phi([],1).
to_phi([[F,1]|L],Phi) :- !,
   to_phi(L,Phi1), Phi is Phi1 * (F - 1).
to_phi([[F,M]|L],Phi) :- M > 1,
   M1 is M - 1, to_phi([[F,M1]|L],Phi1), Phi is Phi1 * F.
%! \end{solution} 
?- totient_phi_2(315, N).
Hint: Use the solution of Determine the prime factors of a given positive integer (2) problem.

Comparing Euler's totient function's methods

(Easy ⭐) Compare the two methods of calculating Euler's totient function. Use the solutions of Euler's totient function's problems to compare the algorithms. Take the number of logical inferences as a measure for efficiency. Try to calculate as an example.

Hint: Load the boxes of Euler's totient function's problems with the solution and make all the queries you want!

List of prime numbers

(Easy ⭐) A list of prime numbers. Given a range of integers by its lower and upper limit, construct a list of all prime numbers in that range.

:- module(_, _, [assertions]). 

sorry :- throw(not_solved_yet). 

is_prime(2).
is_prime(3).
is_prime(P) :- integer(P), P > 3, P mod 2 =\= 0, \+ has_factor(P,3).  

has_factor(N,L) :- N mod L =:= 0.
has_factor(N,L) :- L * L < N, L2 is L + 2, has_factor(N,L2).

:- test prime_list(A,B,C) : (A = 1, B = 50) 
  => (C = [2,3,5,7,11,13,17,19,23,29,31,37,41,43,47]) + (not_fails, is_det).
%! \begin{hint} 
% Example: 
% ?- prime_list(1,10,L).
% [2,3,5,7]

prime_list(A,B,L) :- sorry.
  % L is the list of prime number P with A <= P <= B
%! \end{hint} 
%! \begin{solution} 
% Example: 
% ?- prime_list(1,10,L).
% [2,3,5,7]

prime_list(A,B,L) :- A =< 2, !, p_list(2,B,L).
prime_list(A,B,L) :- A1 is (A // 2) * 2 + 1, p_list(A1,B,L).

p_list(A,B,[]) :- A > B, !.
p_list(A,B,[A|L]) :- is_prime(A), !, 
   next(A,A1), p_list(A1,B,L). 
p_list(A,B,L) :- 
   next(A,A1), p_list(A1,B,L).

next(2,3) :- !.
next(A,A1) :- A1 is A + 2.
%! \end{solution} 
?- prime_list(1,10,L).
Hint: Use the solution of Determine whether a given integer number is prime problem.

Goldbach's conjecture

(Intermediate ⭐️⭐️) Goldbach's conjecture. Goldbach's conjecture says that every positive even number greater than 2 is the sum of two prime numbers. Example: . It is one of the most famous facts in number theory that has not been proved to be correct in the general case. It has been numerically confirmed up to very large numbers (much larger than we can go with our Prolog system). Write a predicate to find the two prime numbers that sum up to a given even integer.

:- module(_, _, [assertions]). 

:- push_prolog_flag(multi_arity_warnings, off).

sorry :- throw(not_solved_yet). 

is_prime(2).
is_prime(3).
is_prime(P) :- integer(P), P > 3, P mod 2 =\= 0, \+ has_factor(P,3).  

has_factor(N,L) :- N mod L =:= 0.
has_factor(N,L) :- L * L < N, L2 is L + 2, has_factor(N,L2).

:- test goldbach(A,B) : (A = 28) => (B = [5,23]) + (not_fails, is_det).

:- test goldbach(A,B) : (A = 4) => (B = [2,2]) + (not_fails, is_det).
%! \begin{hint} 
% Example: 
% ?- goldbach(28, L).
% L = [5,23]

goldbach(N,L) :- sorry.
% L is the list of the two prime numbers that
% sum up to the given N (which must be even).
% (integer,integer) (+,-)
%! \end{hint} 
%! \begin{solution} 
% Example: 
% ?- goldbach(28, L).
% L = [5,23]

goldbach(4,[2,2]) :- !.
goldbach(N,L) :- N mod 2 =:= 0, N > 4, goldbach(N,L,3).

goldbach(N,[P,Q],P) :- Q is N - P, is_prime(Q), !.
goldbach(N,L,P) :- P < N, next_prime(P,P1), goldbach(N,L,P1).

next_prime(P,P1) :- P1 is P + 2, is_prime(P1), !.
next_prime(P,P1) :- P2 is P + 2, next_prime(P2,P1).
%! \end{solution} 
?- goldbach(28, L).
Hint: Use the solution of Determine whether a given integer number is prime problem.

List of Goldbach compositions

(Intermediate ⭐️⭐️) A list of Goldbach compositions. Given a range of integers by its lower and upper limit, print a list of all even numbers and their Goldbach composition. Example, ?- goldbach_list(9,20). ; ; ; ; ;

In most cases, if an even number is written as the sum of two prime numbers, one of them is very small. Very rarely, the primes are both bigger than say 50. Try to find out how many such cases there are in the range 2..3000. Example (for a print limit of 50): ?- goldbach_list(1,2000,50). ; ; ;

:- module(_, _, [assertions]). 

:- use_module(library(streams), [nl/0, display/1]).

:- push_prolog_flag(multi_arity_warnings, off).

sorry :- throw(not_solved_yet). 

is_prime(2).
is_prime(3).
is_prime(P) :- integer(P), P > 3, P mod 2 =\= 0, \+ has_factor(P,3).  

has_factor(N,L) :- N mod L =:= 0.
has_factor(N,L) :- L * L < N, L2 is L + 2, has_factor(N,L2).

goldbach(4,[2,2]) :- !.
goldbach(N,L) :- N mod 2 =:= 0, N > 4, goldbach(N,L,3).

goldbach(N,[P,Q],P) :- Q is N - P, is_prime(Q), !.
goldbach(N,L,P) :- P < N, next_prime(P,P1), goldbach(N,L,P1).

next_prime(P,P1) :- P1 is P + 2, is_prime(P1), !.
next_prime(P,P1) :- P2 is P + 2, next_prime(P2,P1).

%! \begin{hint} 
goldbach_list(A,B) :- sorry.
% print a list of the Goldbach composition
% of all even numbers N in the range A <= N <= B
% (integer,integer) (+,+)

goldbach_list(A,B,L) :- sorry.
  % perform goldbach_list(A,B), but suppress
  % all output when the first prime number is less than the limit L.
%! \end{hint} 
%! \begin{solution} 
goldbach_list(A,B) :- goldbach_list(A,B,2).

goldbach_list(A,B,L) :- A =< 4, !, g_list(4,B,L).
goldbach_list(A,B,L) :- A1 is ((A+1) // 2) * 2, g_list(A1,B,L).

g_list(A,B,_) :- A > B, !.
g_list(A,B,L) :- 
   goldbach(A,[P,Q]),
   print_goldbach(A,P,Q,L),
   A2 is A + 2,
   g_list(A2,B,L).
   
print_goldbach(A,P,Q,L) :- P >= L, !, 
    display(A),
    display(' = '), 
    display(P),
    display(' + '),
    display(Q), nl.
print_goldbach(_,_,_,_).
%! \end{solution} 
Load your code and check if the answer to the query ?- goldbach_list(9, 20) is the same as state above. If not try again!
?- goldbach_list(9, 20).
Hint: Use the solution of Goldbach's conjecture problem.

Logic and Codes

Truth tables

(Intermediate ⭐️⭐️) Truth tables for logical expressions. Define predicates and/2, or/2, nand/2, nor/2, xor/2, impl/2 and equ/2 (for logical equivalence) which succeed or fail according to the result of their respective operations; e.g. and(A,B) will succeed, if and only if both A and B succeed. Note that A and B can be Prolog goals (not only the constants true and fail). A logical expression in two variables can then be written in prefix notation, as in the following example: and(or(A,B),nand(A,B)). Now, write a predicate table/3 which prints the truth table of a given logical expression in two variables.

:- module(_, _, [assertions]). 

:- use_module(library(streams), [nl/0, display/1]).
:- use_module(engine(hiord_rt), [call/1]).

sorry :- throw(not_solved_yet). 

%! \begin{hint} 
% Example: 
% ?- table(A,B,and(A,or(A,B))).
% true true true
% true fail true
% fail true fail
% fail fail fail

table(A,B,Expr) :- sorry.
%! \end{hint} 
%! \begin{solution} 
% Example: 
% ?- table(A,B,and(A,or(A,B))).
% true true true
% true fail true
% fail true fail
% fail fail fail

not(true) :- 
  false.
not(false) :- 
  true.
  
and(A,B) :- A, B.

or(A,_) :- A.
or(_,B) :- B.

equ(A,B) :- or(and(A,B), and(not(A),not(B))).

xor(A,B) :- not(equ(A,B)).

nor(A,B) :- not(or(A,B)).

nand(A,B) :- not(and(A,B)).

impl(A,B) :- or(not(A),B).

% bind(X) :- instantiate X to be true and false successively

bind(true).
bind(fail).

table(A,B,Expr) :- bind(A), bind(B), do(A,B,Expr), fail.

do(A,B,_) :- display(A), display(' '), display(B), display(' '), fail.
do(_,_,Expr) :- Expr, !, display('true'), nl.
do(_,_,_) :- display('fail'), nl.
%! \end{solution} 
Load your code and check if the answer to the query ?- table(A,B,and(A,or(A,B))) is:
true true true
true fail true
fail true fail
fail fail fail
If not, try again!
?- table(A,B,and(A,or(A,B))).
Hint: Use the predefined predicate display/1 and call/1.

Gray code

(Intermediate ⭐️⭐️) Gray code. An n-bit Gray code is a sequence of n-bit strings constructed according to certain rules. For example,

:- module(_, _, [assertions, dynamic]). 

:- use_module(library(classic/classic_predicates),[reverse/2, append/3]).

sorry :- throw(not_solved_yet). 

:- test gray_c(A,B) : (A = 5) =>(B = ['00000','00001','00011','00010','00110','00111','00101','00100','01100','01101','01111','01110','01010','01011','01001','01000','11000','11001','11011','11010','11110','11111','11101','11100','10100','10101','10111','10110','10010','10011','10001','10000']) + (not_fails, is_det).
%! \begin{hint} 
% Find out the construction rules and write a predicate with the
% following specification:
    
gray(N,C) :- sorry.
  % C is the N-bit Gray code
    
% Can you apply the method of "result caching" in order to make the
% predicate more efficient, when it is to be used repeatedly?
%! \end{hint} 
%! \begin{solution} 
gray(1,['0','1']).
gray(N,C) :- N > 1, N1 is N-1,
   gray(N1,C1), reverse(C1,C2),
   prepend('0',C1,C1P),
   prepend('1',C2,C2P),
   append(C1P,C2P,C).

prepend(_,[],[]) :- !.
prepend(X,[C|Cs],[CP|CPs]) :- atom_concat(X,C,CP), prepend(X,Cs,CPs).

% This gives a nice example for the result caching technique:

:- dynamic gray_c/2.

gray_c(1,['0','1']) :- !.
gray_c(N,C) :- N > 1, N1 is N-1, 
   gray_c(N1,C1), reverse(C1,C2),
   prepend('0',C1,C1P),
   prepend('1',C2,C2P),
   append(C1P,C2P,C),
   asserta((gray_c(N,C) :- !)).
%! \end{solution} 
?- gray_c(5,C).
Hint: Use the predefined predicate reverse/2 and append/3.

Huffman code

(Hard ⭐️⭐️⭐️) Huffman code. First of all, consult a good book on discrete mathematics or algorithms for a detailed description of Huffman codes! We suppose a set of symbols with their frequencies, given as a list of fr(S,F) terms. Example: [fr(a,45),fr(b,13),fr(c,12),fr(d,16),fr(e,9),fr(f,5)]. Our objective is to construct a list hc(S,C) terms, where C is the Huffman code word for the symbol S. In our example, the result could be Hs = [hc(a,'0'), hc(b,'101'), hc(c,'100'), hc(d,'111'), hc(e,'1101'), hc(f,'1100')] [hc(a,'01'),...]. The task shall be performed by the predicate huffman/2.

:- module(_, _, [assertions]). 

:- use_module(library(terms), [atom_concat/2]).
:- use_module(library(sort)).

:- push_prolog_flag(multi_arity_warnings, off).

:- test huffman(A,B) : (A =[fr(a,45),fr(b,13),fr(c,12),fr(d,16),fr(e,9),fr(f,5)]) 
  => (B = [hc(a,'0'),hc(b,'101'),hc(c,'100'),hc(d,'111'),hc(e,'1101'),hc(f,'1100')]) + (not_fails, is_det).
  
sorry :- throw(not_solved_yet). 

%! \begin{hint} 
% Example
% ?- huffman([fr(a,45),fr(b,13),fr(c,12),fr(d,16),fr(e,9),fr(f,5)],C).
% C = [hc(a,'0'),hc(b,'101'),hc(c,'100'),hc(d,'111'),hc(e,'1101'),hc(f,'1100')]

huffman(Fs,Hs) :- sorry.
  %  Hs is the Huffman code table for the frequency table Fs
  % (list-of-fr/2-terms, list-of-hc/2-terms)  (+,-).
%! \end{hint} 
%! \begin{solution}
% Example
% ?- huffman([fr(a,45),fr(b,13),fr(c,12),fr(d,16),fr(e,9),fr(f,5)],C).
% C = [hc(a,'0'),hc(b,'101'),hc(c,'100'),hc(d,'111'),hc(e,'1101'),hc(f,'1100')]+

% During the construction process, we need nodes n(F,S) where, at the 
% beginning, F is a frequency and S a symbol. During the process, as n(F,S)
% becomes an internal node, S becomes a term s(L,R) with L and R being 
% again n(F,S) terms. A list of n(F,S) terms, called Ns, is maintained 
% as a sort of priority queue.

huffman(Fs,Cs) :-
   initialize(Fs,Ns),
   make_tree(Ns,T),
   traverse_tree(T,Cs).

initialize(Fs,Ns) :- init(Fs,NsU), sort(NsU,Ns).

init([],[]).
init([fr(S,F)|Fs],[n(F,S)|Ns]) :- init(Fs,Ns).

make_tree([T],T).
make_tree([n(F1,X1),n(F2,X2)|Ns],T) :- 
   F is F1+F2,
   insert(n(F,s(n(F1,X1),n(F2,X2))),Ns,NsR),
   make_tree(NsR,T).

% insert(n(F,X),Ns,NsR) :- insert the node n(F,X) into Ns such that the
%    resulting list NsR is again sorted with respect to the frequency F.

insert(N,[],[N]) :- !.
insert(n(F,X),[n(F0,Y)|Ns],[n(F,X),n(F0,Y)|Ns]) :- F < F0, !.
insert(n(F,X),[n(F0,Y)|Ns],[n(F0,Y)|Ns1]) :- F >= F0, insert(n(F,X),Ns,Ns1).

% traverse_tree(T,Cs) :- traverse the tree T and construct the Huffman 
%    code table Cs,

traverse_tree(T,Cs) :- traverse_tree(T,'',Cs1-[]), sort(Cs1,Cs).

traverse_tree(n(_,A),Code,[hc(A,Code)|Cs]-Cs) :- atom(A). % leaf node
traverse_tree(n(_,s(Left,Right)),Code,Cs1-Cs3) :-         % internal node
   atom_concat(Code,'0',CodeLeft), 
   atom_concat(Code,'1',CodeRight),
   traverse_tree(Left,CodeLeft,Cs1-Cs2),
   traverse_tree(Right,CodeRight,Cs2-Cs3).
%! \end{solution} 
?- huffman([fr(a,45),fr(b,13),fr(c,12),fr(d,16),fr(e,9),fr(f,5)],C).

Binary Trees

A binary tree is either empty or it is composed of a root element and two successors, which are binary trees themselves.

In Prolog we represent the empty tree by the atom 'nil' and the non-empty tree by the term t(X,L,R), where X denotes the root node and L and R denote the left and right subtree, respectively.

The example tree depicted opposite is therefore represented by the following Prolog term:

T1 = t(a,t(b,t(d,nil,nil),t(e,nil,nil)),t(c,nil,t(f,t(g,nil,nil),nil))).

Other examples are a binary tree that consists of a root node only: T2 = t(a,nil,nil) or an empty binary tree: T3 = nil. You can check your predicates using these example trees. They are given as test cases in Is it a binary tree? problem.

Is it a binary tree?

(Easy ⭐) Check whether a given term represents a binary tree. Write a predicate istree/1 which succeeds if and only if its argument is a Prolog term representing a binary tree.

:- module(_, _, [assertions]). 

:- test istree(A) : (A = (t(a,t(b,t(d,nil,nil),t(e,nil,nil)),t(c,nil,t(f,t(g,nil,nil),nil))))) + (not_fails, is_det).

:- test istree(A) : (A = nil) + (not_fails, is_det).

:- test istree(A) : (A = (t(a,nil,nil))) + (not_fails, is_det).

sorry :- throw(not_solved_yet). 

%! \begin{hint} 

% Example
% ?- istree(t(a,t(b,nil,nil))).
% No

istree(T) :- sorry.
  % T is a term representing a binary tree (i), (o)

%! \end{hint} 
%! \begin{solution}
% Example
% ?- istree(t(a,t(b,nil,nil))).
% No

istree(nil).
istree(t(_,L,R)) :- istree(L), istree(R).
%! \end{solution}
?- istree(t(a,t(b,nil,nil),nil)).

Balanced binary trees

(Intermediate ⭐️⭐️) Construct completely balanced binary trees. In a completely balanced binary tree, the following property holds for every node: The number of nodes in its left subtree and the number of nodes in its right subtree are almost equal, which means their difference is not greater than one. Write a predicate cbal_tree/2 to construct completely balanced binary trees for a given number of nodes. The predicate should generate all solutions via backtracking. Put the letter 'x' as information into all nodes of the tree.

:- module(_, _, [assertions]). 

sorry :- throw(not_solved_yet). 

:- test cbal_tree(A,T) : (A = 1) => (T = t(x,nil,nil)) + (not_fails, is_det) .
%! \begin{hint} 
% Example
% ?- cbal_tree(4,T).
% T = t(x, t(x, nil, nil), t(x, nil, t(x, nil, nil))) ;
% T = t(x, t(x, nil, nil), t(x, t(x, nil, nil), nil)) ;
% etc.

cbal_tree(N,T) :- sorry.
  % T is a completely balanced binary tree with N nodes.
  % (integer, tree)  (+,?)
%! \end{hint} 
%! \begin{solution}
% Example
% ?- cbal_tree(4,T).
% T = t(x, t(x, nil, nil), t(x, nil, t(x, nil, nil))) ;
% T = t(x, t(x, nil, nil), t(x, t(x, nil, nil), nil)) ;
% etc.

cbal_tree(0,nil) :- !.
cbal_tree(N,t(x,L,R)) :- N > 0,
        N0 is N - 1, 
        N1 is N0//2, N2 is N0 - N1,
        distrib(N1,N2,NL,NR),
        cbal_tree(NL,L), cbal_tree(NR,R).

distrib(N,N,N,N) :- !.
distrib(N1,N2,N1,N2).
distrib(N1,N2,N2,N1).
%! \end{solution}
?- cbal_tree(4,T).

Symmetric binary trees

(Intermediate ⭐️⭐️) Symmetric binary trees. Let us call a binary tree symmetric if you can draw a vertical line through the root node and then the right subtree is the mirror image of the left subtree. Write a predicate symmetric/1 to check whether a given binary tree is symmetric.

:- module(_, _, [assertions]). 

sorry :- throw(not_solved_yet). 

:- test symmetric(A) : (A = nil) + (not_fails, is_det) .

:- test symmetric(A) : (A = t(5,t(3,t(1,nil,nil),t(4,nil,nil)),t(18,t(12,nil,nil),t(21,nil,nil)))) + (not_fails, is_det).
%! \begin{hint} 
% Example
% ?- symmetric(t(5,t(3,t(1,nil,nil),t(4,nil,nil)),t(18,t(12,nil,nil),t(21,nil,nil))))
% yes

symmetric(T) :- sorry.
  % the binary tree T is symmetric.

%! \end{hint} 
%! \begin{solution}
% Example
% ?- symmetric(t(5,t(3,t(1,nil,nil),t(4,nil,nil)),t(18,t(12,nil,nil),t(21,nil,nil))))
% yes

symmetric(nil).
symmetric(t(_,L,R)) :- mirror(L,R).

mirror(nil,nil).
mirror(t(_,L1,R1),t(_,L2,R2)) :- mirror(L1,R2), mirror(R1,L2).
%! \end{solution}
?- symmetric(t(5,t(3,t(1,nil,nil),t(4,nil,nil)),t(18,t(12,nil,nil),t(21,nil,nil))))
Hint: Write a predicate mirror/2 first to check whether one tree is the mirror image of another. We are only interested in the structure, not in the contents of the nodes.

Generate-and-test paradigm

(Intermediate ⭐️⭐️) Generate-and-test paradigm. Apply the generate-and-test paradigm to construct all symmetric, completely balanced binary trees with a given number of nodes.

:- module(_, _, [assertions]). 

:- use_module(library(streams), [nl/0, display/1]).
:- use_module(library(aggregates), [setof/3]).
:- use_module(library(classic/classic_predicates), [between/3, length/2]).

sorry :- throw(not_solved_yet). 

:- test sym_cbal_trees(A,Ts) : (A = 5) => (Ts = [t(x, t(x, nil, t(x, nil, nil)), t(x, t(x, nil, nil), nil)),t(x, t(x, t(x, nil, nil), nil), t(x, nil, t(x, nil, nil)))]) + (not_fails, is_det).

symmetric(nil).
symmetric(t(_,L,R)) :- mirror(L,R).

mirror(nil,nil).
mirror(t(_,L1,R1),t(_,L2,R2)) :- mirror(L1,R2), mirror(R1,L2).

cbal_tree(0,nil) :- !.
cbal_tree(N,t(x,L,R)) :- N > 0,
        N0 is N - 1, 
        N1 is N0//2, N2 is N0 - N1,
        distrib(N1,N2,NL,NR),
        cbal_tree(NL,L), cbal_tree(NR,R).

distrib(N,N,N,N) :- !.
distrib(N1,N2,N1,N2).
distrib(N1,N2,N2,N1).
%! \begin{hint} 
% Example
% ?- sym_cbal_trees(5,Ts).
% Ts = [t(x, t(x, nil, t(x, nil, nil)), t(x, t(x, nil, nil), nil)),
% t(x, t(x, t(x, nil, nil), nil), t(x, nil, t(x, nil, nil)))]

sym_cbal_tree(N,T) :- sorry.

%! \end{hint} 
%! \begin{solution}
% Example
% ?- sym_cbal_trees(5,Ts).
% Ts = [t(x, t(x, nil, t(x, nil, nil)), t(x, t(x, nil, nil), nil)),
% t(x, t(x, t(x, nil, nil), nil), t(x, nil, t(x, nil, nil)))]

sym_cbal_tree(N,T) :- cbal_tree(N,T), symmetric(T).

sym_cbal_trees(N,Ts) :- setof(T,sym_cbal_tree(N,T),Ts).

investigate(A,B) :-
        between(A,B,N),
        sym_cbal_trees(N,Ts), length(Ts,L),
    display(N), display('  '), display(L), nl,
    fail.
investigate(_,_).
%! \end{solution}
?- sym_cbal_trees(5,Ts).
How many such trees are there with 57 nodes? How many solutions there are for a given number of nodes? What if the number is even? Write an appropriate predicate.

Height-balanced binary trees

(Intermediate ⭐️⭐️) Construct height-balanced binary trees. In a height-balanced binary tree, the following property holds for every node: The height of its left subtree and the height of its right subtree are almost equal, which means their difference is not greater than one. Write a predicate hbal_tree/2 to construct height-balanced binary trees for a given height. The predicate should generate all solutions via backtracking. Put the letter 'x' as information into all nodes of the tree.

:- module(_, _, [assertions]). 

sorry :- throw(not_solved_yet). 

:- test hbal_tree(A,T) : (A = 0) => (T = nil) + (not_fails, is_det).

:- test hbal_tree(A,T) : (A = 1) => (T = t(x,nil,nil)) + (not_fails, is_det).

%! \begin{hint} 
% Example
% ?- hbal_tree(3,T).
% T = t(x, t(x, t(x, nil, nil), t(x, nil, nil)), t(x, t(x, nil, nil),t(x, nil, nil))) ;
% T = t(x, t(x, t(x, nil, nil), t(x, nil, nil)), t(x, t(x, nil, nil),nil)) ;
% etc.

hbal_tree(D,T) :- sorry.
  % T is a height-balanced binary tree with depth T

%! \end{hint} 
%! \begin{solution}
% Example
% ?- hbal_tree(3,T).
% T = t(x, t(x, t(x, nil, nil), t(x, nil, nil)), t(x, t(x, nil, nil),t(x, nil, nil))) ;
% T = t(x, t(x, t(x, nil, nil), t(x, nil, nil)), t(x, t(x, nil, nil),nil)) ;
% etc.

hbal_tree(0,nil) :- !.
hbal_tree(1,t(x,nil,nil)) :- !.
hbal_tree(D,t(x,L,R)) :- D > 1,
        D1 is D - 1, D2 is D - 2,
        distr(D1,D2,DL,DR),
        hbal_tree(DL,L), hbal_tree(DR,R).

distr(D1,_,D1,D1).
distr(D1,D2,D1,D2).
distr(D1,D2,D2,D1).
%! \end{solution}
?- hbal_tree(3,T).

(Intermediate ⭐️⭐️) Construct height-balanced binary trees with a given number of nodes. Consider a height-balanced binary tree of height H. What is the maximum number of nodes it can contain? Clearly, MaxN = 2**H - 1. However, what is the minimum number MinN? This question is more difficult. Try to find a recursive statement and turn it into a predicate minNodes/2. On the other hand, we might ask: what is the maximum height H a height-balanced binary tree with N nodes can have?

:- module(_, _, [assertions]). 

:- use_module(library(aggregates), [setof/3]).
:- use_module(library(classic/classic_predicates), [between/3, length/2]).

:- push_prolog_flag(multi_arity_warnings, off).

sorry :- throw(not_solved_yet). 

:- test hbal_tree_nodes(A,T) : (A = 0) => (T = nil) + (not_fails, is_det).

:- test hbal_tree_nodes(A,T) : (A = 1) => (T = t(x,nil,nil)) + (not_fails, is_det).

:- test hbal_tree_nodes(A,T) : (A = 3) => (T = t(x,t(x,nil,nil),t(x,nil,nil))) + (not_fails, is_det).

hbal_tree(0,nil) :- !.
hbal_tree(1,t(x,nil,nil)) :- !.
hbal_tree(D,t(x,L,R)) :- D > 1,
        D1 is D - 1, D2 is D - 2,
        distr(D1,D2,DL,DR),
        hbal_tree(DL,L), hbal_tree(DR,R).

distr(D1,_,D1,D1).
distr(D1,D2,D1,D2).
distr(D1,D2,D2,D1).
%! \begin{hint} 
% Example
% ?- hbal_tree_nodes(3,T).
% T = t(x,t(x,nil,nil),t(x,nil,nil))

minNodes(H,N) :- sorry.
  % N is the minimum number of nodes in a height-balanced binary tree of height H
  % (integer,integer) (+,?)

maxNodes(H,N) :- sorry.
  % N is the maximum number of nodes in a height-balanced binary tree of height H
  % (integer,integer) (+,?)

minHeight(N,H) :- sorry.
  % H is the minimum height of a height-balanced binary tree with N nodes
  % (integer,integer) (+,?)

maxHeight(N,H) :- sorry.
  % H is the maximum height of a height-balanced binary tree with N nodes
  % (integer,integer), (+,?)

% Now, we can attack the main problem: construct all the
% height-balanced binary trees with a given number of nodes.

hbal_tree_nodes(N,T) :- sorry.
  % T is a height-balanced binary tree with N nodes.
  % T is a height-balanced binary tree with N nodes.
%! \end{hint} 
%! \begin{solution}
% Example
% ?- hbal_tree_nodes(3,T).
% T = t(x,t(x,nil,nil),t(x,nil,nil))

minNodes(0,0) :- !.
minNodes(1,1) :- !.
minNodes(H,N) :- H > 1, 
        H1 is H - 1, H2 is H - 2,
        minNodes(H1,N1), minNodes(H2,N2),
        N is 1 + N1 + N2.

maxNodes(H,N) :- N is 2**H - 1.

minHeight(0,0) :- !.
minHeight(N,H) :- N > 0, N1 is N//2, minHeight(N1,H1), H is H1 + 1.

maxHeight(N,H) :- maxHeight(N,H,1,1).

maxHeight(N,H,H1,N1) :- N1 > N, !, H is H1 - 1.
maxHeight(N,H,H1,N1) :- N1 =< N, 
        H2 is H1 + 1, minNodes(H2,N2), maxHeight(N,H,H2,N2).

hbal_tree_nodes(N,T) :- 
        minHeight(N,Hmin), maxHeight(N,Hmax),
        between(Hmin,Hmax,H),
        hbal_tree(H,T), nodes(T,N).
        
%  nodes(T,N) :- the binary tree T has N nodes
% (tree,integer);  (i,*) 
 
nodes(nil,0).
nodes(t(_,Left,Right),N) :-
   nodes(Left,NLeft),
   nodes(Right,NRight),
   N is NLeft + NRight + 1.
   
count_hbal_trees(N,C) :- setof(T,hbal_tree_nodes(N,T),Ts), length(Ts,C). 
%! \end{solution}
Find out how many height-balanced trees exist for N = 15.
?- count_hbal_trees(15,C).

Leaves of a binary tree

(Easy ⭐) Count the leaves of a binary tree. A leaf is a node with no successors. Write a predicate count_leaves/2 to count them.

:- module(_, _, [assertions]). 

:- test count_leaves(A,B) : (A = nil) => (B = 0) + (not_fails, is_det).

:- test count_leaves(A,B) : (A = t(_,nil,nil)) => (B = 1) + (not_fails, is_det).

:- test count_leaves(A,B) : (A = t(x, t(x, t(x, nil, nil), t(x, nil, nil)), t(x, t(x, nil, nil),t(x, nil, nil)))) => (B = 4) + (not_fails, is_det).
sorry :- throw(not_solved_yet). 

%! \begin{hint} 
% Example
% ?- count_leaves(t(x, t(x, t(x, nil, nil), t(x, nil, nil)), t(x, t(x, nil, nil),t(x, nil, nil))),B).
% B = 4

count_leaves(T,N) :- sorry.
  % the binary tree T has N leaves

%! \end{hint} 
%! \begin{solution}
% Example
% ?- count_leaves(t(x, t(x, t(x, nil, nil), t(x, nil, nil)), t(x, t(x, nil, nil),t(x, nil, nil))),B).
% B = 4

count_leaves(nil,0).
count_leaves(t(_,nil,nil),1).
count_leaves(t(_,L,nil),N) :- L = t(_,_,_), count_leaves(L,N).
count_leaves(t(_,nil,R),N) :- R = t(_,_,_), count_leaves(R,N).
count_leaves(t(_,L,R),N) :- L = t(_,_,_), R = t(_,_,_),
   count_leaves(L,NL), count_leaves(R,NR), N is NL + NR.

% The above solution works in the flow patterns (i,o) and (i,i)
% without cut and produces a single correct result. Using a cut 
% we can obtain the same result in a much shorter program, like this:

count_leaves1(nil,0).
count_leaves1(t(_,nil,nil),1) :- !.
count_leaves1(t(_,L,R),N) :- 
    count_leaves1(L,NL), count_leaves1(R,NR), N is NL+NR.
        
% For the flow pattern (o,i) see P61A
%! \end{solution}
?- count_leaves(t(x, t(x, t(x, nil, nil), t(x, nil, nil)), t(x, t(x, nil, nil),t(x, nil, nil))),B).
(Easy ⭐) Collect the leaves of a binary tree in a list. A leaf is a node with no successors. Write a predicate leaves/2 to collect them in a list.

:- module(_, _, [assertions]). 

:- use_module(library(classic/classic_predicates), [between/3, append/3]).

:- push_prolog_flag(multi_arity_warnings, off).

:- test leaves(A,B) : (A = nil) => (B = []) + (not_fails, is_det).

:- test leaves(A,B) : (A = t(x,nil,nil)) => (B = [x]) + (not_fails, is_det).

:- test leaves(A,B) : (A = t(x, t(x, t(x, nil, nil), t(x, nil, nil)), t(x, t(x, nil, nil),t(x, nil, nil)))) => (B = [x,x,x,x]) + (not_fails, is_det).

sorry :- throw(not_solved_yet). 

%! \begin{hint} 
% Example
% ?- count_leaves(t(x, t(x, t(x, nil, nil), t(x, nil, nil)), t(x, t(x, nil, nil),t(x, nil, nil))),B).
% B = [x,x,x,x]

leaves(T,S) :- sorry.
  % S is the list of the leaves of the binary tree T

nnodes(T,N) :- sorry.
  % T is a binary tree with N nodes (o,i)

leaves2(T,S) :- sorry.
  % S is the list of leaves of the tree T (o,i)

%! \end{hint} 
%! \begin{solution}
% Example
% ?- count_leaves(t(x, t(x, t(x, nil, nil), t(x, nil, nil)), t(x, t(x, nil, nil),t(x, nil, nil))),B).
% B = [x,x,x,x]

leaves(nil,[]).
leaves(t(X,nil,nil),[X]).
leaves(t(_,L,nil),S) :- L = t(_,_,_), leaves(L,S).
leaves(t(_,nil,R),S) :- R = t(_,_,_), leaves(R,S).
leaves(t(_,L,R),S) :- L = t(_,_,_), R = t(_,_,_),
    leaves(L,SL), leaves(R,SR), append(SL,SR,S).

% The above solution works in the flow patterns (i,o) and (i,i)
% without cut and produces a single correct result. Using a cut 
% we can obtain the same result in a much shorter program, like this:

leaves1(nil,[]).
leaves1(t(X,nil,nil),[X]) :- !.
leaves1(t(_,L,R),S) :- 
    leaves1(L,SL), leaves1(R,SR), append(SL,SR,S).

% To write a predicate that works in the flow pattern (o,i)
% is a more difficult problem, because using append/3 in
% the flow pattern (o,o,i) always generates an empty list 
% as first solution and the result is an infinite recursion
% along the left subtree of the generated binary tree.
% A possible solution is the following trick: we successively
% construct binary tree structures for a given number of nodes
% and fill the leaf nodes with the elements of the leaf list.
% We then increment the number of tree nodes successively,
% and so on. 

nnodes(nil,0) :- !.
nnodes(t(_,L,R),N) :- N > 0, N1 is N-1, 
   between(0,N1,NL), NR is N1-NL,
   nnodes(L,NL), nnodes(R,NR).

leaves2(T,S) :- leaves2(T,S,0).

leaves2(T,S,N) :- nnodes(T,N), leaves1(T,S).
leaves2(T,S,N) :- N1 is N+1, leaves2(T,S,N1).
%! \end{solution}
?- leaves(t(x, t(x, t(x, nil, nil), t(x, nil, nil)), t(x, t(x, nil, nil),t(x, nil, nil))),B).

Nodes of a binary tree

(Easy ⭐) Collect the internal nodes of a binary tree in a list. An internal node of a binary tree has either one or two non-empty successors. Write a predicate internals/2 to collect them in a list.

:- module(_, _, [assertions]). 

:- use_module(library(classic/classic_predicates), [append/3]).

:- test internals(A,B) : (A = nil) => (B = []) + (not_fails, is_det).

:- test internals(A,B) : (A = t(_,nil,nil)) => (B = []) + (not_fails, is_det).

:- test internals(A,B) : (A = t(x, t(x, t(x, nil, nil), t(x, nil, nil)), t(x, t(x, nil, nil),t(x, nil, nil)))) => (B = [x,x,x]) + (not_fails, is_det).

sorry :- throw(not_solved_yet). 

%! \begin{hint} 
% Example
% ?- internals(t(x, t(x, t(x, nil, nil), t(x, nil, nil)), t(x, t(x, nil, nil),t(x, nil, nil))),B).
% B = [x,x,x]

internals(T,S) :- sorry.
  % S is the list of internal nodes of the binary tree T.

%! \end{hint} 
%! \begin{solution}
% Example
% ?- internals(t(x, t(x, t(x, nil, nil), t(x, nil, nil)), t(x, t(x, nil, nil),t(x, nil, nil))),B).
% B = [x,x,x]

internals(nil,[]).
internals(t(_,nil,nil),[]).
internals(t(X,L,nil),[X|S]) :- L = t(_,_,_), internals(L,S).
internals(t(X,nil,R),[X|S]) :- R = t(_,_,_), internals(R,S).
internals(t(X,L,R),[X|S]) :- L = t(_,_,_), R = t(_,_,_), 
   internals(L,SL), internals(R,SR), append(SL,SR,S).

% The above solution works in the flow patterns (i,o) and (i,i)
% without cut and produces a single correct result. Using a cut 
% we can obtain the same result in a much shorter program, like this:

internals1(nil,[]).
internals1(t(_,nil,nil),[]) :- !.
internals1(t(X,L,R),[X|S]) :- 
    internals1(L,SL), internals1(R,SR), append(SL,SR,S).

% For the flow pattern (o,i) there is the following very
% elegant solution:

internals2(nil,[]).
internals2(t(X,L,R),[X|S]) :- 
   append(SL,SR,S), internals2(L,SL), internals2(R,SR).

%! \end{solution}
?- internals(t(x, t(x, t(x, nil, nil), t(x, nil, nil)), t(x, t(x, nil, nil),t(x, nil, nil))),B).
Collect the nodes at a given level in a list. A node of a binary tree is at level N if the path from the root to the node has length N-1. The root node is at level 1. Write a predicate atlevel/3 to collect all nodes at a given level in a list.

:- module(_, _, [assertions]). 

:- use_module(library(classic/classic_predicates), [between/3, append/3]).

:- push_prolog_flag(multi_arity_warnings, off).

:- test atlevel(A,_B,C) : (A = nil) => (C = []) + (not_fails, is_det).

:- test atlevel(A,B,C) : (A = t(X,_,_), B = 1) => (C = [X]) + (not_fails, is_det).

:- test atlevel(A,B,C) : (A = t(x, t(x, t(x, nil, nil), t(x, nil, nil)), t(x, t(x, nil, nil),t(x, nil, nil))), B = 2) => (C = [x,x]) + (not_fails, is_det).

sorry :- throw(not_solved_yet). 

%! \begin{hint} 
% Example
% ?- atlevel(t(x, t(x, t(x, nil, nil), t(x, nil, nil)), t(x, t(x, nil, nil),t(x, nil, nil))),2,C).
% B = [x,x]

atlevel(T,L,S) :- sorry.
  % S is the list of nodes of the binary tree T at level L

%! \end{hint} 
%! \begin{solution}
% Example
% ?- atlevel(t(x, t(x, t(x, nil, nil), t(x, nil, nil)), t(x, t(x, nil, nil),t(x, nil, nil))),2,C).
% B = [x,x]

atlevel(nil,_,[]).
atlevel(t(X,_,_),1,[X]).
atlevel(t(_,L,R),D,S) :- D > 1, D1 is D-1,
   atlevel(L,D1,SL), atlevel(R,D1,SR), append(SL,SR,S).

% The following is a quick-and-dirty solution for the
% level-order sequence

levelorder(T,S) :- levelorder(T,S,1).

levelorder(T,[],D) :- atlevel(T,D,[]), !.
levelorder(T,S,D) :- atlevel(T,D,SD),
   D1 is D+1, levelorder(T,S1,D1), append(SD,S1,S).
%! \end{solution}
?- atlevel(t(x, t(x, t(x, nil, nil), t(x, nil, nil)), t(x, t(x, nil, nil),t(x, nil, nil))),2,T).
Hint: Using atlevel/3 it is easy to construct a predicate levelorder/2 which creates the level-order sequence of the nodes. However, there are more efficient ways to do that.

Complete binary tree

(Intermediate ⭐️⭐️) Construct a complete binary tree A complete binary tree with height H is defined as follows: The levels 1,2,3,...,H-1 contain the maximum number of nodes (i.e 2**(i-1) at the level i, note that we start counting the levels from 1 at the root). In level H, which may contain less than the maximum possible number of nodes, all the nodes are "left-adjusted". This means that in a levelorder tree traversal all internal nodes come first, the leaves come second, and empty successors (the nil's which are not really nodes!) come last. Particularly, complete binary trees are used as data structures (or addressing schemes) for heaps.

We can assign an address number to each node in a complete binary tree by enumerating the nodes in level order, starting at the root with number 1. In doing so, we realize that for every node X with address A the following property holds: The address of X's left and right successors are 2*A and 2*A+1, respectively, supposed the successors do exist. This fact can be used to elegantly construct a complete binary tree structure. Write a predicate complete_binary_tree/2.

:- module(_, _, [assertions]). 

:- use_module(library(classic/classic_predicates), [length/2]).

:- push_prolog_flag(multi_arity_warnings, off).

:- test complete_binary_tree(A,B) : (A = 0) => (B = nil) + (not_fails, is_det).

:- test complete_binary_tree(A,B) : (A = 1) => (B = t(_,nil,nil)) + (not_fails, is_det).

:- test complete_binary_tree(A,B) : (A = 2) => (B = t(_,t(_,nil,nil),nil)) + (not_fails, is_det).

sorry :- throw(not_solved_yet). 

%! \begin{hint} 

complete_binary_tree(N,T) :- sorry.
  % T is a complete binary tree with N nodes. (+,?)

%! \end{hint} 
%! \begin{solution}

complete_binary_tree(N,T) :-
  complete_binary_tree(N,T,1).

complete_binary_tree(N,nil,A) :- A > N, !.
complete_binary_tree(N,t(_,L,R),A) :- A =< N,
        AL is 2 * A, AR is AL + 1,
        complete_binary_tree(N,L,AL),
        complete_binary_tree(N,R,AR).
        
% ----------------------------------------------------------------------

% This was the solution to the exercise. What follows is an application
% of this result.

% We define a heap as a term heap(N,T) where N is the number of elements
% and T a complete binary tree (in the sense used above).

% The conservative usage of a heap is first to declare it with a predicate
% declare_heap/2 and then use it with a predicate element_at/3.

% declare_heap(H,N) :- 
%    declare H to be a heap with a fixed number N  of elements

declare_heap(heap(N,T),N) :- complete_binary_tree(N,T).

% element_at(H,K,X) :- X is the element at address K in the heap H. 
%  The first element has address 1.
%  (+,+,?)

element_at(heap(_,T),K,X) :- 
   binary_path(K,[],BP), element_at_path(T,BP,X).

binary_path(1,Bs,Bs) :- !.
binary_path(K,Acc,Bs) :- K > 1, 
   B is K /\ 1, K1 is K >> 1, binary_path(K1,[B|Acc],Bs).

element_at_path(t(X,_,_),[],X) :- !.
element_at_path(t(_,L,_),[0|Bs],X) :- !, element_at_path(L,Bs,X).
element_at_path(t(_,_,R),[1|Bs],X) :- element_at_path(R,Bs,X).


% We can transform lists into heaps and vice versa with the following
% useful predicate:

% list_heap(L,H) :- transform a list into a (limited) heap and vice versa.

list_heap(L,H) :- list(L), list_to_heap(L,H).
list_heap(L,heap(N,T)) :- integer(N), fill_list(heap(N,T),N,1,L).

list_to_heap(L,H) :- 
   length(L,N), declare_heap(H,N), fill_heap(H,L,1).

fill_heap(_,[],_).
fill_heap(H,[X|Xs],K) :- element_at(H,K,X), K1 is K+1, fill_heap(H,Xs,K1).

fill_list(_,N,K,[]) :- K > N.
fill_list(H,N,K,[X|Xs]) :- K =< N, 
   element_at(H,K,X), K1 is K+1, fill_list(H,N,K1,Xs).


% However, a more aggressive usage is *not* to define the heap in the
% beginning, but to use it as a partially instantiated data structure.
% Used in this way, the number of elements in the heap is unlimited.
% This is Power-Prolog!

% Try the following and find out exactly what happens.

% ?- element_at(H,5,alfa), element_at(H,2,beta), element(H,5,A).

%! \end{solution}
Test your predicate in an appropriate way (e.g. pressing the northeast pointing arrow () will load the code in a separate Prolog playground window).

Layout a binary tree

(Intermediate ⭐️⭐️) Layout a binary tree (1). Given a binary tree as the usual Prolog term t(X,L,R) (or nil). As a preparation for drawing the tree, a layout algorithm is required to determine the position of each node in a rectangular grid. Several layout methods are conceivable, one of them is shown in the illustration below.

In this layout strategy, the position of a node v is obtained by the following two rules:

  • x(v) is equal to the position of the node v in the inorder sequence
  • y(v) is equal to the depth of the node v in the tree
:- module(_, _, [assertions]). 

:- push_prolog_flag(multi_arity_warnings, off).

sorry :- throw(not_solved_yet). 

:- test layout_binary_tree(T,PT) : (T = nil) => (PT = nil) + (not_fails, is_det).

:- test layout_binary_tree(T,PT) : (T = t(n,t(k,t(c,t(a,nil,nil),t(h,t(g,t(e,nil,nil),nil),nil)),t(m,nil,nil)),t(u,t(p,nil,t(s,t(q,nil,nil),nil)),nil))) => (PT = t(n,8,1,t(k,6,2,t(c,2,3,t(a,1,4,nil,nil),t(h,5,4,t(g,4,5,t(e,3,6,nil,nil),nil),nil)),t(m,7,3,nil,nil)),t(u,12,2,t(p,9,3,nil,t(s,11,4,t(q,10,5,nil,nil),nil)),nil))) + (not_fails, is_det). 

add(X,nil,t(X,nil,nil)).
add(X,t(Root,L,R),t(Root,L1,R)) :- X @< Root, add(X,L,L1).
add(X,t(Root,L,R),t(Root,L,R1)) :- X @> Root, add(X,R,R1).

construct(L,T) :- construct(L,T,nil).

construct([],T,T).
construct([N|Ns],T,T0) :- add(N,T0,T1), construct(Ns,T,T1).
%! \begin{hint} 

layout_binary_tree(T,PT) :- sorry.
  % PT is the "positionned" binary
  % tree obtained from the binary tree T. (+,?) or (?,+)

layout_binary_tree(T,PT,In,Out,D) :- sorry.
  % T and PT as in layout_binary_tree/2;
  % In is the position in the inorder sequence where the tree T (or PT)
  % begins, Out is the position after the last node of T (or PT) in the 
  % inorder sequence. D is the depth of the root of T (or PT). 
  % (+,?,+,?,+) or (?,+,+,?,+)

%! \end{hint} 
%! \begin{solution}

layout_binary_tree(T,PT) :- layout_binary_tree(T,PT,1,_,1).
 
layout_binary_tree(nil,nil,I,I,_).
layout_binary_tree(t(W,L,R),t(W,X,Y,PL,PR),Iin,Iout,Y) :- 
   Y1 is Y + 1,
   layout_binary_tree(L,PL,Iin,X,Y1), 
   X1 is X + 1,
   layout_binary_tree(R,PR,X1,Iout,Y1).
%! \end{solution}
?-  construct([n,k,m,c,a,h,g,e,u,p,s,q],T),layout_binary_tree(T,PT).

(Intermediate ⭐️⭐️) Layout a binary tree (2).

An alternative layout method is depicted in the illustration opposite. Find out the rules and write the corresponding Prolog predicate. Hint: On a given level, the horizontal distance between neighbouring nodes is constant.

:- module(_, _, [assertions]). 

:- push_prolog_flag(multi_arity_warnings, off).

sorry :- throw(not_solved_yet). 

:- test layout_binary_tree2(T,PT) : (T = nil) => (PT = nil) + (not_fails, is_det).

:- test layout_binary_tree2(T,PT) : (T = t(n,t(k,t(c,t(a,nil,nil),t(h,t(g,t(e,nil,nil),nil),nil)),t(m,nil,nil)),t(u,t(p,nil,t(s,t(q,nil,nil),nil)),nil))) => (PT = t(n,29,1,t(k,13,2,t(c,5,3,t(a,1,4,nil,nil),t(h,9,4,t(g,7,5,t(e,6,6,nil,nil),nil),nil)),t(m,21,3,nil,nil)),t(u,45,2,t(p,37,3,nil,t(s,41,4,t(q,39,5,nil,nil),nil)),nil))) + (not_fails, is_det). 

add(X,nil,t(X,nil,nil)).
add(X,t(Root,L,R),t(Root,L1,R)) :- X @< Root, add(X,L,L1).
add(X,t(Root,L,R),t(Root,L,R1)) :- X @> Root, add(X,R,R1).

construct(L,T) :- construct(L,T,nil).

construct([],T,T).
construct([N|Ns],T,T0) :- add(N,T0,T1), construct(Ns,T,T1).
%! \begin{hint} 

x_pos(T,X,D) :- sorry.
  % X is the horizontal position of the root node of T
  % with respect to the picture co-ordinate system. D is the horizontal
  % distance between the root node of T and its successor(s) (if any).
  % (+,-,+)

layout_binary_tree2(T,PT,X,Y,D) :- sorry.
  % T and PT as in layout_binary_tree/2;
  % D is the the horizontal distance between the root node of T and 
  % its successor(s) (if any). X, Y are the co-ordinates of the root node.
  % (+,-,+,+,+)

hor_dist(T,D4) :- sorry.
  % D4 is four times the horizontal distance between the 
  %    root node of T and its successor(s) (if any).
  %    (+,-)
  
layout_binary_tree2(T,PT) :- sorry.
  % PT is the "positionned" binary
  % tree obtained from the binary tree T. (+,?)
%! \end{hint} 
%! \begin{solution}

layout_binary_tree2(nil,nil) :- !. 
layout_binary_tree2(T,PT) :- 
   hor_dist(T,D4), D is D4//4, x_pos(T,X,D), 
   layout_binary_tree2(T,PT,X,1,D).

hor_dist(nil,1).
hor_dist(t(_,L,R),D4) :- 
   hor_dist(L,D4L), 
   hor_dist(R,D4R),
   max(D4L,D4R,M),
   D4 is 2 * M.
   
max(X,Y,X):- X >= Y, !.
max(_X,Y,Y).

x_pos(t(_,nil,_),1,_) :- !.
x_pos(t(_,L,_),X,D) :- D2 is D//2, x_pos(L,XL,D2), X is XL+D.

layout_binary_tree2(nil,nil,_,_,_).
layout_binary_tree2(t(W,L,R),t(W,X,Y,PL,PR),X,Y,D) :- 
   Y1 is Y + 1,
   Xleft is X - D,
   D2 is D//2,
   layout_binary_tree2(L,PL,Xleft,Y1,D2), 
   Xright is X + D,
   layout_binary_tree2(R,PR,Xright,Y1,D2).
%! \end{solution}
?- construct([n,k,m,c,a,e,d,g,u,p,q],T),layout_binary_tree2(T,PT).
Hint:Use the same conventions as in Layout a binary tree (1) problem and test your predicate in an appropriate way.

(Hard ⭐️⭐️⭐️) Layout a binary tree (3)

Yet another layout strategy is shown in the illustration opposite. The method yields a very compact layout while maintaining a certain symmetry in every node. Find out the rules and write the corresponding Prolog predicate. Use the same conventions as in Layout a binary tree (1) and (2) problems and test your predicate in an appropriate way. Note: This is a difficult problem. Don't give up too early! Which layout do you like most?

:- module(_, _, [assertions]). 

sorry :- throw(not_solved_yet). 

:- test layout_binary_tree3(T,PT) : (T = nil) => (PT = nil) + (not_fails, is_det).

:- test layout_binary_tree3(T,PT) : (T = t(n,t(k,t(c,t(a,nil,nil),t(h,t(g,t(e,nil,nil),nil),nil)),t(m,nil,nil)),t(u,t(p,nil,t(s,t(q,nil,nil),nil)),nil))) => (PT = t(n,5,1,t(k,3,2,t(c,2,3,t(a,1,4,nil,nil),t(h,3,4,t(g,2,5,t(e,1,6,nil,nil),nil),nil)),t(m,4,3,nil,nil)),t(u,7,2,t(p,6,3,nil,t(s,7,4,t(q,6,5,nil,nil),nil)),nil))) + (not_fails, is_det). 

add(X,nil,t(X,nil,nil)).
add(X,t(Root,L,R),t(Root,L1,R)) :- X @< Root, add(X,L,L1).
add(X,t(Root,L,R),t(Root,L,R1)) :- X @> Root, add(X,R,R1).

construct(L,T) :- construct(L,T,nil).

construct([],T,T).
construct([N|Ns],T,T0) :- add(N,T0,T1), construct(Ns,T,T1).
%! \begin{hint} 

layout_binary_tree3(T,PT) :- sorry.
  % PT is the "positionned" binary
  % tree obtained from the binary tree T. (+,?)

%! \end{hint} 
%! \begin{solution}
layout_binary_tree3(nil,nil) :- !. 
layout_binary_tree3(T,PT) :-
   contour_tree(T,CT),      % construct the "contour" tree CT
   CT = t(_,_,_,Contour),
   mincont(Contour,MC,0),   % find the position of the leftmost node
   Xroot is 1-MC,
   layout_binary_tree3(CT,PT,Xroot,1).

contour_tree(nil,nil).
contour_tree(t(X,L,R),t(X,CL,CR,Contour)) :- 
   contour_tree(L,CL),
   contour_tree(R,CR),
   combine(CL,CR,Contour).

combine(nil,nil,[]).
combine(t(_,_,_,CL),nil,[c(-1,-1)|Cs]) :- shift(CL,-1,Cs).
combine(nil,t(_,_,_,CR),[c(1,1)|Cs]) :- shift(CR,1,Cs).
combine(t(_,_,_,CL),t(_,_,_,CR),[c(DL,DR)|Cs]) :-
   maxdiff(CL,CR,MD,0), 
   DR is (MD+2)//2, DL is -DR,
   merge(CL,CR,DL,DR,Cs).

shift([],_,[]).
shift([c(L,R)|Cs],S,[c(LS,RS)|CsS]) :-
   LS is L+S, RS is R+S, shift(Cs,S,CsS).

maxdiff([],_,MD,MD) :- !.
maxdiff(_,[],MD,MD) :- !.
maxdiff([c(_,R1)|Cs1],[c(L2,_)|Cs2],MD,A) :- 
   Su is R1-L2,
   max(A,Su,A1),
   maxdiff(Cs1,Cs2,MD,A1).

max(X,Y,X):- X >= Y, !.
max(_X,Y,Y).

merge([],CR,_,DR,Cs) :- !, shift(CR,DR,Cs).
merge(CL,[],DL,_,Cs) :- !, shift(CL,DL,Cs).
merge([c(L1,_)|Cs1],[c(_,R2)|Cs2],DL,DR,[c(L,R)|Cs]) :-
   L is L1+DL, R is R2+DR,
   merge(Cs1,Cs2,DL,DR,Cs).

mincont([],MC,MC).
mincont([c(L,_)|Cs],MC,A) :- 
    min(A,L,A1), 
    mincont(Cs,MC,A1).

min(X,Y,X):- X < Y, !.
min(_X,Y,Y).

layout_binary_tree3(nil,nil,_,_).
layout_binary_tree3(t(W,nil,nil,_),t(W,X,Y,nil,nil),X,Y) :- !. 
layout_binary_tree3(t(W,L,R,[c(DL,DR)|_]),t(W,X,Y,PL,PR),X,Y) :- 
   Y1 is Y + 1,
   Xleft is X + DL,
   layout_binary_tree3(L,PL,Xleft,Y1), 
   Xright is X + DR,
   layout_binary_tree3(R,PR,Xright,Y1).

%! \end{solution}
?- construct([n,k,m,c,a,e,d,g,u,p,q],T),layout_binary_tree3(T,PT).
Hint: Consider the horizontal distance between a node and its successor nodes. How tight can you pack together two subtrees to construct the combined binary tree?

String representation

(Intermediate ⭐️⭐️) A string representation of binary trees.

Somebody represents binary trees as strings of the following type (see example opposite): a(b(d,e),c(,f(g,)))

[a)] Write a Prolog predicate which generates this string representation, if the tree is given as usual (as nil or t(X,L,R) term). Then write a predicate which does this inverse; i.e. given the string representation, construct the tree in the usual form. Finally, combine the two predicates in a single predicate tree_string/2 which can be used in both directions.

:- module(_, _, [assertions]). 

:- use_module(library(classic/classic_predicates),[append/3, atom_chars/2]).

:- push_prolog_flag(multi_arity_warnings, off).

sorry :- throw(not_solved_yet). 

:- test tree_string(T,ST) : (T = nil) => (ST = '') + (not_fails, is_det).

:- test tree_string(T,ST) : (T = t(x, t(x, t(x, nil, nil), t(x, nil, nil)), t(x, t(x, nil, nil),t(x, nil, nil)))) => (PT = 'x(x(x,x),x(x,x))') + (not_fails, is_det).

%! \begin{hint} 
% The string representation has the following syntax:
%
% <tree> ::=  | <letter><subtrees>
%
% <subtrees> ::=  | '(' <tree> ',' <tree> ')'
%
% According to this syntax, a leaf node (with letter x) could
% be represented by x(,) and not only by the single character x.
% However, we will avoid this when generating the string 
% representation.

tree_string(T,S) :- sorry.

%! \end{hint} 
%! \begin{solution}
tree_string(T,S) :- nonvar(T), !, tree_to_string(T,S). 
tree_string(T,S) :- nonvar(S), string_to_tree(S,T). 

tree_to_string(T,S) :- tree_to_list(T,L), atom_chars(S,L).

tree_to_list(nil,[]).
tree_to_list(t(X,nil,nil),[X]) :- !.
tree_to_list(t(X,L,R),[X,'('|List]) :- 
   tree_to_list(L,LsL),
   tree_to_list(R,LsR),
   append(LsL,[','],List1),
   append(List1,LsR,List2),
   append(List2,[')'],List).

string_to_tree(S,T) :- atom_chars(S,L), list_to_tree(L,T).

list_to_tree([],nil).
list_to_tree([X],t(X,nil,nil)) :- character_code(X).
list_to_tree([X,'('|List],t(X,Left,Right)) :- character_code(X),
   append(List1,[')'],List),
   append(LeftList,[','|RightList],List1),
   list_to_tree(LeftList,Left),
   list_to_tree(RightList,Right).
%! \end{solution}
?- tree_string(t(n,t(k,t(c,t(a,nil,nil),t(h,t(g,t(e,nil,nil),nil),nil)),t(m,nil,nil)),t(u,t(p,nil,t(s,t(q,nil,nil),nil)),nil)),A).
[b)] Write the same predicate tree_string/2 using difference lists and a single predicate tree_dlist/2 which does the conversion between a tree and a difference list in both directions. For simplicity, suppose the information in the nodes is a single letter and there are no spaces in the string.
:- module(_, _, [assertions]). 

:- use_module(library(classic/classic_predicates),[append/3, atom_chars/2]).

:- push_prolog_flag(multi_arity_warnings, off).

sorry :- throw(not_solved_yet). 

:- test tree_string(T,ST) : (T = nil) => (ST = '') + (not_fails, is_det).

:- test tree_string(T,ST) : (T = t(x, t(x, t(x, nil, nil), t(x, nil, nil)), t(x, t(x, nil, nil),t(x, nil, nil)))) => (PT = 'x(x(x,x),x(x,x))') + (not_fails, is_det).

%! \begin{hint} 

tree_string(T,S) :- sorry.

%! \end{hint} 
%! \begin{solution}
% Most elegant solution using difference lists.

tree_string(T,S) :- nonvar(T), tree_dlist(T,L-[]), !, atom_chars(S,L). 
tree_string(T,S) :- nonvar(S), atom_chars(S,L), tree_dlist(T,L-[]).

% tree_dlist/2 does the trick in both directions!

tree_dlist(nil,L-L).
tree_dlist(t(X,nil,nil),L1-L2) :- 
   letter(X,L1-L2).
tree_dlist(t(X,Left,Right),L1-L7) :- 
   letter(X,L1-L2), 
   symbol('(',L2-L3),
   tree_dlist(Left,L3-L4),
   symbol(',',L4-L5),
   tree_dlist(Right,L5-L6),
   symbol(')',L6-L7).

symbol(X,[X|Xs]-Xs).

letter(X,L1-L2) :- symbol(X,L1-L2).
%! \end{solution}
?- tree_string(t(n,t(k,t(c,t(a,nil,nil),t(h,t(g,t(e,nil,nil),nil),nil)),t(m,nil,nil)),t(u,t(p,nil,t(s,t(q,nil,nil),nil)),nil)),A).

Preorder and inorder

(Intermediate ⭐️⭐️) Preorder and inorder sequences of binary trees. We consider binary trees with nodes that are identified by single lower-case letters, as in the example of String representation of binary trees problem.

[a)] Write predicates preorder/2 and inorder/2 that construct the preorder and inorder sequence of a given binary tree, respectively. The results should be atoms, e.g. 'abdecfg' for the preorder sequence of the example in String representation of binary trees problem.

:- module(_, _, [assertions]). 

:- use_module(library(classic/classic_predicates),[append/3, atom_chars/2]).

sorry :- throw(not_solved_yet). 

:- test preorder(T,S) : (T = nil) => (S = '') + (not_fails, is_det).

:- test preorder(T,S) : (T = t(n,t(k,t(c,t(a,nil,nil),t(h,t(g,t(e,nil,nil),nil),nil)),t(m,nil,nil)),t(u,t(p,nil,t(s,t(q,nil,nil),nil)),nil))) => (S = nkcahgemupsq) + (not_fails, is_det).

%! \begin{hint} 

preorder(T,S) :- sorry.

%! \end{hint} 
%! \begin{solution}
preorder(T,S) :- preorder_tl(T,L), atom_chars(S,L).

preorder_tl(nil,[]).
preorder_tl(t(X,Left,Right),[X|List]) :-
   preorder_tl(Left,ListLeft),
   preorder_tl(Right,ListRight),
   append(ListLeft,ListRight,List).

inorder(T,S) :- inorder_tl(T,L), atom_chars(S,L).

inorder_tl(nil,[]).
inorder_tl(t(X,Left,Right),List) :-
   inorder_tl(Left,ListLeft),
   inorder_tl(Right,ListRight),
   append(ListLeft,[X|ListRight],List).
%! \end{solution}
?- preorder(t(n,t(k,t(c,t(a,nil,nil),t(h,t(g,t(e,nil,nil),nil),nil)),t(m,nil,nil)),t(u,t(p,nil,t(s,t(q,nil,nil),nil)),nil)),A).
[b)] Can you use preorder/2 from problem part a) in the reverse direction; i.e. given a preorder sequence, construct a corresponding tree? If not, make the necessary arrangements.
:- module(_, _, [assertions]). 

:- use_module(library(classic/classic_predicates),[append/3, atom_chars/2]).

sorry :- throw(not_solved_yet). 

:- test preorder(T,S) : (T = nil) => (S = '') + (not_fails, is_det).

:- test preorder(T,S) : (T = t(n,t(k,t(c,t(a,nil,nil),t(h,t(g,t(e,nil,nil),nil),nil)),t(m,nil,nil)),t(u,t(p,nil,t(s,t(q,nil,nil),nil)),nil))) => (S = nkcahgemupsq) + (not_fails, is_det).

:- test inorder(T,S) : (T = nil) => (S = '') + (not_fails, is_det).

:- test inorder(T,S) : (T = t(n,t(k,t(c,t(a,nil,nil),t(h,t(g,t(e,nil,nil),nil),nil)),t(m,nil,nil)),t(u,t(p,nil,t(s,t(q,nil,nil),nil)),nil))) => (S = aceghkmnpqsu) + (not_fails, is_det).
%! \begin{hint} 
inorder(T,S) :- sorry.
  % S is the inorder tre traversal sequence of the
  % nodes of the binary tree T. (tree,atom) (+,?) or (?,+)

preorder(T,S) :- sorry.
  % S is the preorder tre traversal sequence of the
  % nodes of the binary tree T. (tree,atom) (+,?) or (?,+)
%! \end{hint} 
%! \begin{solution}
preorder(T,S) :- nonvar(T), !, preorder_tl(T,L), atom_chars(S,L). 
preorder(T,S) :- atom(S), atom_chars(S,L), preorder_lt(T,L).

preorder_tl(nil,[]).
preorder_tl(t(X,Left,Right),[X|List]) :-
   preorder_tl(Left,ListLeft),
   preorder_tl(Right,ListRight),
   append(ListLeft,ListRight,List).

preorder_lt(nil,[]).
preorder_lt(t(X,Left,Right),[X|List]) :-
   append(ListLeft,ListRight,List),
   preorder_lt(Left,ListLeft),
   preorder_lt(Right,ListRight).

inorder(T,S) :- nonvar(T), !, inorder_tl(T,L), atom_chars(S,L). 
inorder(T,S) :- atom(S), atom_chars(S,L), inorder_lt(T,L).

inorder_tl(nil,[]).
inorder_tl(t(X,Left,Right),List) :-
   inorder_tl(Left,ListLeft),
   inorder_tl(Right,ListRight),
   append(ListLeft,[X|ListRight],List).

inorder_lt(nil,[]).
inorder_lt(t(X,Left,Right),List) :-
   append(ListLeft,[X|ListRight],List),
   inorder_lt(Left,ListLeft),
   inorder_lt(Right,ListRight).
%! \end{solution}
?- preorder(t(n,t(k,t(c,t(a,nil,nil),t(h,t(g,t(e,nil,nil),nil),nil)),t(m,nil,nil)),t(u,t(p,nil,t(s,t(q,nil,nil),nil)),nil)),A).
?- inorder(t(n,t(k,t(c,t(a,nil,nil),t(h,t(g,t(e,nil,nil),nil),nil)),t(m,nil,nil)),t(u,t(p,nil,t(s,t(q,nil,nil),nil)),nil)),A).
[c)] If both the preorder sequence and the inorder sequence of the nodes of a binary tree are given, then the tree is determined unambiguously. Write a predicate pre_in_tree/3 that does the job.
:- module(_, _, [assertions]). 

:- use_module(library(classic/classic_predicates),[append/3, atom_chars/2]).

sorry :- throw(not_solved_yet). 

:- test pre_in_tree(A,B,C) : (A = abc, B = bca) => (C = t(a,t(b,nil,t(c,nil,nil)),nil)) + (not_fails, is_det).

preorder(T,S) :- nonvar(T), !, preorder_tl(T,L), atom_chars(S,L). 
preorder(T,S) :- atom(S), atom_chars(S,L), preorder_lt(T,L).

preorder_tl(nil,[]).
preorder_tl(t(X,Left,Right),[X|List]) :-
   preorder_tl(Left,ListLeft),
   preorder_tl(Right,ListRight),
   append(ListLeft,ListRight,List).

preorder_lt(nil,[]).
preorder_lt(t(X,Left,Right),[X|List]) :-
   append(ListLeft,ListRight,List),
   preorder_lt(Left,ListLeft),
   preorder_lt(Right,ListRight).

inorder(T,S) :- nonvar(T), !, inorder_tl(T,L), atom_chars(S,L). 
inorder(T,S) :- atom(S), atom_chars(S,L), inorder_lt(T,L).

inorder_tl(nil,[]).
inorder_tl(t(X,Left,Right),List) :-
   inorder_tl(Left,ListLeft),
   inorder_tl(Right,ListRight),
   append(ListLeft,[X|ListRight],List).

inorder_lt(nil,[]).
inorder_lt(t(X,Left,Right),List) :-
   append(ListLeft,[X|ListRight],List),
   inorder_lt(Left,ListLeft),
   inorder_lt(Right,ListRight).
%! \begin{hint} 

pre_in_tree(P,I,T) :- sorry.
  % T is the binary tree that has the preorder
  % sequence P and inorder sequence I.
  % (atom,atom,tree) (+,+,?)

%! \end{hint} 
%! \begin{solution}

pre_in_tree(P,I,T) :- preorder(T,P), inorder(T,I).

% This is a nice application of the generate-and-test method.

% We can push the tester inside the generator in order to get
% a (much) better performance.

pre_in_tree_push(P,I,T) :- 
   atom_chars(P,PL), atom_chars(I,IL), pre_in_tree_pu(PL,IL,T).

pre_in_tree_pu([],[],nil).
pre_in_tree_pu([X|PL],IL,t(X,Left,Right)) :- 
   append(ILeft,[X|IRight],IL),
   append(PLeft,PRight,PL),
   pre_in_tree_pu(PLeft,ILeft,Left),
   pre_in_tree_pu(PRight,IRight,Right).
   
% Nice. But there is a still better solution. See problem d)!
%! \end{solution}
?- pre_in_tree(abc,bca,B).
[d)] Solve problems a) to c) using difference lists. Cool! Use the predefined predicate time/1 to compare the solutions.
:- module(_, _, [assertions]). 

:- use_module(library(classic/classic_predicates),[atom_chars/2]).

sorry :- throw(not_solved_yet). 

:- test pre_in_tree_d(A,B,C) : (A = abc, B = bca) => (C = t(a,t(b,nil,t(c,nil,nil)),nil)) + (not_fails, is_det).

%! \begin{hint} 
pre_in_tree_d(P,I,T) :- sorry.
  % T is the binary tree that has the preorder
  % sequence P and inorder sequence I.
  % (atom,atom,tree) (+,+,?)

%! \end{hint} 
%! \begin{solution}
pre_in_tree_d(P,I,T) :-  
   atom_chars(P,PL), atom_chars(I,IL), pre_in_tree_dl(PL-[],IL-[],T).

pre_in_tree_dl(P-P,I-I,nil).
pre_in_tree_dl(P1-P4,I1-I4,t(X,Left,Right)) :-
   symbol(X,P1-P2), symbol(X,I2-I3),
   pre_in_tree_dl(P2-P3,I1-I2,Left),
   pre_in_tree_dl(P3-P4,I3-I4,Right).

symbol(X,[X|Xs]-Xs).

% Isn't it cool? But the best of it is the performance!

% With the generate-and-test solution (p68c):
% ?- time(pre_in_tree(abdecfg,dbeacgf,_)).
% 9,048 inferences in 0.01 seconds (904800 Lips)  

% With the "pushed" generate-and-test solution (p68c):
% ?- time(pre_in_tree_push(abdecfg,dbeacgf,_)).
% 67 inferences in 0.00 seconds (Infinite Lips)
  
% With the difference list solution (p68d):
% ?- time(pre_in_tree_d(abdecfg,dbeacgf,_)).
% 32 inferences in 0.00 seconds (Infinite Lips)                     

% Note that the predicate pre_in_tree_dl/3 runs in almost any
% flow pattern. Try it out!

%! \end{solution}
What happens if the same character appears in more than one node. Try for instance:
?- pre_in_tree_d(aba,baa,T).

Dotstring representation

(Intermediate ⭐️⭐️) Dotstring representation of binary trees We consider again binary trees with nodes that are identified by single lower-case letters, as in the example of String representation of binary trees problem. Such a tree can be represented by the preorder sequence of its nodes in which dots (.) are inserted where an empty subtree (nil) is encountered during the tree traversal. For example, the tree shown in String representation of binary trees problem is represented as 'abd..e..c.fg...'. First, try to establish a syntax (BNF or syntax diagrams) and then write a predicate tree_dotstring/2 which does the conversion in both directions. Use difference lists.

:- module(_, _, [assertions]). 

:- use_module(library(classic/classic_predicates),[atom_chars/2]).

sorry :- throw(not_solved_yet). 

:- test tree_dotstring(A,B) : (B = 'abd..e..c.fg...') => (A = t(a,t(b,t(d,nil,nil),t(e,nil,nil)),t(c,nil,t(f,t(g,nil,nil),nil)))) + (not_fails, is_det).

:- test tree_dotstring(A,B) : (A = nil) => (B = '.') + (not_fails, is_det).
%! \begin{hint} 
% The syntax of the dotstring representation is super simple:
%
% <tree> ::= . | <letter> <tree> <tree>

tree_dotstring(T,S) :- sorry.
%! \end{hint} 
%! \begin{solution}
tree_dotstring(T,S) :- nonvar(T), !, tree_dots_dl(T,L-[]), atom_chars(S,L). 
tree_dotstring(T,S) :- atom(S), atom_chars(S,L), tree_dots_dl(T,L-[]).

tree_dots_dl(nil,L1-L2) :- symbol('.',L1-L2).
tree_dots_dl(t(X,Left,Right),L1-L4) :- 
   letter(X,L1-L2),
   tree_dots_dl(Left,L2-L3),
   tree_dots_dl(Right,L3-L4).

symbol(X,[X|Xs]-Xs).

letter(X,L1-L2) :- symbol(X,L1-L2).
%! \end{solution}
?- tree_dotstring(A,'abd..e..c.fg...').

Multiway Trees

A multiway tree is composed of a root element and a (possibly empty) set of successors which are multiway trees themselves. A multiway tree is never empty. The set of successor trees is sometimes called a forest.

In Prolog we represent a multiway tree by a term t(X,F), where X denotes the root node and F denotes the forest of successor trees (a Prolog list). The example tree depicted opposite is therefore represented by the following Prolog term:

T = t(a,[t(f,[t(g,[])]),t(c,[]),t(b,[t(d,[]),t(e,[])])])

Is it a multiway tree?

(Easy⭐️) Check whether a given term represents a multiway tree. Write a predicate istree/1 which succeeds if and only if its argument is a Prolog term representing a multiway tree.
:- module(_, _, [assertions]). 

sorry :- throw(not_solved_yet). 

:- test istree(A) : (A = t(a,[t(f,[t(g,[])]),t(c,[]),t(b,[t(d,[]),t(e,[])])])) + (not_fails, is_det).

:- test istree(A) : (A = t(_,[])) + (not_fails, is_det).
%! \begin{hint}

istree(T) :- sorry.
  % T is a term representing a multiway tree (i), (o)

%! \end{hint}
%! \begin{solution}
istree(t(_,F)) :- isforest(F).

isforest([]).
isforest([T|Ts]) :- istree(T), isforest(Ts).
%! \end{solution}
?- istree(t(a,[t(f,[t(g,[])]),t(c,[]),t(b,[t(d,[]),t(e,[])])])).

Number of nodes

(Easy ⭐️) Count the nodes of a multiway tree. Write a predicate nnodes/1 which counts the nodes of a given multiway tree. Write another version of the predicate that allows for a flow pattern (o,i).
:- module(_, _, [assertions]). 

:- use_module(library(classic/classic_predicates), [between/3]).

sorry :- throw(not_solved_yet). 

:- test nnodes(A,N) : (A = t(a,[t(f,[])])) => (N = 2) + (not_fails, is_det).

%! \begin{hint}

nnodes(T,N) :- sorry.
  % the multiway tree T has N nodes (i,o))

%! \end{hint}
%! \begin{solution}
nnodes(t(_,F),N) :- nnodes(F,NF), N is NF+1.

nnodes([],0).
nnodes([T|Ts],N) :- nnodes(T,NT), nnodes(Ts,NTs), N is NT+NTs.

% Note that nnodes is called for trees and forests. An early
% form of polymorphism!

% For the flow pattern (o,i) we can write:

nnodes2(t(_,F),N) :- N > 0, NF is N-1, nnodes2F(F,NF).

nnodes2F([],0).
nnodes2F([T|Ts],N) :- N > 0, 
   between(1,N,NT), nnodes2(T,NT), 
   NTs is N-NT, nnodes2F(Ts,NTs).
%! \end{solution}
?- nnodes(t(a,[t(f,[])]),N).

Tree construction

(Intermediate ⭐️⭐️) Tree construction from a node string. We suppose that the nodes of a multiway tree contain single characters. In the depth-first order sequence of its nodes, a special character ^ has been inserted whenever, during the tree traversal, the move is a backtrack to the previous level.

By this rule, the tree in the figure opposite is represented as: afg^^c^bd^e^^^

Define the syntax of the string and write a predicate tree(String,Tree) to construct the Tree when the String is given. Work with atoms (instead of strings). Make your predicate work in both directions.

:- module(_, _, [assertions]). 

:- use_module(library(classic/classic_predicates), [append/3, atom_chars/2]).

sorry :- throw(not_solved_yet). 

:- test tree(A,B) : (B = t(a,[t(f,[t(g,[])]),t(c,[]),t(b,[t(d,[]),t(e,[])])])) => (A = 'afg^^c^bd^e^^^') + (not_fails, is_det).
%! \begin{hint}
% Syntax in BNF:

% <tree> ::= <letter> <forest> '^'

% <forest> ::= | <tree> <forest> 

tree(TS,T) :- sorry.
%! \end{hint}
%! \begin{solution}
% First a nice solution using difference lists

tree(TS,T) :- atom(TS), !, atom_chars(TS,TL), tree_d(TL-[],T). % (+,?)
tree(TS,T) :- nonvar(T), tree_d(TL-[],T), atom_chars(TS,TL).   % (?,+)

tree_d([X|F1]-T, t(X,F)) :- forest_d(F1-['^'|T],F).

forest_d(F-F,[]).
forest_d(F1-F3,[T|F]) :- tree_d(F1-F2,T), forest_d(F2-F3,F).


% Another solution, not as elegant as the previous one.

tree_2(TS,T) :- atom(TS), !, atom_chars(TS,TL), tree_a(TL,T). % (+,?)
tree_2(TS,T) :- nonvar(T), tree_a(TL,T), atom_chars(TS,TL).   % (?,+)

tree_a(TL,t(X,F)) :- 
   append([X],FL,L1), append(L1,['^'],TL), forest_a(FL,F).

forest_a([],[]).
forest_a(FL,[T|Ts]) :- append(TL,TsL,FL), 
   tree_a(TL,T), forest_a(TsL,Ts).
%! \end{solution}
?- tree(A,t(a,[t(f,[t(g,[])]),t(c,[]),t(b,[t(d,[]),t(e,[])])])).

Internal path

(Easy ⭐️) Determine the internal path length of a tree. We define the internal path length of a multiway tree as the total sum of the path lengths from the root to all nodes of the tree. By this definition, the tree in the figure at the beginning of the section has an internal path length of 9. Write a predicate ipl(Tree,IPL) for the flow pattern (+,-).
:- module(_, _, [assertions]). 

:- push_prolog_flag(multi_arity_warnings, off).

sorry :- throw(not_solved_yet). 

:- test ipl(A,B) : (A = t(a,[t(f,[t(g,[])]),t(c,[]),t(b,[t(d,[]),t(e,[])])])) => (B = 9)+ (not_fails, is_det).
%! \begin{hint}

ipl(Tree,L) :- sorry.
  % L is the internal path length of the tree Tree
  % (multiway-tree, integer) (+,?)

%! \end{hint}
%! \begin{solution}
ipl(T,L) :- ipl(T,0,L).

ipl(t(_,F),D,L) :- D1 is D+1, ipl(F,D1,LF), L is LF+D.

ipl([],_,0).
ipl([T1|Ts],D,L) :- ipl(T1,D,L1), ipl(Ts,D,Ls), L is L1+Ls.

% Notice the polymorphism: ipl is called with trees and with forests
% as first argument.
%! \end{solution}
?- ipl(t(a,[t(f,[t(g,[])]),t(c,[]),t(b,[t(d,[]),t(e,[])])]),B).

Bottom-up order sequence

(Easy ⭐️) Construct the bottom-up order sequence of the tree nodes. Write a predicate bottom_up(Tree,Seq) which constructs the bottom-up sequence of the nodes of the multiway tree Tree. Seq should be a Prolog list. What happens if you run your predicate backwards?
:- module(_, _, [assertions]). 

:- use_module(library(classic/classic_predicates), [append/3]).

:- push_prolog_flag(multi_arity_warnings, off).

sorry :- throw(not_solved_yet). 

:- test bottom_up(A,B) : (A = t(a,[t(f,[t(g,[])]),t(c,[]),t(b,[t(d,[]),t(e,[])])])) => (B = [g,f,c,d,e,b,a])+ (not_fails, is_det).
%! \begin{hint}

bottom_up(Tree,Seq) :- sorry.
  % Seq is the bottom-up sequence of the nodes of
  % the multiway tree Tree. (+,?)

%! \end{hint}
%! \begin{solution}
bottom_up_f(t(X,F),Seq) :- 
        bottom_up_f(F,SeqF), append(SeqF,[X],Seq).

bottom_up_f([],[]).
bottom_up_f([T|Ts],Seq):-
        bottom_up_f(T,SeqT), bottom_up_f(Ts,SeqTs), append(SeqT,SeqTs,Seq).

% The predicate bottom_up/2 produces a stack overflow when called
% in the (-,+) flow pattern. There are two problems with that.
% First, the polymorphism does not work properly, because during
% decomposing the string, the program cannot guess whether it should
% construct a tree or a forest next. We can fix this using two
% separate predicates bottom_up_tree/2 and bottom_up_forset/2.
% Secondly, if we maintain the order of the subgoals, then
% the interpreter falls into an endless loop after finding the
% first solution. We can fix this by changing the order of the
% goals as follows:

bottom_up_tree(t(X,F),Seq) :- 
        append(SeqF,[X],Seq), bottom_up_forest(F,SeqF).

bottom_up_forest([],[]).
bottom_up_forest([T|Ts],Seq):-
        append(SeqT,SeqTs,Seq),
        bottom_up_tree(T,SeqT), bottom_up_forest(Ts,SeqTs).

% Unfortunately, this version doesn't run in both directions either.

% In order to have a predicate which runs forward and backward, we
% have to determine the flow pattern and then call one of the above
% predicates, as follows:

bottom_up(T,Seq) :- nonvar(T), !, bottom_up_f(T,Seq).
bottom_up(T,Seq) :- nonvar(Seq), bottom_up_tree(T,Seq).

% This is not very elegant, I agree.
%! \end{solution}
?- bottom_up(t(a,[t(f,[t(g,[])]),t(c,[]),t(b,[t(d,[]),t(e,[])])]),B).

Lisp-like tree representation

(Intermediate ⭐️⭐️) Lisp-like tree representation. There is a particular notation for multiway trees in Lisp. Lisp is a prominent functional programming language, which is used primarily for artificial intelligence problems. As such it is one of the main competitors of Prolog. In Lisp, almost everything is a list, just as in Prolog everything is a term.

The following pictures show how multiway tree structures are represented in Lisp.

Note that in the "lispy" notation a node with successors (children) in the tree is always the first element in a list, followed by its children. The "lispy" representation of a multiway tree is a sequence of atoms and parentheses ( and ), which we shall collectively call "tokens". We can represent this sequence of tokens as a Prolog list; e.g. the lispy expression (a (b c)) could be represented as the Prolog list ['(', a, '(', b, c, ')', ')']. Write a predicate tree_ltl(T,LTL) which constructs the "lispy token list" LTL if the tree is given as term T in the usual Prolog notation.

:- module(_, _, [assertions]). 

:- use_module(library(classic/classic_predicates), [append/3]).

:- push_prolog_flag(multi_arity_warnings, off).

sorry :- throw(not_solved_yet). 

:- test tree_ltl(A,B) :(B = ['(', a, '(', b, c, ')', ')']) => (A = t(a,[t(b,[t(c,[])])])) + (not_fails, is_det).
%! \begin{hint}

tree_ltl(T,L) :- sorry.
  % L is the "lispy token list" of the multiway tree T

%! \end{hint}
%! \begin{solution}
tree_ltl(T,L) :- tree_ltl_d(T,L-[]).

% using difference lists

tree_ltl_d(t(X,[]),[X|L]-L) :- X \= '('.
tree_ltl_d(t(X,[T|Ts]),['(',X|L]-R) :- forest_ltl_d([T|Ts],L-[')'|R]).

forest_ltl_d([],L-L).
forest_ltl_d([T|Ts],L-R) :- tree_ltl_d(T,L-M), forest_ltl_d(Ts,M-R).

%! \end{solution}
?- tree_ltl(A,['(', a, '(', b, c, ')', ')']).
As a second, even more interesting exercise try to rewrite tree_ltl/2 in a way that the inverse conversion is also possible: Given the list LTL, construct the Prolog tree T. Use difference lists.

Graphs

A graph is defined as a set of nodes and a set of edges, where each edge is a pair of nodes. There are several ways to represent graphs in Prolog. One method is to represent each edge separately as one clause (fact). In this form, the graph depicted below is represented as the following predicate:

edge(h,g).
edge(k,f).
edge(f,b).
...
We call this edge-clause form. Obviously, isolated nodes cannot be represented. Another method is to represent the whole graph as one data object. According to the definition of the graph as a pair of two sets (nodes and edges), we may use the following Prolog term to represent the example graph:
graph([b,c,d,f,g,h,k],[e(b,c),e(b,f),e(c,f),e(f,k),e(g,h)])
We call this graph-term form. Note, that the lists are kept sorted, they are really sets, without duplicated elements. Each edge appears only once in the edge list; i.e. an edge from a node x to another node y is represented as e(x,y), the term e(y,x) is not present. The graph-term form is our default representation. In Ciao Prolog there are predefined predicates to work with sets.

A third representation method is to associate with each node the set of nodes that are adjacent to that node. We call this the adjacency-list form. In our example:

[n(b,[c,f]), n(c,[b,f]), n(d,[]), n(f,[b,c,k]), ...]
The representations we introduced so far are Prolog terms and therefore well suited for automated processing, but their syntax is not very user-friendly. Typing the terms by hand is cumbersome and error-prone. We can define a more compact and "human-friendly" notation as follows: A graph is represented by a list of atoms and terms of the type X-Y (i.e. functor - and arity 2). The atoms stand for isolated nodes, the X-Y terms describe edges. If an X appears as an endpoint of an edge, it is automatically defined as a node. Our example could be written as:
[b-c, f-c, g-h, d, f-b, k-f, h-g]
We call this the human-friendly form. As the example shows, the list does not have to be sorted and may even contain the same edge multiple times. Notice the isolated node d. (Actually, isolated nodes do not even have to be atoms in the Prolog sense, they can be compound terms, as in d(3.75,blue) instead of d in the example).

When the edges are directed we call them arcs. These are represented by ordered pairs. Such a graph is called directed graph. To represent a directed graph, the forms discussed above are slightly modified. The example graph opposite is represented as follows:

  • Arc-clause form:
arc(s,u).
arc(u,r).
...
  • Graph-term form:
digraph([r,s,t,u,v],[a(s,r),a(s,u),a(u,r),a(u,s),a(v,u)])
  • Adjacency-list form:
[n(r,[]),n(s,[r,u]),n(t,[]),n(u,[r]),n(v,[u])]
Note that the adjacency-list does not have the information on whether it is a graph or a digraph.

  • Human-friendly form:
[s > r, t, u > r, s > u, u > s, v > u]
Finally, graphs and digraphs may have additional information attached to nodes and edges (arcs). For the nodes, this is no problem, as we can easily replace the single character identifiers with arbitrary compound terms, such as city('London',4711). On the other hand, for edges we have to extend our notation. Graphs with additional information attached to edges are called labelled graphs.

  • Arc-clause form:
arc(m,q,7).
arc(p,q,9).
arc(p,m,5).
  • Graph-term form:
digraph([k,m,p,q],[a(m,p,7),a(p,m,5),a(p,q,9)])
  • Adjacency-list form:
[n(k,[]),n(m,[q/7]),n(p,[m/5,q/9]),n(q,[])]
Notice how the edge information has been packed into a term with functor / and arity 2, together with the corresponding node.

  • Human-friendly form:
[p>q/9, m>q/7, k, p>m/5]
The notation for labelled graphs can also be used for so-called multi-graphs, where more than one edge (or arc) are allowed between two given nodes.

Conversions

(Hard ⭐️⭐️⭐️) Conversions. Write predicates to convert between the different graph representations. With these predicates, all representations are equivalent; i.e. for the following problems you can always pick freely the most convenient form. The reason this problem is rated (Hard ⭐️⭐️⭐️) is not because it's particularly difficult, but because it's a lot of work to deal with all the special cases.

:- module(_, _, [assertions]). 

:- use_module(library(idlists), [memberchk/2]).
:- use_module(library(sort)).
:- use_module(library(aggregates), [findall/3]).
:- use_module(library(classic/classic_predicates), [delete/3]).

:- data edge/2.
:- data arc/2.

:- test alist_gterm(Type,AL,G1) : (Type = graph, AL = [n(b,[c,f]),n(c,[b,f]),n(d,[]),n(f,[b,c,k]),n(g,[h]),n(h,[g]),n(k,[f])]) => (G1 = graph([b,c,d,f,g,h,k],[e(b,c),e(b,f),e(c,f),e(f,k),e(g,h)]))+ (not_fails, is_det).

:- test alist_gterm(Type,AL,G1) : (Type = graph, AL = [n(t,[]),n(s>r,[]),n(s>u,[]),n(u>r,[]),n(u>s,[]),n(v>u,[])]) => (G1 = graph([t,s>r,s>u,u>r,u>s,v>u],[])) + (not_fails, is_det).

:- test alist_gterm(Type,AL,G1) : (Type = graph, AL = []) => (G1 = graph([],[])) + (not_fails, is_det).

sorry :- throw(not_solved_yet). 

%! \begin{hint} 
% We use the following notation:
%
% adjacency-list (alist): [n(b,[c,g,h]), n(c,[b,d,f,h]), n(d,[c,f]), ...]
%
% graph-term (gterm)  graph([b,c,d,f,g,h,k],[e(b,c),e(b,g),e(b,h), ...]) or
%                     digraph([r,s,t,u],[a(r,s),a(r,t),a(s,t), ...])
%
% edge-clause (ecl):  edge(b,g).  (in program database)
% arc-clause (acl):   arc(r,s).   (in program database)
%
% human-friendly (hf): [a-b,c,g-h,d-e]  or [a>b,h>g,c,b>a]
%
% The main conversion predicates are: alist_gterm/3 and human_gterm/2 which
% both (hopefully) work in either direction and for graphs as well as
% for digraphs, labelled or not.

alist_gterm(Type,AL,GT) :- sorry.
  % convert between adjacency-list and graph-term
  % representation. Type is either 'graph' or 'digraph'.
  % (atom,alist,gterm)  (+,+,?) or (?,?,+)
%! \end{hint} 
%! \begin{solution}
alist_gterm(Type,AL,GT):- nonvar(GT), !, gterm_to_alist(GT,Type,AL).
alist_gterm(Type,AL,GT):- atom(Type), nonvar(AL), alist_to_gterm(Type,AL,GT).

gterm_to_alist(graph(Ns,Es),graph,AL) :- memberchk(e(_,_,_),Es), ! ,
   lgt_al(Ns,Es,AL).
gterm_to_alist(graph(Ns,Es),graph,AL) :- !, 
   gt_al(Ns,Es,AL).
gterm_to_alist(digraph(Ns,As),digraph,AL) :- memberchk(a(_,_,_),As), !,
   ldt_al(Ns,As,AL).
gterm_to_alist(digraph(Ns,As),digraph,AL) :- 
   dt_al(Ns,As,AL).

% labelled graph
lgt_al([],_,[]).
lgt_al([V|Vs],Es,[n(V,L)|Ns]) :-
   findall(T,((member(e(X,V,I),Es) ; member(e(V,X,I),Es)),T = X/I),L),
   lgt_al(Vs,Es,Ns).

% unlabelled graph
gt_al([],_,[]).
gt_al([V|Vs],Es,[n(V,L)|Ns]) :-
   findall(X,(member(e(X,V),Es) ; member(e(V,X),Es)),L), gt_al(Vs,Es,Ns).

% labelled digraph
ldt_al([],_,[]).
ldt_al([V|Vs],As,[n(V,L)|Ns]) :-
   findall(T,(member(a(V,X,I),As), T=X/I),L), ldt_al(Vs,As,Ns).

% unlabelled digraph
dt_al([],_,[]).
dt_al([V|Vs],As,[n(V,L)|Ns]) :-
   findall(X,member(a(V,X),As),L), dt_al(Vs,As,Ns).


alist_to_gterm(graph,AL,graph(Ns,Es)) :- !, al_gt(AL,Ns,EsU,[]), sort(EsU,Es).
alist_to_gterm(digraph,AL,digraph(Ns,As)) :- al_dt(AL,Ns,AsU,[]), sort(AsU,As).

al_gt([],[],Es,Es).
al_gt([n(V,Xs)|Ns],[V|Vs],Es,Acc) :- 
   add_edges(V,Xs,Acc1,Acc), al_gt(Ns,Vs,Es,Acc1). 

add_edges(_,[],Es,Es).
add_edges(V,[X/_|Xs],Es,Acc) :- V @> X, !, add_edges(V,Xs,Es,Acc).
add_edges(V,[X|Xs],Es,Acc) :- V @> X, !, add_edges(V,Xs,Es,Acc).
add_edges(V,[X/I|Xs],Es,Acc) :- V @=< X, !, add_edges(V,Xs,Es,[e(V,X,I)|Acc]).
add_edges(V,[X|Xs],Es,Acc) :- V @=< X, add_edges(V,Xs,Es,[e(V,X)|Acc]).

al_dt([],[],As,As).
al_dt([n(V,Xs)|Ns],[V|Vs],As,Acc) :- 
   add_arcs(V,Xs,Acc1,Acc), al_dt(Ns,Vs,As,Acc1). 

add_arcs(_,[],As,As).
add_arcs(V,[X/I|Xs],As,Acc) :- !, add_arcs(V,Xs,As,[a(V,X,I)|Acc]).
add_arcs(V,[X|Xs],As,Acc) :- add_arcs(V,Xs,As,[a(V,X)|Acc]).

% ---------------------------------------------------------------------------

% ecl_to_gterm(GT) :- construct a graph-term from edge/2 facts in the
%    program database.

ecl_to_gterm(GT) :-
   findall(E,(edge(X,Y),E=X-Y),Es), human_gterm(Es,GT).

% acl_to_gterm(GT) :- construct a graph-term from arc/2 facts in the
%    program database.

acl_to_gterm(GT) :-
   findall(A,(arc(X,Y),A= >(X,Y)),As), human_gterm(As,GT).

% ---------------------------------------------------------------------------

% human_gterm(HF,GT) :- convert between human-friendly and graph-term
%    representation.
%    (list,gterm) (+,?) or (?,+)

human_gterm(HF,GT):- nonvar(GT), !, gterm_to_human(GT,HF).
human_gterm(HF,GT):- nonvar(HF), human_to_gterm(HF,GT).

gterm_to_human(graph(Ns,Es),HF) :-  memberchk(e(_,_,_),Es), !, 
   lgt_hf(Ns,Es,HF).
gterm_to_human(graph(Ns,Es),HF) :-  !, 
   gt_hf(Ns,Es,HF).
gterm_to_human(digraph(Ns,As),HF) :- memberchk(a(_,_,_),As), !, 
   ldt_hf(Ns,As,HF).
gterm_to_human(digraph(Ns,As),HF) :- 
   dt_hf(Ns,As,HF).

% labelled graph
lgt_hf(Ns,[],Ns).
lgt_hf(Ns,[e(X,Y,I)|Es],[X-Y/I|Hs]) :-
   delete(Ns,X,Ns1),
   delete(Ns1,Y,Ns2),
   lgt_hf(Ns2,Es,Hs).

% unlabelled graph
gt_hf(Ns,[],Ns).
gt_hf(Ns,[e(X,Y)|Es],[X-Y|Hs]) :-
   delete(Ns,X,Ns1),
   delete(Ns1,Y,Ns2),
   gt_hf(Ns2,Es,Hs).

% labelled digraph
ldt_hf(Ns,[],Ns).
ldt_hf(Ns,[a(X,Y,I)|As],[X>Y/I|Hs]) :-
   delete(Ns,X,Ns1),
   delete(Ns1,Y,Ns2),
   ldt_hf(Ns2,As,Hs).

% unlabelled digraph
dt_hf(Ns,[],Ns).
dt_hf(Ns,[a(X,Y)|As],[X>Y|Hs]) :-
   delete(Ns,X,Ns1),
   delete(Ns1,Y,Ns2),
   dt_hf(Ns2,As,Hs).

% we guess that if there is a '>' term then it's a digraph, else a graph
human_to_gterm(HF,digraph(Ns,As)) :- memberchk(_>_,HF), !, 
   hf_dt(HF,Ns1,As1), sort(Ns1,Ns), sort(As1,As).
human_to_gterm(HF,graph(Ns,Es)) :- 
   hf_gt(HF,Ns1,Es1), sort(Ns1,Ns), sort(Es1,Es).
% remember: sort/2 removes duplicates!

hf_gt([],[],[]).
hf_gt([X-Y/I|Hs],[X,Y|Ns],[e(U,V,I)|Es]) :- !, 
   sort0([X,Y],[U,V]), hf_gt(Hs,Ns,Es).
hf_gt([X-Y|Hs],[X,Y|Ns],[e(U,V)|Es]) :- !,
   sort0([X,Y],[U,V]), hf_gt(Hs,Ns,Es).
hf_gt([H|Hs],[H|Ns],Es) :- hf_gt(Hs,Ns,Es).

hf_dt([],[],[]).
hf_dt([X>Y/I|Hs],[X,Y|Ns],[a(X,Y,I)|As]) :- !, 
   hf_dt(Hs,Ns,As).
hf_dt([X>Y|Hs],[X,Y|Ns],[a(X,Y)|As]) :- !,
   hf_dt(Hs,Ns,As).
hf_dt([H|Hs],[H|Ns],As) :-  hf_dt(Hs,Ns,As).

sort0([X,Y],[X,Y]) :- X @=< Y, !.
sort0([X,Y],[Y,X]) :- X @> Y.
%! \end{solution}
?- human_gterm([b-c, f-c, g-h, d, f-b, k-f, h-g],G1), alist_gterm(Type,AL,G1).

Path

(Intermediate ⭐️⭐️) Path from one node to another one. Write a predicate path(G,A,B,P) to find an acyclic path P from node A to node B in the graph G. The predicate should return all paths via backtracking.

:- module(_, _, [assertions]). 

:- use_module(library(idlists), [memberchk/2]).
:- use_module(library(sort)).
:- use_module(library(aggregates), [findall/3]).
:- use_module(library(classic/classic_predicates), [delete/3]).

:- data edge/2.
:- data arc/2.

:- test path(G,A,B,P) : (G = graph([b,c,d,f,g,h,k],[e(b,c),e(b,f),e(c,f),e(f,k),e(g,h)]), A = b, B = k) => (P = [b,f,k]; P = [b,c,f,k]) + (not_fails, num_solutions(2)).

sorry :- throw(not_solved_yet). 

alist_gterm(Type,AL,GT):- nonvar(GT), !, gterm_to_alist(GT,Type,AL).
alist_gterm(Type,AL,GT):- atom(Type), nonvar(AL), alist_to_gterm(Type,AL,GT).

gterm_to_alist(graph(Ns,Es),graph,AL) :- memberchk(e(_,_,_),Es), ! ,
   lgt_al(Ns,Es,AL).
gterm_to_alist(graph(Ns,Es),graph,AL) :- !, 
   gt_al(Ns,Es,AL).
gterm_to_alist(digraph(Ns,As),digraph,AL) :- memberchk(a(_,_,_),As), !,
   ldt_al(Ns,As,AL).
gterm_to_alist(digraph(Ns,As),digraph,AL) :- 
   dt_al(Ns,As,AL).

lgt_al([],_,[]).
lgt_al([V|Vs],Es,[n(V,L)|Ns]) :-
   findall(T,((member(e(X,V,I),Es) ; member(e(V,X,I),Es)),T = X/I),L),
   lgt_al(Vs,Es,Ns).

gt_al([],_,[]).
gt_al([V|Vs],Es,[n(V,L)|Ns]) :-
   findall(X,(member(e(X,V),Es) ; member(e(V,X),Es)),L), gt_al(Vs,Es,Ns).

ldt_al([],_,[]).
ldt_al([V|Vs],As,[n(V,L)|Ns]) :-
   findall(T,(member(a(V,X,I),As), T=X/I),L), ldt_al(Vs,As,Ns).

dt_al([],_,[]).
dt_al([V|Vs],As,[n(V,L)|Ns]) :-
   findall(X,member(a(V,X),As),L), dt_al(Vs,As,Ns).

alist_to_gterm(graph,AL,graph(Ns,Es)) :- !, al_gt(AL,Ns,EsU,[]), sort(EsU,Es).
alist_to_gterm(digraph,AL,digraph(Ns,As)) :- al_dt(AL,Ns,AsU,[]), sort(AsU,As).

al_gt([],[],Es,Es).
al_gt([n(V,Xs)|Ns],[V|Vs],Es,Acc) :- 
   add_edges(V,Xs,Acc1,Acc), al_gt(Ns,Vs,Es,Acc1). 

add_edges(_,[],Es,Es).
add_edges(V,[X/_|Xs],Es,Acc) :- V @> X, !, add_edges(V,Xs,Es,Acc).
add_edges(V,[X|Xs],Es,Acc) :- V @> X, !, add_edges(V,Xs,Es,Acc).
add_edges(V,[X/I|Xs],Es,Acc) :- V @=< X, !, add_edges(V,Xs,Es,[e(V,X,I)|Acc]).
add_edges(V,[X|Xs],Es,Acc) :- V @=< X, add_edges(V,Xs,Es,[e(V,X)|Acc]).

al_dt([],[],As,As).
al_dt([n(V,Xs)|Ns],[V|Vs],As,Acc) :- 
   add_arcs(V,Xs,Acc1,Acc), al_dt(Ns,Vs,As,Acc1). 

add_arcs(_,[],As,As).
add_arcs(V,[X/I|Xs],As,Acc) :- !, add_arcs(V,Xs,As,[a(V,X,I)|Acc]).
add_arcs(V,[X|Xs],As,Acc) :- add_arcs(V,Xs,As,[a(V,X)|Acc]).

ecl_to_gterm(GT) :-
   findall(E,(edge(X,Y),E=X-Y),Es), human_gterm(Es,GT).

acl_to_gterm(GT) :-
   findall(A,(arc(X,Y),A= >(X,Y)),As), human_gterm(As,GT).
   
human_gterm(HF,GT):- nonvar(GT), !, gterm_to_human(GT,HF).
human_gterm(HF,GT):- nonvar(HF), human_to_gterm(HF,GT).

gterm_to_human(graph(Ns,Es),HF) :-  memberchk(e(_,_,_),Es), !, 
   lgt_hf(Ns,Es,HF).
gterm_to_human(graph(Ns,Es),HF) :-  !, 
   gt_hf(Ns,Es,HF).
gterm_to_human(digraph(Ns,As),HF) :- memberchk(a(_,_,_),As), !, 
   ldt_hf(Ns,As,HF).
gterm_to_human(digraph(Ns,As),HF) :- 
   dt_hf(Ns,As,HF).

lgt_hf(Ns,[],Ns).
lgt_hf(Ns,[e(X,Y,I)|Es],[X-Y/I|Hs]) :-
   delete(Ns,X,Ns1),
   delete(Ns1,Y,Ns2),
   lgt_hf(Ns2,Es,Hs).

gt_hf(Ns,[],Ns).
gt_hf(Ns,[e(X,Y)|Es],[X-Y|Hs]) :-
   delete(Ns,X,Ns1),
   delete(Ns1,Y,Ns2),
   gt_hf(Ns2,Es,Hs).

ldt_hf(Ns,[],Ns).
ldt_hf(Ns,[a(X,Y,I)|As],[X>Y/I|Hs]) :-
   delete(Ns,X,Ns1),
   delete(Ns1,Y,Ns2),
   ldt_hf(Ns2,As,Hs).

dt_hf(Ns,[],Ns).
dt_hf(Ns,[a(X,Y)|As],[X>Y|Hs]) :-
   delete(Ns,X,Ns1),
   delete(Ns1,Y,Ns2),
   dt_hf(Ns2,As,Hs).

human_to_gterm(HF,digraph(Ns,As)) :- memberchk(_>_,HF), !, 
   hf_dt(HF,Ns1,As1), sort(Ns1,Ns), sort(As1,As).
human_to_gterm(HF,graph(Ns,Es)) :- 
   hf_gt(HF,Ns1,Es1), sort(Ns1,Ns), sort(Es1,Es).
   
hf_gt([],[],[]).
hf_gt([X-Y/I|Hs],[X,Y|Ns],[e(U,V,I)|Es]) :- !, 
   sort0([X,Y],[U,V]), hf_gt(Hs,Ns,Es).
hf_gt([X-Y|Hs],[X,Y|Ns],[e(U,V)|Es]) :- !,
   sort0([X,Y],[U,V]), hf_gt(Hs,Ns,Es).
hf_gt([H|Hs],[H|Ns],Es) :- hf_gt(Hs,Ns,Es).

hf_dt([],[],[]).
hf_dt([X>Y/I|Hs],[X,Y|Ns],[a(X,Y,I)|As]) :- !, 
   hf_dt(Hs,Ns,As).
hf_dt([X>Y|Hs],[X,Y|Ns],[a(X,Y)|As]) :- !,
   hf_dt(Hs,Ns,As).
hf_dt([H|Hs],[H|Ns],As) :-  hf_dt(Hs,Ns,As).

sort0([X,Y],[X,Y]) :- X @=< Y, !.
sort0([X,Y],[Y,X]) :- X @> Y.
%! \begin{hint} 
path(G,A,B,P) :- sorry.
  % P is a (acyclic) path from node A to node B in the graph G.
  % G is given in graph-term form.
  % (+,+,+,?)

%! \end{hint} 
%! \begin{solution}
path(G,A,B,P) :- path1(G,A,[B],P).

path1(_,A,[A|P1],[A|P1]).
path1(G,A,[Y|P1],P) :- 
   adjacent(X,Y,G), \+ memberchk(X,[Y|P1]), path1(G,A,[X,Y|P1],P).

% A useful predicate: adjacent/3

adjacent(X,Y,graph(_,Es)) :- member(e(X,Y),Es).
adjacent(X,Y,graph(_,Es)) :- member(e(Y,X),Es).
adjacent(X,Y,graph(_,Es)) :- member(e(X,Y,_),Es).
adjacent(X,Y,graph(_,Es)) :- member(e(Y,X,_),Es).
adjacent(X,Y,digraph(_,As)) :- member(a(X,Y),As).
adjacent(X,Y,digraph(_,As)) :- member(a(X,Y,_),As).
%! \end{solution}
?- path(graph([b,c,d,f,g,h,k],[e(b,c),e(b,f),e(c,f),e(f,k),e(g,h)]),b,k,P).

Cycle

(Easy ⭐️) Cycle from a given node. Write a predicate cycle(G,A,P) to find a closed path (cycle) P starting at a given node A in the graph G. The predicate should return all cycles via backtracking.
:- module(_, _, [assertions]). 

:- use_module(library(idlists), [memberchk/2]).
:- use_module(library(sort)).
:- use_module(library(aggregates), [findall/3]).
:- use_module(library(classic/classic_predicates), [delete/3, append/3, length/2]).

:- data edge/2.
:- data arc/2.

:- test cycle(G,A,P) : (G = graph([b,c,d,f,g,h,k],[e(b,c),e(b,f),e(c,f),e(f,k),e(g,h)]), A = b) => (P = [b,f,c,b]; P = [b,c,f,b]) + (not_fails, num_solutions(2)).

sorry :- throw(not_solved_yet). 

alist_gterm(Type,AL,GT):- nonvar(GT), !, gterm_to_alist(GT,Type,AL).
alist_gterm(Type,AL,GT):- atom(Type), nonvar(AL), alist_to_gterm(Type,AL,GT).

gterm_to_alist(graph(Ns,Es),graph,AL) :- memberchk(e(_,_,_),Es), ! ,
   lgt_al(Ns,Es,AL).
gterm_to_alist(graph(Ns,Es),graph,AL) :- !, 
   gt_al(Ns,Es,AL).
gterm_to_alist(digraph(Ns,As),digraph,AL) :- memberchk(a(_,_,_),As), !,
   ldt_al(Ns,As,AL).
gterm_to_alist(digraph(Ns,As),digraph,AL) :- 
   dt_al(Ns,As,AL).

lgt_al([],_,[]).
lgt_al([V|Vs],Es,[n(V,L)|Ns]) :-
   findall(T,((member(e(X,V,I),Es) ; member(e(V,X,I),Es)),T = X/I),L),
   lgt_al(Vs,Es,Ns).

gt_al([],_,[]).
gt_al([V|Vs],Es,[n(V,L)|Ns]) :-
   findall(X,(member(e(X,V),Es) ; member(e(V,X),Es)),L), gt_al(Vs,Es,Ns).

ldt_al([],_,[]).
ldt_al([V|Vs],As,[n(V,L)|Ns]) :-
   findall(T,(member(a(V,X,I),As), T=X/I),L), ldt_al(Vs,As,Ns).

dt_al([],_,[]).
dt_al([V|Vs],As,[n(V,L)|Ns]) :-
   findall(X,member(a(V,X),As),L), dt_al(Vs,As,Ns).

alist_to_gterm(graph,AL,graph(Ns,Es)) :- !, al_gt(AL,Ns,EsU,[]), sort(EsU,Es).
alist_to_gterm(digraph,AL,digraph(Ns,As)) :- al_dt(AL,Ns,AsU,[]), sort(AsU,As).

al_gt([],[],Es,Es).
al_gt([n(V,Xs)|Ns],[V|Vs],Es,Acc) :- 
   add_edges(V,Xs,Acc1,Acc), al_gt(Ns,Vs,Es,Acc1). 

add_edges(_,[],Es,Es).
add_edges(V,[X/_|Xs],Es,Acc) :- V @> X, !, add_edges(V,Xs,Es,Acc).
add_edges(V,[X|Xs],Es,Acc) :- V @> X, !, add_edges(V,Xs,Es,Acc).
add_edges(V,[X/I|Xs],Es,Acc) :- V @=< X, !, add_edges(V,Xs,Es,[e(V,X,I)|Acc]).
add_edges(V,[X|Xs],Es,Acc) :- V @=< X, add_edges(V,Xs,Es,[e(V,X)|Acc]).

al_dt([],[],As,As).
al_dt([n(V,Xs)|Ns],[V|Vs],As,Acc) :- 
   add_arcs(V,Xs,Acc1,Acc), al_dt(Ns,Vs,As,Acc1). 

add_arcs(_,[],As,As).
add_arcs(V,[X/I|Xs],As,Acc) :- !, add_arcs(V,Xs,As,[a(V,X,I)|Acc]).
add_arcs(V,[X|Xs],As,Acc) :- add_arcs(V,Xs,As,[a(V,X)|Acc]).

ecl_to_gterm(GT) :-
   findall(E,(edge(X,Y),E=X-Y),Es), human_gterm(Es,GT).

acl_to_gterm(GT) :-
   findall(A,(arc(X,Y),A= >(X,Y)),As), human_gterm(As,GT).
   
human_gterm(HF,GT):- nonvar(GT), !, gterm_to_human(GT,HF).
human_gterm(HF,GT):- nonvar(HF), human_to_gterm(HF,GT).

gterm_to_human(graph(Ns,Es),HF) :-  memberchk(e(_,_,_),Es), !, 
   lgt_hf(Ns,Es,HF).
gterm_to_human(graph(Ns,Es),HF) :-  !, 
   gt_hf(Ns,Es,HF).
gterm_to_human(digraph(Ns,As),HF) :- memberchk(a(_,_,_),As), !, 
   ldt_hf(Ns,As,HF).
gterm_to_human(digraph(Ns,As),HF) :- 
   dt_hf(Ns,As,HF).

lgt_hf(Ns,[],Ns).
lgt_hf(Ns,[e(X,Y,I)|Es],[X-Y/I|Hs]) :-
   delete(Ns,X,Ns1),
   delete(Ns1,Y,Ns2),
   lgt_hf(Ns2,Es,Hs).

gt_hf(Ns,[],Ns).
gt_hf(Ns,[e(X,Y)|Es],[X-Y|Hs]) :-
   delete(Ns,X,Ns1),
   delete(Ns1,Y,Ns2),
   gt_hf(Ns2,Es,Hs).

ldt_hf(Ns,[],Ns).
ldt_hf(Ns,[a(X,Y,I)|As],[X>Y/I|Hs]) :-
   delete(Ns,X,Ns1),
   delete(Ns1,Y,Ns2),
   ldt_hf(Ns2,As,Hs).

dt_hf(Ns,[],Ns).
dt_hf(Ns,[a(X,Y)|As],[X>Y|Hs]) :-
   delete(Ns,X,Ns1),
   delete(Ns1,Y,Ns2),
   dt_hf(Ns2,As,Hs).

human_to_gterm(HF,digraph(Ns,As)) :- memberchk(_>_,HF), !, 
   hf_dt(HF,Ns1,As1), sort(Ns1,Ns), sort(As1,As).
human_to_gterm(HF,graph(Ns,Es)) :- 
   hf_gt(HF,Ns1,Es1), sort(Ns1,Ns), sort(Es1,Es).
   
hf_gt([],[],[]).
hf_gt([X-Y/I|Hs],[X,Y|Ns],[e(U,V,I)|Es]) :- !, 
   sort0([X,Y],[U,V]), hf_gt(Hs,Ns,Es).
hf_gt([X-Y|Hs],[X,Y|Ns],[e(U,V)|Es]) :- !,
   sort0([X,Y],[U,V]), hf_gt(Hs,Ns,Es).
hf_gt([H|Hs],[H|Ns],Es) :- hf_gt(Hs,Ns,Es).

hf_dt([],[],[]).
hf_dt([X>Y/I|Hs],[X,Y|Ns],[a(X,Y,I)|As]) :- !, 
   hf_dt(Hs,Ns,As).
hf_dt([X>Y|Hs],[X,Y|Ns],[a(X,Y)|As]) :- !,
   hf_dt(Hs,Ns,As).
hf_dt([H|Hs],[H|Ns],As) :-  hf_dt(Hs,Ns,As).

sort0([X,Y],[X,Y]) :- X @=< Y, !.
sort0([X,Y],[Y,X]) :- X @> Y.

path(G,A,B,P) :- path1(G,A,[B],P).

path1(_,A,[A|P1],[A|P1]).
path1(G,A,[Y|P1],P) :- 
   adjacent(X,Y,G), \+ memberchk(X,[Y|P1]), path1(G,A,[X,Y|P1],P).

adjacent(X,Y,graph(_,Es)) :- member(e(X,Y),Es).
adjacent(X,Y,graph(_,Es)) :- member(e(Y,X),Es).
adjacent(X,Y,graph(_,Es)) :- member(e(X,Y,_),Es).
adjacent(X,Y,graph(_,Es)) :- member(e(Y,X,_),Es).
adjacent(X,Y,digraph(_,As)) :- member(a(X,Y),As).
adjacent(X,Y,digraph(_,As)) :- member(a(X,Y,_),As).
%! \begin{hint} 
cycle(G,A,P) :- sorry.
  % P is a closed path starting at node A in the graph G.
  % G is given in graph-term form.
  % (+,+,?)
%! \end{hint} 
%! \begin{solution}
cycle(G,A,P) :- 
   adjacent(B,A,G), path(G,A,B,P1), length(P1,L), L > 2, append(P1,[A],P).
%! \end{solution}
?- cycle(graph([b,c,d,f,g,h,k],[e(b,c),e(b,f),e(c,f),e(f,k),e(g,h)]),b,P).

Spanning trees

(Intermediate ⭐️⭐️) Construct all spanning trees. Write a predicate s_tree(Graph,Tree) to construct (by backtracking) all spanning trees of a given graph. With this predicate, find out how many spanning trees there are for the graph depicted to the left. When you have a correct solution for the s_tree/2 predicate, use it to define two other useful predicates: is_tree(Graph) and is_connected(Graph). Both are five-minute tasks! This is an example graph:

:- module(_, _, [assertions]). 

:- use_module(library(idlists), [memberchk/2]).
:- use_module(library(sort)).
:- use_module(library(aggregates), [findall/3]).
:- use_module(library(classic/classic_predicates), [delete/3, select/3]).

:- data edge/2.
:- data arc/2.

sorry :- throw(not_solved_yet). 

:- test is_connected(A) : (A = graph([a,b,c,d,e,f,g,h],[e(a,b),e(a,d),e(b,c),e(b,e),e(c,e),e(e,d),e(d,f),e(e,h),e(d,g),e(f,g), e(g,h)])) + (not_fails).

:- test is_tree(A) : (A = graph([a,b,c,d,e,f,g,h],[e(a,b),e(a,d),e(b,c),e(b,e),e(c,e),e(e,d),e(d,f),e(e,h),e(d,g),e(f,g), e(g,h)])) + (fails).

alist_gterm(Type,AL,GT):- nonvar(GT), !, gterm_to_alist(GT,Type,AL).
alist_gterm(Type,AL,GT):- atom(Type), nonvar(AL), alist_to_gterm(Type,AL,GT).

gterm_to_alist(graph(Ns,Es),graph,AL) :- memberchk(e(_,_,_),Es), ! ,
   lgt_al(Ns,Es,AL).
gterm_to_alist(graph(Ns,Es),graph,AL) :- !, 
   gt_al(Ns,Es,AL).
gterm_to_alist(digraph(Ns,As),digraph,AL) :- memberchk(a(_,_,_),As), !,
   ldt_al(Ns,As,AL).
gterm_to_alist(digraph(Ns,As),digraph,AL) :- 
   dt_al(Ns,As,AL).

lgt_al([],_,[]).
lgt_al([V|Vs],Es,[n(V,L)|Ns]) :-
   findall(T,((member(e(X,V,I),Es) ; member(e(V,X,I),Es)),T = X/I),L),
   lgt_al(Vs,Es,Ns).

gt_al([],_,[]).
gt_al([V|Vs],Es,[n(V,L)|Ns]) :-
   findall(X,(member(e(X,V),Es) ; member(e(V,X),Es)),L), gt_al(Vs,Es,Ns).

ldt_al([],_,[]).
ldt_al([V|Vs],As,[n(V,L)|Ns]) :-
   findall(T,(member(a(V,X,I),As), T=X/I),L), ldt_al(Vs,As,Ns).

dt_al([],_,[]).
dt_al([V|Vs],As,[n(V,L)|Ns]) :-
   findall(X,member(a(V,X),As),L), dt_al(Vs,As,Ns).

alist_to_gterm(graph,AL,graph(Ns,Es)) :- !, al_gt(AL,Ns,EsU,[]), sort(EsU,Es).
alist_to_gterm(digraph,AL,digraph(Ns,As)) :- al_dt(AL,Ns,AsU,[]), sort(AsU,As).

al_gt([],[],Es,Es).
al_gt([n(V,Xs)|Ns],[V|Vs],Es,Acc) :- 
   add_edges(V,Xs,Acc1,Acc), al_gt(Ns,Vs,Es,Acc1). 

add_edges(_,[],Es,Es).
add_edges(V,[X/_|Xs],Es,Acc) :- V @> X, !, add_edges(V,Xs,Es,Acc).
add_edges(V,[X|Xs],Es,Acc) :- V @> X, !, add_edges(V,Xs,Es,Acc).
add_edges(V,[X/I|Xs],Es,Acc) :- V @=< X, !, add_edges(V,Xs,Es,[e(V,X,I)|Acc]).
add_edges(V,[X|Xs],Es,Acc) :- V @=< X, add_edges(V,Xs,Es,[e(V,X)|Acc]).

al_dt([],[],As,As).
al_dt([n(V,Xs)|Ns],[V|Vs],As,Acc) :- 
   add_arcs(V,Xs,Acc1,Acc), al_dt(Ns,Vs,As,Acc1). 

add_arcs(_,[],As,As).
add_arcs(V,[X/I|Xs],As,Acc) :- !, add_arcs(V,Xs,As,[a(V,X,I)|Acc]).
add_arcs(V,[X|Xs],As,Acc) :- add_arcs(V,Xs,As,[a(V,X)|Acc]).

ecl_to_gterm(GT) :-
   findall(E,(edge(X,Y),E=X-Y),Es), human_gterm(Es,GT).

acl_to_gterm(GT) :-
   findall(A,(arc(X,Y),A= >(X,Y)),As), human_gterm(As,GT).
   
human_gterm(HF,GT):- nonvar(GT), !, gterm_to_human(GT,HF).
human_gterm(HF,GT):- nonvar(HF), human_to_gterm(HF,GT).

gterm_to_human(graph(Ns,Es),HF) :-  memberchk(e(_,_,_),Es), !, 
   lgt_hf(Ns,Es,HF).
gterm_to_human(graph(Ns,Es),HF) :-  !, 
   gt_hf(Ns,Es,HF).
gterm_to_human(digraph(Ns,As),HF) :- memberchk(a(_,_,_),As), !, 
   ldt_hf(Ns,As,HF).
gterm_to_human(digraph(Ns,As),HF) :- 
   dt_hf(Ns,As,HF).

lgt_hf(Ns,[],Ns).
lgt_hf(Ns,[e(X,Y,I)|Es],[X-Y/I|Hs]) :-
   delete(Ns,X,Ns1),
   delete(Ns1,Y,Ns2),
   lgt_hf(Ns2,Es,Hs).

gt_hf(Ns,[],Ns).
gt_hf(Ns,[e(X,Y)|Es],[X-Y|Hs]) :-
   delete(Ns,X,Ns1),
   delete(Ns1,Y,Ns2),
   gt_hf(Ns2,Es,Hs).

ldt_hf(Ns,[],Ns).
ldt_hf(Ns,[a(X,Y,I)|As],[X>Y/I|Hs]) :-
   delete(Ns,X,Ns1),
   delete(Ns1,Y,Ns2),
   ldt_hf(Ns2,As,Hs).

dt_hf(Ns,[],Ns).
dt_hf(Ns,[a(X,Y)|As],[X>Y|Hs]) :-
   delete(Ns,X,Ns1),
   delete(Ns1,Y,Ns2),
   dt_hf(Ns2,As,Hs).

human_to_gterm(HF,digraph(Ns,As)) :- memberchk(_>_,HF), !, 
   hf_dt(HF,Ns1,As1), sort(Ns1,Ns), sort(As1,As).
human_to_gterm(HF,graph(Ns,Es)) :- 
   hf_gt(HF,Ns1,Es1), sort(Ns1,Ns), sort(Es1,Es).
   
hf_gt([],[],[]).
hf_gt([X-Y/I|Hs],[X,Y|Ns],[e(U,V,I)|Es]) :- !, 
   sort0([X,Y],[U,V]), hf_gt(Hs,Ns,Es).
hf_gt([X-Y|Hs],[X,Y|Ns],[e(U,V)|Es]) :- !,
   sort0([X,Y],[U,V]), hf_gt(Hs,Ns,Es).
hf_gt([H|Hs],[H|Ns],Es) :- hf_gt(Hs,Ns,Es).

hf_dt([],[],[]).
hf_dt([X>Y/I|Hs],[X,Y|Ns],[a(X,Y,I)|As]) :- !, 
   hf_dt(Hs,Ns,As).
hf_dt([X>Y|Hs],[X,Y|Ns],[a(X,Y)|As]) :- !,
   hf_dt(Hs,Ns,As).
hf_dt([H|Hs],[H|Ns],As) :-  hf_dt(Hs,Ns,As).

sort0([X,Y],[X,Y]) :- X @=< Y, !.
sort0([X,Y],[Y,X]) :- X @> Y.
%! \begin{hint} 
s_tree(G,T) :- sorry.
  % T is a spanning tree of the graph G
  % (graph-term graph-term) (+,?)
  
transfer(Ns,GEs,TEs) :- sorry.
  % transfer edges from GEs (graph edges)
  % to TEs (tree edges) until the list NS of still unconnected tree nodes
  % becomes empty. An edge is accepted if and only if one end-point is 
  % already connected to the tree and the other is not.

% Another use is the following connectivity tester:

is_tree(G) :- sorry.
  % the graph G is a tree

is_connected(G) :- sorry.
  % the graph G is connected
%! \end{hint} 
%! \begin{solution}
s_tree(graph([N|Ns],GraphEdges),graph([N|Ns],TreeEdges)) :- 
   transfer(Ns,GraphEdges,TreeEdgesUnsorted),
   sort(TreeEdgesUnsorted,TreeEdges).

transfer([],_,[]).
transfer(Ns,GEs,[GE|TEs]) :- 
   select(GE,GEs,GEs1),        
   incident(GE,X,Y),
   acceptable(X,Y,Ns),
   delete(Ns,X,Ns1),
   delete(Ns1,Y,Ns2),
   transfer(Ns2,GEs1,TEs).

incident(e(X,Y),X,Y).
incident(e(X,Y,_),X,Y).

acceptable(X,Y,Ns) :- memberchk(X,Ns), \+ memberchk(Y,Ns), !.
acceptable(X,Y,Ns) :- memberchk(Y,Ns), \+ memberchk(X,Ns).

% An almost trivial use of the predicate s_tree/2 is the following
% tree tester predicate:
 
% is_tree(G) :- the graph G is a tree
is_tree(G) :- s_tree(G,G), !.

% is_connected(G) :- the graph G is connected
is_connected(G) :- s_tree(G,_), !.

%! \end{solution}
?- is_connected(graph([a,b,c,d,e,f,g,h],[e(a,b),e(a,d),e(b,c),e(b,e),e(c,e),e(e,d),e(d,f),e(e,h),e(d,g),e(f,g), e(g,h)])).
?- is_tree(graph([a,b,c,d,e,f,g,h],[e(a,b),e(a,d),e(b,c),e(b,e),e(c,e),e(d,e),e(d,f),e(d,g),e(e,h),e(f,g),e(g,h)])).

Node degree and graph coloration

(Intermediate ⭐️⭐️) Node degree and graph coloration.

a) Write a predicate degree(Graph,Node,Deg) that determines the degree of a given node.

b) Write a predicate that generates a list of all nodes of a graph sorted according to decreasing degree.

c) Use Welch-Powell's algorithm to paint the nodes of a graph in such a way that adjacent nodes have different colors.

:- module(_, _, [assertions]). 

:- push_prolog_flag(multi_arity_warnings, off).

:- use_module(library(idlists), [memberchk/2]).
:- use_module(library(sort)).
:- use_module(library(aggregates), [findall/3]).
:- use_module(library(classic/classic_predicates), [delete/3, length/2]).

:- data edge/2.
:- data arc/2.

sorry :- throw(not_solved_yet). 

alist_gterm(Type,AL,GT):- nonvar(GT), !, gterm_to_alist(GT,Type,AL).
alist_gterm(Type,AL,GT):- atom(Type), nonvar(AL), alist_to_gterm(Type,AL,GT).

gterm_to_alist(graph(Ns,Es),graph,AL) :- memberchk(e(_,_,_),Es), ! ,
   lgt_al(Ns,Es,AL).
gterm_to_alist(graph(Ns,Es),graph,AL) :- !, 
   gt_al(Ns,Es,AL).
gterm_to_alist(digraph(Ns,As),digraph,AL) :- memberchk(a(_,_,_),As), !,
   ldt_al(Ns,As,AL).
gterm_to_alist(digraph(Ns,As),digraph,AL) :- 
   dt_al(Ns,As,AL).

lgt_al([],_,[]).
lgt_al([V|Vs],Es,[n(V,L)|Ns]) :-
   findall(T,((member(e(X,V,I),Es) ; member(e(V,X,I),Es)),T = X/I),L),
   lgt_al(Vs,Es,Ns).

gt_al([],_,[]).
gt_al([V|Vs],Es,[n(V,L)|Ns]) :-
   findall(X,(member(e(X,V),Es) ; member(e(V,X),Es)),L), gt_al(Vs,Es,Ns).

ldt_al([],_,[]).
ldt_al([V|Vs],As,[n(V,L)|Ns]) :-
   findall(T,(member(a(V,X,I),As), T=X/I),L), ldt_al(Vs,As,Ns).

dt_al([],_,[]).
dt_al([V|Vs],As,[n(V,L)|Ns]) :-
   findall(X,member(a(V,X),As),L), dt_al(Vs,As,Ns).

alist_to_gterm(graph,AL,graph(Ns,Es)) :- !, al_gt(AL,Ns,EsU,[]), sort(EsU,Es).
alist_to_gterm(digraph,AL,digraph(Ns,As)) :- al_dt(AL,Ns,AsU,[]), sort(AsU,As).

al_gt([],[],Es,Es).
al_gt([n(V,Xs)|Ns],[V|Vs],Es,Acc) :- 
   add_edges(V,Xs,Acc1,Acc), al_gt(Ns,Vs,Es,Acc1). 

add_edges(_,[],Es,Es).
add_edges(V,[X/_|Xs],Es,Acc) :- V @> X, !, add_edges(V,Xs,Es,Acc).
add_edges(V,[X|Xs],Es,Acc) :- V @> X, !, add_edges(V,Xs,Es,Acc).
add_edges(V,[X/I|Xs],Es,Acc) :- V @=< X, !, add_edges(V,Xs,Es,[e(V,X,I)|Acc]).
add_edges(V,[X|Xs],Es,Acc) :- V @=< X, add_edges(V,Xs,Es,[e(V,X)|Acc]).

al_dt([],[],As,As).
al_dt([n(V,Xs)|Ns],[V|Vs],As,Acc) :- 
   add_arcs(V,Xs,Acc1,Acc), al_dt(Ns,Vs,As,Acc1). 

add_arcs(_,[],As,As).
add_arcs(V,[X/I|Xs],As,Acc) :- !, add_arcs(V,Xs,As,[a(V,X,I)|Acc]).
add_arcs(V,[X|Xs],As,Acc) :- add_arcs(V,Xs,As,[a(V,X)|Acc]).

ecl_to_gterm(GT) :-
   findall(E,(edge(X,Y),E=X-Y),Es), human_gterm(Es,GT).

acl_to_gterm(GT) :-
   findall(A,(arc(X,Y),A= >(X,Y)),As), human_gterm(As,GT).
   
human_gterm(HF,GT):- nonvar(GT), !, gterm_to_human(GT,HF).
human_gterm(HF,GT):- nonvar(HF), human_to_gterm(HF,GT).

gterm_to_human(graph(Ns,Es),HF) :-  memberchk(e(_,_,_),Es), !, 
   lgt_hf(Ns,Es,HF).
gterm_to_human(graph(Ns,Es),HF) :-  !, 
   gt_hf(Ns,Es,HF).
gterm_to_human(digraph(Ns,As),HF) :- memberchk(a(_,_,_),As), !, 
   ldt_hf(Ns,As,HF).
gterm_to_human(digraph(Ns,As),HF) :- 
   dt_hf(Ns,As,HF).

lgt_hf(Ns,[],Ns).
lgt_hf(Ns,[e(X,Y,I)|Es],[X-Y/I|Hs]) :-
   delete(Ns,X,Ns1),
   delete(Ns1,Y,Ns2),
   lgt_hf(Ns2,Es,Hs).

gt_hf(Ns,[],Ns).
gt_hf(Ns,[e(X,Y)|Es],[X-Y|Hs]) :-
   delete(Ns,X,Ns1),
   delete(Ns1,Y,Ns2),
   gt_hf(Ns2,Es,Hs).

ldt_hf(Ns,[],Ns).
ldt_hf(Ns,[a(X,Y,I)|As],[X>Y/I|Hs]) :-
   delete(Ns,X,Ns1),
   delete(Ns1,Y,Ns2),
   ldt_hf(Ns2,As,Hs).

dt_hf(Ns,[],Ns).
dt_hf(Ns,[a(X,Y)|As],[X>Y|Hs]) :-
   delete(Ns,X,Ns1),
   delete(Ns1,Y,Ns2),
   dt_hf(Ns2,As,Hs).

human_to_gterm(HF,digraph(Ns,As)) :- memberchk(_>_,HF), !, 
   hf_dt(HF,Ns1,As1), sort(Ns1,Ns), sort(As1,As).
human_to_gterm(HF,graph(Ns,Es)) :- 
   hf_gt(HF,Ns1,Es1), sort(Ns1,Ns), sort(Es1,Es).
   
hf_gt([],[],[]).
hf_gt([X-Y/I|Hs],[X,Y|Ns],[e(U,V,I)|Es]) :- !, 
   sort0([X,Y],[U,V]), hf_gt(Hs,Ns,Es).
hf_gt([X-Y|Hs],[X,Y|Ns],[e(U,V)|Es]) :- !,
   sort0([X,Y],[U,V]), hf_gt(Hs,Ns,Es).
hf_gt([H|Hs],[H|Ns],Es) :- hf_gt(Hs,Ns,Es).

hf_dt([],[],[]).
hf_dt([X>Y/I|Hs],[X,Y|Ns],[a(X,Y,I)|As]) :- !, 
   hf_dt(Hs,Ns,As).
hf_dt([X>Y|Hs],[X,Y|Ns],[a(X,Y)|As]) :- !,
   hf_dt(Hs,Ns,As).
hf_dt([H|Hs],[H|Ns],As) :-  hf_dt(Hs,Ns,As).

sort0([X,Y],[X,Y]) :- X @=< Y, !.
sort0([X,Y],[Y,X]) :- X @> Y.

path(G,A,B,P) :- path1(G,A,[B],P).

path1(_,A,[A|P1],[A|P1]).
path1(G,A,[Y|P1],P) :- 
   adjacent(X,Y,G), \+ memberchk(X,[Y|P1]), path1(G,A,[X,Y|P1],P).

adjacent(X,Y,graph(_,Es)) :- member(e(X,Y),Es).
adjacent(X,Y,graph(_,Es)) :- member(e(Y,X),Es).
adjacent(X,Y,graph(_,Es)) :- member(e(X,Y,_),Es).
adjacent(X,Y,graph(_,Es)) :- member(e(Y,X,_),Es).
adjacent(X,Y,digraph(_,As)) :- member(a(X,Y),As).
adjacent(X,Y,digraph(_,As)) :- member(a(X,Y,_),As).
%! \begin{hint} 
% a) Write a predicate degree(Graph,Node,Deg) that determines the degree 
% of a given node. 

degree(Graph,Node,Deg) :- sorry.
  % Deg is the degree of the node Node in the graph Graph.
  % (graph-term, node, integer), (+,+,?).

% --------------------------------------------------------------------------

% b) Write a predicate that generates a list of all nodes of a graph 
% sorted according to decreasing degree.

degree_sorted_nodes(Graph,Nodes) :- sorry.
  % Nodes is the list of the nodes
  % of the graph Graph, sorted according to decreasing degree.
  
% --------------------------------------------------------------------------

% c) Use Welch-Powell's algorithm to paint the nodes of a graph in such 
% a way that adjacent nodes have different colors.

% Use Welch-Powell's algorithm to paint the nodes of a graph
% in such a way that adjacent nodes have different colors.

paint(Graph,ColoredNodes) :- sorry.

paint_nodes(Graph,Ns,AccNodes,Color,ColoNodes) :- sorry. 
  % paint the remaining nodes Ns with a color number Color or higher. 
  % AccNodes is the set of nodes already colored. Return the result in ColoNodes.
  % (graph-term,node-list,c-node-list,integer,c-node-list)
  % (+,+,+,+,-) 
   
paint_nodes(Graph,DSNs,Ns,AccNodes,Color,ColoNodes) :- sorry.
  % paint the nodes in Ns with a fixed color number Color, if possible.
  % If Ns is empty, continue with the next color number.
  % AccNodes is the set of nodes already colored. Return the result in ColoNodes.
  % (graph-term,node-list,c-node-list,c-node-list,integer,c-node-list)
  % (+,+,+,+,+,-) 
%! \end{hint} 
%! \begin{solution}
% a) Write a predicate degree(Graph,Node,Deg) that determines the degree 
% of a given node. 

degree(graph(Ns,Es),Node,Deg) :- 
   alist_gterm(graph,AList,graph(Ns,Es)),
   member(n(Node,AdjList),AList), !,
   length(AdjList,Deg).

% --------------------------------------------------------------------------

% b) Write a predicate that generates a list of all nodes of a graph 
% sorted according to decreasing degree.

% degree_sorted_nodes(Graph,Nodes) :- Nodes is the list of the nodes
%    of the graph Graph, sorted according to decreasing degree.

degree_sorted_nodes(graph(Ns,Es),DSNodes) :- 
   alist_gterm(graph,AList,graph(Ns,Es)),  
   predsort(compare_degree,AList,AListDegreeSorted),
   reduce(AListDegreeSorted,DSNodes).

compare_degree(Order,n(N1,AL1),n(N2,AL2)) :-
   length(AL1,D1), length(AL2,D2),
   compare(Order,D2+N1,D1+N2).

% Note: compare(Order,D2+N1,D1+N2) sorts the nodes according to 
% decreasing degree, but alphabetically if the degrees are equal. Cool!

reduce([],[]).
reduce([n(N,_)|Ns],[N|NsR]) :- reduce(Ns,NsR).

% --------------------------------------------------------------------------

% c) Use Welch-Powell's algorithm to paint the nodes of a graph in such 
% a way that adjacent nodes have different colors.

% Use Welch-Powell's algorithm to paint the nodes of a graph
% in such a way that adjacent nodes have different colors.

paint(Graph,ColoredNodes) :-
   degree_sorted_nodes(Graph,DSNs),
   paint_nodes(Graph,DSNs,[],1,ColoredNodes).

% paint_nodes(Graph,Ns,AccNodes,Color,ColoNodes) :- paint the remaining
%    nodes Ns with a color number Color or higher. AccNodes is the set
%    of nodes already colored. Return the result in ColoNodes.
%    (graph-term,node-list,c-node-list,integer,c-node-list)
%    (+,+,+,+,-) 
paint_nodes(_,[],ColoNodes,_,ColoNodes) :- !.
paint_nodes(Graph,Ns,AccNodes,Color,ColoNodes) :-
   paint_nodes(Graph,Ns,Ns,AccNodes,Color,ColoNodes).
   
% paint_nodes(Graph,DSNs,Ns,AccNodes,Color,ColoNodes) :- paint the
%    nodes in Ns with a fixed color number Color, if possible.
%    If Ns is empty, continue with the next color number.
%    AccNodes is the set of nodes already colored. 
%    Return the result in ColoNodes.
%    (graph-term,node-list,c-node-list,c-node-list,integer,c-node-list)
%    (+,+,+,+,+,-) 
paint_nodes(Graph,Ns,[],AccNodes,Color,ColoNodes) :- !,
   Color1 is Color+1,
   paint_nodes(Graph,Ns,AccNodes,Color1,ColoNodes).
paint_nodes(Graph,DSNs,[N|Ns],AccNodes,Color,ColoNodes) :- 
   \+ has_neighbor(Graph,N,Color,AccNodes), !,
   delete(DSNs,N,DSNs1),
   paint_nodes(Graph,DSNs1,Ns,[c(N,Color)|AccNodes],Color,ColoNodes).
paint_nodes(Graph,DSNs,[_|Ns],AccNodes,Color,ColoNodes) :- 
   paint_nodes(Graph,DSNs,Ns,AccNodes,Color,ColoNodes).
   
has_neighbor(Graph,N,Color,AccNodes) :- 
   adjacent(N,X,Graph),
   memberchk(c(X,Color),AccNodes).
%! \end{solution}
?- degree(graph([a,b,c,d,e,f,g,h],[e(a,b,5),e(a,d,3),e(b,c,2),e(b,e,4),e(c,e,6),e(e,d,7),e(d,f,4),e(e,h,3),e(d,g,5),e(f,g,4),e(g,h,1)]),b,A).

(Intermediate ⭐️⭐️) Depth-first order graph traversal. Write a predicate that generates a depth-first order graph traversal sequence. The starting point should be specified, and the output should be a list of nodes that are reachable from this starting point (in depth-first order).
:- module(_, _, [assertions, datafacts]). 

:- push_prolog_flag(multi_arity_warnings, off).

:- use_module(library(idlists), [memberchk/2]).
:- use_module(library(sort)).
:- use_module(library(aggregates), [findall/3, bagof/3]).
:- use_module(library(classic/classic_predicates), [delete/3, recorded/3, recorda/3, recordz/3]).

:- data edge/2.
:- data arc/2.

:- test depth_first_order(A,B,C) : (A = graph([a,b,c,d,e,f,g,h],[e(a,e),e(a,b),e(a,d),e(b,c),e(b,e),e(c,e),e(e,d),e(d,f),e(g,h)]), B = h) => (C = [g]; C = [h]) + (not_fails, num_solutions(2)).
sorry :- throw(not_solved_yet). 

:- test depth_first_order(A,B,C) : (A = graph([h],[e(h,h)]), B = h) => (C = [h]) + (not_fails, is_det).
sorry :- throw(not_solved_yet). 

alist_gterm(Type,AL,GT):- nonvar(GT), !, gterm_to_alist(GT,Type,AL).
alist_gterm(Type,AL,GT):- atom(Type), nonvar(AL), alist_to_gterm(Type,AL,GT).

gterm_to_alist(graph(Ns,Es),graph,AL) :- memberchk(e(_,_,_),Es), ! ,
   lgt_al(Ns,Es,AL).
gterm_to_alist(graph(Ns,Es),graph,AL) :- !, 
   gt_al(Ns,Es,AL).
gterm_to_alist(digraph(Ns,As),digraph,AL) :- memberchk(a(_,_,_),As), !,
   ldt_al(Ns,As,AL).
gterm_to_alist(digraph(Ns,As),digraph,AL) :- 
   dt_al(Ns,As,AL).

lgt_al([],_,[]).
lgt_al([V|Vs],Es,[n(V,L)|Ns]) :-
   findall(T,((member(e(X,V,I),Es) ; member(e(V,X,I),Es)),T = X/I),L),
   lgt_al(Vs,Es,Ns).

gt_al([],_,[]).
gt_al([V|Vs],Es,[n(V,L)|Ns]) :-
   findall(X,(member(e(X,V),Es) ; member(e(V,X),Es)),L), gt_al(Vs,Es,Ns).

ldt_al([],_,[]).
ldt_al([V|Vs],As,[n(V,L)|Ns]) :-
   findall(T,(member(a(V,X,I),As), T=X/I),L), ldt_al(Vs,As,Ns).

dt_al([],_,[]).
dt_al([V|Vs],As,[n(V,L)|Ns]) :-
   findall(X,member(a(V,X),As),L), dt_al(Vs,As,Ns).

alist_to_gterm(graph,AL,graph(Ns,Es)) :- !, al_gt(AL,Ns,EsU,[]), sort(EsU,Es).
alist_to_gterm(digraph,AL,digraph(Ns,As)) :- al_dt(AL,Ns,AsU,[]), sort(AsU,As).

al_gt([],[],Es,Es).
al_gt([n(V,Xs)|Ns],[V|Vs],Es,Acc) :- 
   add_edges(V,Xs,Acc1,Acc), al_gt(Ns,Vs,Es,Acc1). 

add_edges(_,[],Es,Es).
add_edges(V,[X/_|Xs],Es,Acc) :- V @> X, !, add_edges(V,Xs,Es,Acc).
add_edges(V,[X|Xs],Es,Acc) :- V @> X, !, add_edges(V,Xs,Es,Acc).
add_edges(V,[X/I|Xs],Es,Acc) :- V @=< X, !, add_edges(V,Xs,Es,[e(V,X,I)|Acc]).
add_edges(V,[X|Xs],Es,Acc) :- V @=< X, add_edges(V,Xs,Es,[e(V,X)|Acc]).

al_dt([],[],As,As).
al_dt([n(V,Xs)|Ns],[V|Vs],As,Acc) :- 
   add_arcs(V,Xs,Acc1,Acc), al_dt(Ns,Vs,As,Acc1). 

add_arcs(_,[],As,As).
add_arcs(V,[X/I|Xs],As,Acc) :- !, add_arcs(V,Xs,As,[a(V,X,I)|Acc]).
add_arcs(V,[X|Xs],As,Acc) :- add_arcs(V,Xs,As,[a(V,X)|Acc]).

ecl_to_gterm(GT) :-
   findall(E,(edge(X,Y),E=X-Y),Es), human_gterm(Es,GT).

acl_to_gterm(GT) :-
   findall(A,(arc(X,Y),A= >(X,Y)),As), human_gterm(As,GT).
   
human_gterm(HF,GT):- nonvar(GT), !, gterm_to_human(GT,HF).
human_gterm(HF,GT):- nonvar(HF), human_to_gterm(HF,GT).

gterm_to_human(graph(Ns,Es),HF) :-  memberchk(e(_,_,_),Es), !, 
   lgt_hf(Ns,Es,HF).
gterm_to_human(graph(Ns,Es),HF) :-  !, 
   gt_hf(Ns,Es,HF).
gterm_to_human(digraph(Ns,As),HF) :- memberchk(a(_,_,_),As), !, 
   ldt_hf(Ns,As,HF).
gterm_to_human(digraph(Ns,As),HF) :- 
   dt_hf(Ns,As,HF).

lgt_hf(Ns,[],Ns).
lgt_hf(Ns,[e(X,Y,I)|Es],[X-Y/I|Hs]) :-
   delete(Ns,X,Ns1),
   delete(Ns1,Y,Ns2),
   lgt_hf(Ns2,Es,Hs).

gt_hf(Ns,[],Ns).
gt_hf(Ns,[e(X,Y)|Es],[X-Y|Hs]) :-
   delete(Ns,X,Ns1),
   delete(Ns1,Y,Ns2),
   gt_hf(Ns2,Es,Hs).

ldt_hf(Ns,[],Ns).
ldt_hf(Ns,[a(X,Y,I)|As],[X>Y/I|Hs]) :-
   delete(Ns,X,Ns1),
   delete(Ns1,Y,Ns2),
   ldt_hf(Ns2,As,Hs).

dt_hf(Ns,[],Ns).
dt_hf(Ns,[a(X,Y)|As],[X>Y|Hs]) :-
   delete(Ns,X,Ns1),
   delete(Ns1,Y,Ns2),
   dt_hf(Ns2,As,Hs).

human_to_gterm(HF,digraph(Ns,As)) :- memberchk(_>_,HF), !, 
   hf_dt(HF,Ns1,As1), sort(Ns1,Ns), sort(As1,As).
human_to_gterm(HF,graph(Ns,Es)) :- 
   hf_gt(HF,Ns1,Es1), sort(Ns1,Ns), sort(Es1,Es).
   
hf_gt([],[],[]).
hf_gt([X-Y/I|Hs],[X,Y|Ns],[e(U,V,I)|Es]) :- !, 
   sort0([X,Y],[U,V]), hf_gt(Hs,Ns,Es).
hf_gt([X-Y|Hs],[X,Y|Ns],[e(U,V)|Es]) :- !,
   sort0([X,Y],[U,V]), hf_gt(Hs,Ns,Es).
hf_gt([H|Hs],[H|Ns],Es) :- hf_gt(Hs,Ns,Es).

hf_dt([],[],[]).
hf_dt([X>Y/I|Hs],[X,Y|Ns],[a(X,Y,I)|As]) :- !, 
   hf_dt(Hs,Ns,As).
hf_dt([X>Y|Hs],[X,Y|Ns],[a(X,Y)|As]) :- !,
   hf_dt(Hs,Ns,As).
hf_dt([H|Hs],[H|Ns],As) :-  hf_dt(Hs,Ns,As).

sort0([X,Y],[X,Y]) :- X @=< Y, !.
sort0([X,Y],[Y,X]) :- X @> Y.

path(G,A,B,P) :- path1(G,A,[B],P).

path1(_,A,[A|P1],[A|P1]).
path1(G,A,[Y|P1],P) :- 
   adjacent(X,Y,G), \+ memberchk(X,[Y|P1]), path1(G,A,[X,Y|P1],P).

adjacent(X,Y,graph(_,Es)) :- member(e(X,Y),Es).
adjacent(X,Y,graph(_,Es)) :- member(e(Y,X),Es).
adjacent(X,Y,graph(_,Es)) :- member(e(X,Y,_),Es).
adjacent(X,Y,graph(_,Es)) :- member(e(Y,X,_),Es).
adjacent(X,Y,digraph(_,As)) :- member(a(X,Y),As).
adjacent(X,Y,digraph(_,As)) :- member(a(X,Y,_),As).
%! \begin{hint} 
depth_first_order(Graph,Start,Seq) :- sorry.
%! \end{hint} 
%! \begin{solution} 
depth_first_order(Graph,Start,Seq) :- 
   (Graph = graph(Ns,_), !; Graph = digraph(Ns,_)),
   memberchk(Start,Ns),
   clear_rdb(dfo),
   recorda(dfo,Start,_),
   (dfo(Graph,Start); true),
   bagof(X,recorded(dfo,X,_),Seq).

dfo(Graph,X) :-
   adjacent(X,Y,Graph), 
   \+ recorded(dfo,Y,_),
   recordz(dfo,Y,_),
   dfo(Graph,Y).

clear_rdb(Key) :-
   recorded(Key,_,Ref), erase(Ref), fail.
clear_rdb(_).
%! \end{solution}
?- depth_first_order(graph([a,b,c,d,e,f,g,h],[e(a,e),e(a,b),e(a,d),e(b,c),e(b,e),e(c,e),e(e,d),e(d,f),e(e,h),e(d,g),e(f,g),e(g,h)]),h,A).

(Intermediate ⭐️⭐️) Connected components. Write a predicate that splits a graph into its connected components.
:- module(_, _, [assertions, datafacts]). 

:- push_prolog_flag(multi_arity_warnings, off).

:- use_module(library(idlists), [memberchk/2, subtract/3]).
:- use_module(library(sets),[ord_subset/2]).
:- use_module(library(sort)).
:- use_module(engine(hiord_rt),[call/2]).
:- use_module(library(aggregates), [findall/3]).
:- use_module(library(classic/classic_predicates), [delete/3]).

:- data edge/2.
:- data arc/2.

:- test connected_components(G,Gs) : (G = graph([],[])) => (Gs = []) + (not_fails, is_det).

:- test connected_components(G,Gs) : (G = graph([a,b,c,d,e,f,g,h],[e(a,e),e(a,b),e(a,d),e(b,c),e(b,e),e(c,e),e(e,d),e(d,f),e(e,h),e(d,g),e(f,g),e(g,h)])) => (Gs = [graph([a,b,c,d,e,f,g,h],[e(a,e),e(a,b),e(a,d),e(b,c),e(b,e),e(c,e),e(e,d),e(d,f),e(e,h),e(d,g),e(f,g),e(g,h)])]) + (not_fails, is_det).

:- test connected_components(G,Gs) : (G = graph([a,b,c,d,e,f,g,h],[e(a,e),e(a,b),e(a,d),e(b,c),e(b,e),e(c,e),e(e,d),e(d,f),e(g,h)])) => (Gs = [graph([a,b,c,d,e,f],[e(a,e),e(a,b),e(a,d),e(b,c),e(b,e),e(c,e),e(e,d),e(d,f)]),graph([g,h],[e(g,h)])]) + (not_fails, is_det).

sorry :- throw(not_solved_yet). 

alist_gterm(Type,AL,GT):- nonvar(GT), !, gterm_to_alist(GT,Type,AL).
alist_gterm(Type,AL,GT):- atom(Type), nonvar(AL), alist_to_gterm(Type,AL,GT).

gterm_to_alist(graph(Ns,Es),graph,AL) :- memberchk(e(_,_,_),Es), ! ,
   lgt_al(Ns,Es,AL).
gterm_to_alist(graph(Ns,Es),graph,AL) :- !, 
   gt_al(Ns,Es,AL).
gterm_to_alist(digraph(Ns,As),digraph,AL) :- memberchk(a(_,_,_),As), !,
   ldt_al(Ns,As,AL).
gterm_to_alist(digraph(Ns,As),digraph,AL) :- 
   dt_al(Ns,As,AL).

lgt_al([],_,[]).
lgt_al([V|Vs],Es,[n(V,L)|Ns]) :-
   findall(T,((member(e(X,V,I),Es) ; member(e(V,X,I),Es)),T = X/I),L),
   lgt_al(Vs,Es,Ns).

gt_al([],_,[]).
gt_al([V|Vs],Es,[n(V,L)|Ns]) :-
   findall(X,(member(e(X,V),Es) ; member(e(V,X),Es)),L), gt_al(Vs,Es,Ns).

ldt_al([],_,[]).
ldt_al([V|Vs],As,[n(V,L)|Ns]) :-
   findall(T,(member(a(V,X,I),As), T=X/I),L), ldt_al(Vs,As,Ns).

dt_al([],_,[]).
dt_al([V|Vs],As,[n(V,L)|Ns]) :-
   findall(X,member(a(V,X),As),L), dt_al(Vs,As,Ns).

alist_to_gterm(graph,AL,graph(Ns,Es)) :- !, al_gt(AL,Ns,EsU,[]), sort(EsU,Es).
alist_to_gterm(digraph,AL,digraph(Ns,As)) :- al_dt(AL,Ns,AsU,[]), sort(AsU,As).

al_gt([],[],Es,Es).
al_gt([n(V,Xs)|Ns],[V|Vs],Es,Acc) :- 
   add_edges(V,Xs,Acc1,Acc), al_gt(Ns,Vs,Es,Acc1). 

add_edges(_,[],Es,Es).
add_edges(V,[X/_|Xs],Es,Acc) :- V @> X, !, add_edges(V,Xs,Es,Acc).
add_edges(V,[X|Xs],Es,Acc) :- V @> X, !, add_edges(V,Xs,Es,Acc).
add_edges(V,[X/I|Xs],Es,Acc) :- V @=< X, !, add_edges(V,Xs,Es,[e(V,X,I)|Acc]).
add_edges(V,[X|Xs],Es,Acc) :- V @=< X, add_edges(V,Xs,Es,[e(V,X)|Acc]).

al_dt([],[],As,As).
al_dt([n(V,Xs)|Ns],[V|Vs],As,Acc) :- 
   add_arcs(V,Xs,Acc1,Acc), al_dt(Ns,Vs,As,Acc1). 

add_arcs(_,[],As,As).
add_arcs(V,[X/I|Xs],As,Acc) :- !, add_arcs(V,Xs,As,[a(V,X,I)|Acc]).
add_arcs(V,[X|Xs],As,Acc) :- add_arcs(V,Xs,As,[a(V,X)|Acc]).

ecl_to_gterm(GT) :-
   findall(E,(edge(X,Y),E=X-Y),Es), human_gterm(Es,GT).

acl_to_gterm(GT) :-
   findall(A,(arc(X,Y),A= >(X,Y)),As), human_gterm(As,GT).
   
human_gterm(HF,GT):- nonvar(GT), !, gterm_to_human(GT,HF).
human_gterm(HF,GT):- nonvar(HF), human_to_gterm(HF,GT).

gterm_to_human(graph(Ns,Es),HF) :-  memberchk(e(_,_,_),Es), !, 
   lgt_hf(Ns,Es,HF).
gterm_to_human(graph(Ns,Es),HF) :-  !, 
   gt_hf(Ns,Es,HF).
gterm_to_human(digraph(Ns,As),HF) :- memberchk(a(_,_,_),As), !, 
   ldt_hf(Ns,As,HF).
gterm_to_human(digraph(Ns,As),HF) :- 
   dt_hf(Ns,As,HF).

lgt_hf(Ns,[],Ns).
lgt_hf(Ns,[e(X,Y,I)|Es],[X-Y/I|Hs]) :-
   delete(Ns,X,Ns1),
   delete(Ns1,Y,Ns2),
   lgt_hf(Ns2,Es,Hs).

gt_hf(Ns,[],Ns).
gt_hf(Ns,[e(X,Y)|Es],[X-Y|Hs]) :-
   delete(Ns,X,Ns1),
   delete(Ns1,Y,Ns2),
   gt_hf(Ns2,Es,Hs).

ldt_hf(Ns,[],Ns).
ldt_hf(Ns,[a(X,Y,I)|As],[X>Y/I|Hs]) :-
   delete(Ns,X,Ns1),
   delete(Ns1,Y,Ns2),
   ldt_hf(Ns2,As,Hs).

dt_hf(Ns,[],Ns).
dt_hf(Ns,[a(X,Y)|As],[X>Y|Hs]) :-
   delete(Ns,X,Ns1),
   delete(Ns1,Y,Ns2),
   dt_hf(Ns2,As,Hs).

human_to_gterm(HF,digraph(Ns,As)) :- memberchk(_>_,HF), !, 
   hf_dt(HF,Ns1,As1), sort(Ns1,Ns), sort(As1,As).
human_to_gterm(HF,graph(Ns,Es)) :- 
   hf_gt(HF,Ns1,Es1), sort(Ns1,Ns), sort(Es1,Es).
   
hf_gt([],[],[]).
hf_gt([X-Y/I|Hs],[X,Y|Ns],[e(U,V,I)|Es]) :- !, 
   sort0([X,Y],[U,V]), hf_gt(Hs,Ns,Es).
hf_gt([X-Y|Hs],[X,Y|Ns],[e(U,V)|Es]) :- !,
   sort0([X,Y],[U,V]), hf_gt(Hs,Ns,Es).
hf_gt([H|Hs],[H|Ns],Es) :- hf_gt(Hs,Ns,Es).

hf_dt([],[],[]).
hf_dt([X>Y/I|Hs],[X,Y|Ns],[a(X,Y,I)|As]) :- !, 
   hf_dt(Hs,Ns,As).
hf_dt([X>Y|Hs],[X,Y|Ns],[a(X,Y)|As]) :- !,
   hf_dt(Hs,Ns,As).
hf_dt([H|Hs],[H|Ns],As) :-  hf_dt(Hs,Ns,As).

sort0([X,Y],[X,Y]) :- X @=< Y, !.
sort0([X,Y],[Y,X]) :- X @> Y.

path(G,A,B,P) :- path1(G,A,[B],P).

path1(_,A,[A|P1],[A|P1]).
path1(G,A,[Y|P1],P) :- 
   adjacent(X,Y,G), \+ memberchk(X,[Y|P1]), path1(G,A,[X,Y|P1],P).

adjacent(X,Y,graph(_,Es)) :- member(e(X,Y),Es).
adjacent(X,Y,graph(_,Es)) :- member(e(Y,X),Es).
adjacent(X,Y,graph(_,Es)) :- member(e(X,Y,_),Es).
adjacent(X,Y,graph(_,Es)) :- member(e(Y,X,_),Es).
adjacent(X,Y,digraph(_,As)) :- member(a(X,Y),As).
adjacent(X,Y,digraph(_,As)) :- member(a(X,Y,_),As).
%! \begin{hint} 
connected_components(G,Gs) :- sorry.
  % Gs is the list of the connected components
  % of the graph G (only for graphs, not for digraphs!)
  % (gterm, list-of-gterms), (+,-)
%! \end{hint} 
%! \begin{solution} 
connected_components(graph([],[]),[]) :- !.
connected_components(graph(Ns,Es),[graph(Ns1,Es1)|Gs]) :-
   Ns = [N|_],
   component(graph(Ns,Es),N,graph(Ns1,Es1)),
   subtract(Ns,Ns1,NsR),
   subgraph(graph(Ns,Es),graph(NsR,EsR)),
   connected_components(graph(NsR,EsR),Gs).

component(graph(Ns,Es),N,graph(Ns1,Es1)) :-
   Pred =..[is_path,graph(Ns,Es),N],
   include(Pred,Ns,Ns1),
   subgraph(graph(Ns,Es),graph(Ns1,Es1)).

is_path(Graph,A,B) :- path(Graph,A,B,_).

% subgraph(G,G1) :- G1 is a subgraph of G
subgraph(graph(Ns,Es),graph(Ns1,Es1)) :-
   ord_subset(Ns1,Ns),
   Pred =.. [edge_is_compatible,Ns1],
   include(Pred,Es,Es1).

include(_,[],[]).
include(P,[X|Xs],[X|S]) :- 
  call(P,X), !,
  include(P,Xs,S).
include(P,[_|Xs],S) :- 
  include(P,Xs,S).

edge_is_compatible(Ns1,Z) :- 
   (Z = e(X,Y),!; Z = e(X,Y,_)),
   memberchk(X,Ns1), 
   memberchk(Y,Ns1). 
%! \end{solution}
?- connected_components(graph([a,b,c,d,e,f,g,h],[e(a,e),e(a,b),e(a,d),e(b,c),e(b,e),e(c,e),e(e,d),e(d,f),e(g,h)]),A).

Miscellaneous Problems

Eight queens problem

(Intermediate ⭐️⭐️) Eight queens problem. This is a classical problem in computer science. The objective is to place eight queens on a chessboard so that no two queens are attacking each other; i.e., no two queens are in the same row, the same column, or on the same diagonal.
:- module(_,_,[assertions]).

:- push_prolog_flag(multi_arity_warnings, off).

:- use_module(library(idlists), [memberchk/2]).

:- test queens_1(A,B) : (A = 4) => (B = [2,4,1,3]; B = [3,1,4,2]) + (not_fails, num_solutions(2)).
sorry :- throw(not_solved_yet). 

%! \begin{hint}
queens_1(N,Qs) :- sorry.
  % Qs is a solution of the N-queens problem
  
range(A,B,L) :- sorry.
  % L is the list of numbers A..B

permu(Xs,Zs) :- sorry.
  % the list Zs is a permutation of the list Xs

test(Qs) :- sorry.
  % the list Qs represents a non-attacking queens solution

test(Qs,X,Cs,Ds) :- sorry.
  % the queens in Qs, representing columns X to N,
  % are not in conflict with the diagonals Cs and Ds
%! \end{hint}
%! \begin{solution}
% The first version is a simple generate-and-test solution.

queens_1(N,Qs) :- range(1,N,Rs), permu(Rs,Qs), test(Qs).

range(A,A,[A]).
range(A,B,[A|L]) :- A < B, A1 is A+1, range(A1,B,L).

permu([],[]).
permu(Qs,[Y|Ys]) :- del(Y,Qs,Rs), permu(Rs,Ys).

del(X,[X|Xs],Xs).
del(X,[Y|Ys],[Y|Zs]) :- del(X,Ys,Zs).

test(Qs) :- test(Qs,1,[],[]).

test([],_,_,_).
test([Y|Ys],X,Cs,Ds) :- 
        C is X-Y, \+ memberchk(C,Cs),
        D is X+Y, \+ memberchk(D,Ds),
        X1 is X + 1,
        test(Ys,X1,[C|Cs],[D|Ds]).

%--------------------------------------------------------------

% Now, in version 2, the tester is pushed completely inside the
% generator permu.

queens_2(N,Qs) :- range(1,N,Rs), permu_test(Rs,Qs,1,[],[]).

permu_test([],[],_,_,_).
permu_test(Qs,[Y|Ys],X,Cs,Ds) :- 
        del(Y,Qs,Rs), 
        C is X-Y, \+ memberchk(C,Cs),
        D is X+Y, \+ memberchk(D,Ds),
        X1 is X+1,
        permu_test(Rs,Ys,X1,[C|Cs],[D|Ds]).
%! \end{solution}
?- queens_1(8,A).
Hint: Represent the positions of the queens as a list of numbers 1..N. Example: [4,2,7,3,6,8,5,1] means that the queen in the first column is in row 4, the queen in the second column is in row 2, etc. Use the generate-and-test paradigm.

Knight's tour

(Intermediate ⭐️⭐️) Knight's tour. Another famous problem is this one: How can a knight jump on an NxN chessboard in such a way that it visits every square exactly once?
:- module(_,_,[assertions]).

:- push_prolog_flag(multi_arity_warnings, off).

:- use_module(library(idlists), [memberchk/2]).

:- test knights(A,B) : (A = 5) => (B = [1/5,3/4,5/5,4/3,5/1,3/2,1/3,2/5,4/4,5/2,3/1,1/2,2/4,4/5,5/3,4/1,2/2,1/4,3/3,2/1,4/2,5/4,3/5,2/3,1/1]) + (not_fails, num_solutions(2)).

sorry :- throw(not_solved_yet). 

%! \begin{hint}
knights(N,Knights) :- sorry.
  % Knights is a knight's tour on a NxN chessboard 
  
closed_knights(N,Knights) :- sorry.
  % Knights is a knight's tour on a NxN 
  % chessboard which ends at the same square where it began.
  
knights(N,M,Visited,Knights) :- sorry.
  % the list of squares Visited must be
  % extended by M further squares to give the solution Knights of the
  % NxN chessboard knight's tour problem. 
%! \end{hint}
%! \begin{solution}

knights(N,Knights) :- M is N*N-1,  knights(N,M,[1/1],Knights).

closed_knights(N,Knights) :- 
   knights(N,Knights), Knights = [X/Y|_], jump(N,X/Y,1/1). 

knights(_,0,Knights,Knights).
knights(N,M,Visited,Knights) :-
   Visited = [X/Y|_],
   jump(N,X/Y,U/V),
   \+ memberchk(U/V,Visited),
   M1 is M-1,
   knights(N,M1,[U/V|Visited],Knights).

% jumps on an NxN chessboard from square A/B to C/D
jump(N,A/B,C/D) :- 
   jump_dist(X,Y), 
   C is A+X, C > 0, C =< N,
   D is B+Y, D > 0, D =< N.

% jump distances
jump_dist(1,2).
jump_dist(2,1).
jump_dist(2,-1).
jump_dist(1,-2).
jump_dist(-1,-2).
jump_dist(-2,-1).
jump_dist(-2,1).
jump_dist(-1,2).
%! \end{solution}
?- knights(5,A).
Hints: Represent the squares by pairs of their coordinates of the form X/Y, where both X and Y are integers between 1 and N. (Note that / is just a convenient functor, not division!) Define the relation jump(N,X/Y,U/V) to express the fact that a knight can jump from X/Y to U/V on a NxN chessboard. And finally, represent the solution of our problem as a list of N*N knight positions (the knight's tour).

Arithmetic puzzle

(Hard ⭐️⭐️⭐️) An arithmetic puzzle. Given a list of integer numbers, find a correct way of inserting arithmetic signs (operators) such that the result is a correct equation. Example: With the list of numbers [2,3,5,7,11] we can form the equations or (and ten others!).

:- module(_,_,[assertions]).

:- use_module(library(streams),[display/1,nl/0]).
:- use_module(library(classic/classic_predicates),[append/3]).

:- push_prolog_flag(multi_arity_warnings, off).

sorry :- throw(not_solved_yet). 

%! \begin{hint}
equation(L,LT,RT) :- sorry.
  % L is the list of numbers which are the leaves
  % in the arithmetic terms LT and RT - from left to right. The 
  % arithmetic evaluation yields the same result for LT and RT.
%! \end{hint}
%! \begin{solution}
equation(L,LT,RT) :-
   split(L,LL,RL),              % decompose the list L
   term(LL,LT),                 % construct the left term
   term(RL,RT),                 % construct the right term
   LT =:= RT.                   % evaluate and compare the terms

% term(L,T) :- L is the list of numbers which are the leaves in
%    the arithmetic term T - from left to right.

term([X],X).                    % a number is a term in itself
% term([X],-X).                   % unary minus
term(L,T) :-                    % general case: binary term
   split(L,LL,RL),              % decompose the list L
   term(LL,LT),                 % construct the left term
   term(RL,RT),                 % construct the right term
   binterm(LT,RT,T).            % construct combined binary term

% binterm(LT,RT,T) :- T is a combined binary term constructed from
%    left-hand term LT and right-hand term RT

binterm(LT,RT,LT+RT).
binterm(LT,RT,LT-RT).
binterm(LT,RT,LT*RT).
binterm(LT,RT,LT/RT) :- RT =\= 0.   % avoid division by zero

% split(L,L1,L2) :- split the list L into non-empty parts L1 and L2
%    such that their concatenation is L

split(L,L1,L2) :- append(L1,L2,L), L1 = [_|_], L2 = [_|_].

% do(L) :- find all solutions to the problem as given by the list of
%    numbers L, and print them out, one solution per line.

do(L) :- 
   equation(L,LT,RT),
      display(LT), display(' = '), display(RT), nl,
   fail.
do(_).
%! \end{solution}
?- do([2,3,5,7,11]).

K-regular simple graphs

(Hard ⭐️⭐️⭐️) Generate K-regular simple graphs with N nodes. In a K-regular graph all nodes have a degree of K; i.e. the number of edges incident in each node is K. How many (non-isomorphic!) 3-regular graphs with 6 nodes are there?

Since it is a very long exercise, open a playground tab and write your code! Click here to see the answer!

English number words

(Intermediate ⭐️⭐️) English number words. On financial documents, like cheques, numbers must sometimes be written in full words. Example: 175 must be written as one-seven-five. Write a predicate full_words/1 to print (non-negative) integer numbers in full words.
:- module(_,_,[assertions]).

:- use_module(library(streams),[display/1, nl/0]).

sorry :- throw(not_solved_yet). 

%! \begin{hint}
full_words(N) :- sorry.
  % print the number N in full words (English)
  % (non-negative integer) (+)
%! \end{hint}
%! \begin{solution}

full_words(0) :- !, display(zero), nl.
full_words(N) :- integer(N), N > 0, full_words1(N), nl.

full_words1(0) :- !.
full_words1(N) :- N > 0,
   Q is N // 10, R is N mod 10,
   full_words1(Q), numberword(R,RW), hyphen(Q), display(RW).

hyphen(0) :- !.
hyphen(Q) :- Q > 0, display('-'). 

numberword(0,zero).
numberword(1,one).
numberword(2,two).
numberword(3,three).
numberword(4,four).
numberword(5,five).
numberword(6,six).
numberword(7,seven).
numberword(8,eight).
numberword(9,nine).
%! \end{solution}
?- full_words(N).

Nonograms

(Hard ⭐️⭐️⭐️) Nonograms. Around 1994, a certain kind of puzzles was very popular in England. The "Sunday Telegraph" newspaper wrote: "Nonograms are puzzles from Japan and are currently published each week only in The Sunday Telegraph. Simply use your logic and skill to complete the grid and reveal a picture or diagram." As a Prolog programmer, you are in a better situation: you can have your computer do the work! Just write a little program ;-).

The puzzle goes like this: Essentially, each row and column of a rectangular bitmap is annotated with the respective lengths of its distinct strings of occupied cells. The person who solves the puzzle must complete the bitmap given only these lengths.

Problem statement:          Solution:

|_|_|_|_|_|_|_|_| 3         |_|X|X|X|_|_|_|_| 3           
|_|_|_|_|_|_|_|_| 2 1       |X|X|_|X|_|_|_|_| 2 1         
|_|_|_|_|_|_|_|_| 3 2       |_|X|X|X|_|_|X|X| 3 2         
|_|_|_|_|_|_|_|_| 2 2       |_|_|X|X|_|_|X|X| 2 2         
|_|_|_|_|_|_|_|_| 6         |_|_|X|X|X|X|X|X| 6           
|_|_|_|_|_|_|_|_| 1 5       |X|_|X|X|X|X|X|_| 1 5         
|_|_|_|_|_|_|_|_| 6         |X|X|X|X|X|X|_|_| 6           
|_|_|_|_|_|_|_|_| 1         |_|_|_|_|X|_|_|_| 1           
|_|_|_|_|_|_|_|_| 2         |_|_|_|X|X|_|_|_| 2           
 1 3 1 7 5 3 4 3             1 3 1 7 5 3 4 3              
 2 1 5 1                     2 1 5 1                      
For the example above, the problem can be stated as the two lists [[3],[2,1],[3,2],[2,2],[6],[1,5],[6],[1],[2]] and [[1,2],[3,1],[1,5],[7,1],[5],[3],[4],[3]] which give the "solid" lengths of the rows and columns, top-to-bottom and left-to-right, respectively. Published puzzles are larger than this example, e.g. 25 x 20, and apparently always have unique solutions.

Since it is a very long exercise, open a playground tab and write your code! Click here to see the answer!